Download as pdf or txt
Download as pdf or txt
You are on page 1of 80

MATEMATICKOFIZI

CKI
LIST (MFL)
za ucenike i nastavnike.
Izlazi u cetiri broja tokom s kolske godine.
Izdaju:

HRVATSKO MATEMATICKO
DRUSTVO
i

HRVATSKO FIZIKALNO DRUSTVO


Pretplata za 2008./ 2009. je 100 kn, pojedini broj
25 kn.
Za inozemstvo pretplata je 28 EUR, a pojedini broj
7 EUR.
(Uplata se moze obaviti u kunama ili devizama po
tecaju u trenutku placanja.)
Adresa lista je:
Matematickofizicki list, Bijenicka 32, 10001
Zagreb, tel. (01) 4833-891, fax (01) 4683-535.
Uplate na z iro racun:
Hrvatsko fizikalno drustvo, Zagreb,
br. 2360000-1101301202 (kn),
ZBZ d.d. SWIFT ZABA HRXX 70313-978-3239853
(EUR).
Na uplatnici kao svrhu uplate molimo naznacite
za MFL!
Molimo Vas da kod svake uplate posaljete (foto-)
kopiju uplatnice ili da nas obavijestite telefonom
ili elektronskom postom o uplati.
URL: http:/ / web.math.hr/ mfl, e-mail: mfl@hfd.hr
- cki odbor:
Urediva

ZELJKO
HANJS (Zagreb), glavni i odgovorni urednik,
e-mail: hanjs@math.hr
MATKO MILIN (Zagreb), urednik za fiziku,
e-mail: matkom@phy.hr
C (Split), IGOR GASPARI

ZDRAVKO
ANTE BILUSI
C,
MIRKO
KURNIK, VLADIMIR PAAR, MAJA PLANINIC,

POLONIJO, DUBRAVKA SALOPEK WEBER, SASA

SINGER, ANA SMONTARA, BOSKO


SEGO,
VLADIMIR
tajnica SANDRA
VOLENEC, MLADEN VUKOVIC,

POZAR
(Zagreb), e-mail: sandra@phy.hr
Izdavacki savjet:
ALEKSA BJELIS (Zagreb), LIDIJA COLOMBO (Zagreb),
BRANIMIR DAKIC (Zagreb), VLADIMIR DEVIDE (Zagreb), MARIJAN HUSAK (Varazdin), MARGITA PA STAR

VLEKOVIC (Osijek), ERNA SU


(Zagreb), PETAR
VRANJKOVIC (Zadar), VLADIS VUJNOVIC (Zagreb),

PASKO ZUPANOVIC (Split)


List financijski pomaze Ministarstvo znanosti, obrazovanja
i s porta Republike Hrvatske.
Slog i prijelom:
Element, Zagreb, Menceticeva 2
Tisak:
Tiskara Zelina d.d., Sv. Ivan Zelina, Ul. K. Krizmanic 1
Naklada ovog broja 3000 primjeraka
Slika na naslovnici prikazuje detektor CMS na CERN-u u
fazi sklapanja.

SADRZAJ
Matematika

Zeljko
Hanjs,
Soroban japanski abak . . . . . . . . . 67
zmesija, Maja Kozulic, Bosko Sego

Mirjana Ci
Primjena osnovne analize vremenskog
niza u poslovanju fondova . . . . . . . . 69
Marko Juraic,
Povijest matrica i determinanti . . . . . . . 76
Roko Pesic,
S razredbenog ispita u Japanu . . . . . . . 80
Petar Vranjkovic,
Jedno svojstvo logaritamske funkcije
i njegova primjena . . . . . . . . . . . . 82
Fizika
Tome Anticic i Vuko Brigljevic,
LHC: zasto mu se divimo? . . . . . . . . 86
Ljiljana Sudar,
Geometrija i problemi kretanja . . . . . . . 95
Astronomija
Dario Hrupec,
Teleskop MAGIC c arobni instrument
astrocesticne fizike . . . . . . . . . . . . 102
Zabavna matematika . . . . . . . . . . 105
Zadaci i rjesenja
A) Zadaci iz matematike . . . . . . . . . 106
B) Zadaci iz fizike . . . . . . . . . . . . 106
C) Rjesenja iz matematike . . . . . . . . . 107
D) Rjesenja iz fizike . . . . . . . . . . . 112
Zanimljivosti
Sanja Varosanec, Matematicka konferencija
u c ast akademiku Josipu Pecaricu . . . . . 117
49. medunarodna
matematicka olimpijada . . 120
Druga srednjoeuropska matematicka olimpijada,
ska, 4. 10. rujna 2008. . . . 123
Olomouc, Ce
Medunarodno
matematicko natjecanje
Klokan bez granica 2008. g. . . . . . . 126
Nove knjige

Zarko
Dadic,
Egzaktne znanosti u Hrvata u
poslijeprosvjetiteljskom razdoblju
(1789.1835.) . . . . . . . . . . . . . . 136
Kvalifikacijski ispiti
Razredbeni ispit na FOI-u iz
matematike 2008. g. . . . . . . . . . . . 138
Nagradni natjecaj br. 185 . . . . . . . . 144

Dragi c itatelji!

Pocetkom jeseni ove godine po cijelom je svijetu odjeknula vijest iz CERN-a o


pustanju u rad Velikog hadronskog sudarivaca, LHC (engl. Large Hadron Collider)
za ispitivanje materije. Ocekuju se sudari s najvisim energijama cestica koje se krecu
skoro brzinama svjetlosti. Protoni i druge cestice krecu se u tunelu opsega 27 km 100
metara ispod povrsine Zemlje. Eksperimenti ukljucuju vise tisuca znanstvenika iz cijelog
svijeta, a trajat ce sljedecih 10 do 20 godina, u nastojanju da se odgonetne s to vise tajni
svemira. U taj pothvat ukljuceno je i desetak hrvatskih fizicara. O ovoj zanimljivoj temi
upoznaju nas Tome Anticic i Vuko Brigljevic, visi znanstveni suradnici u Zavodu za
- Boskovic u Zagrebu.
eksperimentalnu fiziku Instituta Ruder
U prilogu profesorice Ljiljane Sudar iz Leskovca, Geometrija i problemi kretanja,
pokazuje kako se mnogi problemi kretanja efikasno i elegantno rjesavaju pomocu
nestandarne i veoma mocne metode, koja se bazira na primjeni grafova i geometrije.
U proslosti, ne tako davnoj, koristila su se razna racunala u svakodnevnom z ivotu.
Ovdje se, u prilogu Soroban japanski abak, samo prisjecamo ruskog, kineskog i
japanskog abaka. Ucenicima i nastavnicima ekonomskih s kola namijenjen je clanak
Primjena osnovne analize vremenskog niza u poslovanju fondova, koji su priredili
zmesija, Maja Kozulic i Bosko Sego

Mirjana Ci
s Ekonomskog fakulteta u Zagrebu.
U prilogu Povijest matrica i determinanti Marka Juraica iz Zagreba dan je ukratko
povijesni prikaz ove grane matematike. Roko Pesic navodi jedan zanimljiv zadatak
koji se pojavio na razredbeom ispitu na jednom sveucilistu u Japanu. Profesor Petar
Vranjkovic s gimnazije u Zadru opisuje jedno zanimljivo svojstvo logaritamske funkcije
i njegovu primjenu u rjesavanju raznih nejednakosti.
Dario Hrupec iz Koprivnice u prilogu Teleskop MAGIC c arobni instrument
astrocesticne fizike opisuje novo, interdisciplinarno, znanstvenoistrazivacko podrucje
koje ukljucuje fiziku c estica, astronomiju, astrofiziku te kozmologiju.
U Trogiru je odrzana matematicka konferencija u c ast akademiku Josipu Pecaricu,
svjetski poznatom matematicaru iz podrucja matematickih nejednakosti, povodom
njegovog 60-tog rodendana.
Tu su jos izvjesc a za zadacima s 49. medunarodne
matematicke olimpijade, Druge srednjoeuropske matematicke olimpijade i Medunarodnog
matematickog natjecanja Klokan bez granica 2008. g.

Akademik Zarko
Dadic, autor mnogih knjiga iz fizike i matematike objavio je jos
jednu, Egzaktne znanosti u Hrvata u poslijeprosvjetiteljskom razdoblju (1789.1835.).
Na zadnjoj strani omota prisjetili smo se redovitog profesora PMF-a Matematickog
odjela, znanstvenika-geometricara, metodicara, autora udzbenika i popularizatora
matematike.
Urednistvo lista

66

Matematicko-fizicki list, LIX 2 (2008. 2009.)

Soroban japanski abak

Zeljko
Hanjs, Zagreb
Abak je naziv za napravu koja je sluzila za racunanje s
vecim brojevima, a tokom povijesti konstruirane su razlicite
njegove vrste. Spomenut cemo tri najpoznatija abaka: ruski,
kineski i japanski. Pravokutnog su oblika, napravljeni od
drveta, sa z icama ili pruticima za kuglice. Kineski i japanski
abak imaju neparan broj prutica, a moze ih biti i preko 30.
Ruski abak ima deset z ica s po deset kuglica, kao na slici 1.
Engleska rijec abacus etimoloski je izvedena iz grcke
rijeci o , koja oznacava dascice za racunanje
pokrivenu pijeskom, a ova potjece od semitske rijeci koja
ima znacenje dascicu za racunanje pokrivenoj prasinom ili
pijeskom. U grcko i rimsko doba koristili su kuglice ili
zrnca od kamena ili metala.

Slika 2. Kineski abak

Slika 1. Ruski abak

Kineski abak ima svoje porijeklo


u rimskom abaku. Datira iz sredine
16. stoljeca, imao je sedam kuglica na
svakom pruticu, dvije u gornjem i pet
u donjem dijelu, kao s to je prikazano
na slici 2. Prenesen u Japan negdje u
17. stoljecu postao je vrlo popularnom
pomocnom napravom za racunanje u
trgovinama. U Japanu su racionalizirali
ovaj abak odbacujuci suvisne kuglice.

Oko godine 1920. japanski abak, koji se tamo nazviva soroban, poprimio je novu
konfiguraciju. Napravljen je od drveta, najcesc e hrasta, i na svakom pruticu, obicno od
bambusa, ima po pet kuglica, jednu u gornjem i c etiri u donjem dijelu ploce. Pokazalo
se da je to najracionalnije, jer se kod obavljanja operacija na kineskom abaku nikad ne
mora koristiti krajnja donja niti krajnja gornja kuglica. Blaise Pascal, veliki francuski
- prvima je konstruirao nekoliko modernijih racunskih
matematicar, u 17. stoljecu medu
strojeva.

Slika 3. Japanski abak

Japanski abak je jednostavna naprava koja omogucuje brzo obavljanje aritmetickih


racunskih operacija, a bazira se na decimalnom sistemu. Svako od 4 zrnca na donjem
Matematicko-fizicki list, LIX 2 (2008. 2009.)

67

dijelu prutica abaka ima vrijednost 1, a zrnce na njegovom gornjem dijelu ima vrijednost
5. U neutralnom polozaju, kada su donja zrnca pomaknuta do kraja dolje, a gornje do
kraja gore, njegova vrijednost je jednaka nuli. Tek pomakom zrnca prema sredini ono
poprima odredenu
vrijednost, 1 ili 5.
Prije pojave elektronickih racunala/ kalkulatora soroban je u Japanu bio u s irokoj
upotrebi. Bio je vrlo koristan u s kolama za ucenje osnovnih racunskih operacija.

Stovi
se, koristenjem sorobana razvijale su se i sposobnosti mentalnog racunanja. Tada
- da ce abak u buducnosti imati vrlo vaznu ulogu, ne samo u svakodnevnom
se predvidalo
z ivotu.
Jos prije tridesetak godina soroban je bio u velikoj upotrebi, ali su ga potiskivali
kalkulatori i danas se vise gotovo i ne koristi. U Japanu se vise ne proizvode klasicni
sorobani i zadrzali su se jos samo kao neizbjezni turisticki suveniri sa samo do desetak
prutica, najcesc e od plastike. Ipak uz dosta truda moze se tek tu i tamo u trgovini
naci i klasicni soroban s 23 prutica. Po velicini sorobani se dijele na male (s 13
prutica), srednje (s 15 prutica) i velike (s 23 prutica). Inace ih ima i s po 27, pa cak
i s 31 pruticem, ali su najcesci bili oni s 21 pruticem jer su uglavnom zadovoljavali
svakodnevne potrebe. U pravilu broj prutica mora biti neparan.
Godine 1954. Takashi Kojima je izdao knjizicu The Japanese Abacus, Its Use
and Theory da bi 1996. izaslo vec njezino 36. izdanje. Tamo je na mnogo primjera
ilustrirano kako se vrse osnovne racunske operacije: zbrajanje, oduzimanje, mnozenje,
- su se odrzavala natjecanja oko 40-tih godina proslog stoljeca izmedu
dijeljenje. Takoder
tadasnjih elektronickih racunala i natjecatelja koji su koristili soroban. Godine 1946.
odrzano je u japanu jedno takvo natjecanje, a pobjeda japanskog abaka bila je tada
jos uvijek neupitna. Tada su strucnjaci za soroban kod zbrajanja i oduzimanja bili brzi
od tadasnjih elektronickih racunala. U Japanu je nekad postojao Institut za istrazivanje
abaka.
S vremenom je abak/ soroban postigao nevjerojatnu popularnost u Japanu. Godine
1965. oko milijun zainteresiranih ljudi je pristupilo ispitivanju vlastitih sposobnosti
racunanja na sorobanu, od kojih je njih c etvrtina polozila ispit (5000 ih je postiglo
odobrenje prvog, 25 000 drugog i 230 000 treceg stupnja).
Abak je bio ukljucen u nastavni program za osnovne s kole kod ucenja elemenata
aritmetike, a izucavao se i za potrebe u trgovini. Ukratko, abak je postao vrlo popularnim
predmetom kojeg je imalo skoro svako domacinstvo.
Kako objasniti iznenadan pad popularnosti nekad neizbjeznog i vrlo popularnog
abaka, u vrijeme modernizacije tokom dvadesetog stoljeca? Bez ikakve dvojbe glavno
objasnjenje je u cinjenici da su operacijske metode kod sve modernijih kalkulatora
postale znatno jednostavnije, savrsenije i brze.
Iako su se u poslovima u Japanu sve vise koristila elektronicka racunala, nije bilo
- ce s to se abak u velikoj mjeri jos dugo koristio, kako u malim, tako i u
iznenaduju
velikim trgovinama.
Jos sredinom proslog stoljeca strucnjaci za abak/ soroban bili su misljenja da ce
vrijeme utroseno na ucenje i vjezbavanje operacija na tom racunalu, biti korisno za
ucenje aritmetike, a ujedno ce biti ogromna pomoc u poslovima i u svakodnevnom
z ivotu.
Abak je izuzetno vazan i kao pomoc slijepima. U japanskim s kolama za slijepe, sve
su se od 1964. g. provodili ispiti
vrste pomagala za racunanje baziraju se na abaku. Cak
racunanja za slijepe osobe.
Obavljanje operacija na sorobanu je potpuno identicno rukovanju s kineskim abakom.
U c asopisu Matka, 1994. br. 10, opisano je ukratko kako se na kineskom abaku vrsi
zbrajanje, oduzimanje i mnozenje, a u Matematicko-fizickom listu, 1994./ 95. br. 2/ 178,
dani su algoritni za racunanje drugog i treceg korijena.
Ipak i ovo je tek prisjecanje na nekad zlatno doba abaka/ sorobana.

68

Matematicko-fizicki list, LIX 2 (2008. 2009.)

Primjena osnovne analize vremenskog niza u poslovanju fondova

zmesija , Maja Kozulic , Bosko Sego

Mirjana Ci
, Zagreb

Uvod
Analiza vremenskih nizova ima s iroko podrucje primjene kako u strucnim tako i u
znanstvenim istrazivanjima. Analiza dinamike promatrane pojave vazna je za donosenje
poslovnih odluka, a moze posluziti i kao podloga za daljnje slozenije statisticke analize.
Vremenski niz je niz kronoloski uredenih
vrijednosti promatrane pojave, a notira se
na sljedeci nacin:
Y : y1 , y2 , ..., yt , ..., yn t = 1, 2, 3, ..., n,
(1)
pri cemu je Y promatrana pojava u vremenu, yt je vrijednost pojave Y u vremenu t
(frekvencija vremenskog niza), a n je duljina vremenskog niza.
Prema nacinu promatranja pojave u vremenu razlikuju se intervalni i trenutacni
vremenski nizovi. Intervalni niz nastaje promatranjem odredene
pojave u odredenim

vremenskim intervalima (godina, tromjesecje, mjesec, dan i sl.). Cesto


se u ekonomiji
promatraju sljedece pojave: godisnja proizvodnja, mjesecni broj prevezenih putnika,
dnevni ostvareni promet, i dr. Intervalni vremenski niz ima svojstvo kumulativnosti
s to znaci da zbroj frekvencija intervalnog niza ima smislenu interpretaciju. Trenutacni
vremenski niz nastaje promatranjem pojave u odredenom
trenutku (npr. stanje na tekucem
racunu, broj zaposlenih i dr.). Trenutacni niz nema svojstvo kumulativnosti.
Vremenski nizovi mogu biti izvorni i izvedeni. U izvornom nizu frekvencije
nastaju izravnim mjerenjem pojave po odabranim intervalima vremena ili u odabranim
vremenskim tockama. Izvedeni vremenski niz c ine frekvencije koje su nastale primjenom
brojcanih operacija nad izvornim vrijednostima jednog ili vise vremenskih nizova.

Graficka analiza vremenskog niza


Veoma korisno sredstvo u analizi vremenskog niza su graficki prikazi. U tablicnim
kalkulatorima kao s to su Excel, Lotus i drugi moguce je konstruirati razlicite vrste
grafickih prikaza. Najcesce se vremenski nizovi prikazuju povrsinskim i linijskim
grafikonima. Za prikaz vremenskog niza koji pokazuje sezonske oscilacije pojave s
vremenom koristi se polarni dijagram. Grafikon mora imati naslov, izvor podataka
(ako se koriste sekundarni podaci) i po potrebi napomene i legendu. Ako se frekvencije
vremenskog niza nalaze na visokoj brojcanoj razini (daleko od nule) i s malim rasponom
varijacije, prikladno je primijeniti horizontalni prekid mjerila (prekida se mjerilo na osi
ordinata). Uobicajeno se na osi apscisa nalazi aritmeticko mjerilo za vrijeme, a na osi
ordinata je aritmeticko mjerilo za frekvencije razmatrane pojave

Docentica na Ekonomskom fakultetu Sveucilista u Zagrebu, Katedra za statistiku.


Studentica Ekonomskog fakulteta Sveucilista u Zagrebu.
Redoviti profesor na Ekonomskom fakultetu Sveu
cilista u Zagrebu, Katedra za matematiku.

Matematicko-fizicki list, LIX 2 (2008. 2009.)

69

Intervalni niz prikazuje se povrsinskim ili linijskim grafikonom tako da se grafikon


konstruira u prvom kvadrantu koordinatnog sustava. 1 Trenutacni vremenski niz prikazuje
se samo linijskim grafikonom.

Osnovni pokazatelji dinamike vremenskog niza


Iz grafickog prikaza mogu se uociti osnovne karakteristike stanja i razvoja promatrane
pojave u odredenom
vremenu. Te se promjene mogu kvantificirati izracunom osnovnih
pokazatelja dinamike vremenskog niza. Pokazateljima se mjere promjene koje je moguce
razlikovati prema vise kriterija:
prema razdoblju za koje se pokazatelj izracunava:
pojedinacne promjene,
prosjecne promjene,
prema mjernoj jedinici u kojoj su promjene izrazene:
apsolutne promjene,
relativne promjene,
prema razdoblju s kojim se usporeduje:
promjene u odnosu na razinu pojave u odabranom baznom vremenskom
razdoblju,
promjene razine pojave u uzastopnim vremenskim razdobljima.
Slijedi kratki pregled osnovnih pokazatelja dinamike po navedenim kriterijima.
Pojedinacne, apsolutno izrazene, promjene pojave u uzastopnim razdobljima (prve
diferencije)
Ako su Y : y1 , y2 , ..., yt , ..., yn , t = 1, 2, 3, ..., n , frekvencije vremenskog niza
s jednakim intervalima promatranja, pojedinacne promjene u apsolutnom iznosu u
uzastopnim razdobljima dane su izrazom:
yt = yt yt1 ,

t = 2, 3, 4, ..., n.

(2)

Navedene promjene tumace se kao povecanje (ako je vrijednost pojave definirane


formulom (2) pozitivan broj) ili smanjenje (ako je rijec o negativnom broju) promatrane
pojave u tekucem razdoblju u odnosu na prethodno razdoblje. Naravno, ako je yt = 0
za dano t , onda u trenutku t nije doslo do promjene vrijednosti pojave Y u odnosu
na trenutak t 1 . Pri tome su promjene izrazene u istim mjernim jedinicama kao i
promatrana pojava (u apsolutnim mjernim jedinicama).
Pojedinacne, apsolutno izrazene promjene pojave u odnosu na (fiksno) bazno
razdoblje
yt = yt yt1 , t = 1, 2, 3, 4, ..., n.
(3)
Ove se promjene tumace kao porast ili smanjenje promatrane pojave u tekucem
- su izrazene u istim
razdoblju u odnosu na neko fiksno, bazno razdoblje. Takoder
mjernim jedinicama kao i pojava za koju se izracunavaju.
1 Ako se prikazuje intervalni vremenski niz s nejednakim razdobljima promatranja, potrebno je korigirati
frekvencije niza. Korigiranje frekvencija ne provodi se kod trenutacnog niza jer frekvencije toga niza pokazuju
stanje pojave u odredenom
trenutku, a ne nad odredenim
razdobljem.

70

Matematicko-fizicki list, LIX 2 (2008. 2009.)

Prosjecna promjena pojave (prosjecna prva diferencija)


n

yt
yn y1
t=2
y =
ili y =
.
(4)
(n 1)
n1
Buduci da se prosjecna prva diferencija izracunava iz razlike posljednje i prve
frekvencije, a ostale frekvencije vremenskog niza nemaju utjecaj na velicinu prosjeka,
ovu je prosjecnu mjeru promjene pojave uputno koristiti ako pojava ne pokazuje znacajne
- prvog i posljednjeg razdoblja promatranja).
varijacije u vremenu (izmedu
Pojedinacne stope promjene pojave u uzastopnim razdobljima
yt yt1
100, t = 2, 3, 4, ..., n.
(5)
st =
yt1
Stopa promjene pojave pokazuje za koliko se (izrazeno relativno) promijenila razina
pojave u vremenu t u odnosu na vrijeme t 1 .
Pojedinacne stope promjene pojave u odnosu na (fiksno) bazno razdoblje
yt yb
st =
100, t = 1, 2, 3, ..., n.
(6)
yb
Stopa promjene pojave u odnosu na bazno razdoblje pokazuje za koliko se (izrazeno
relativno) promijenila razina pojave u vremenu t u odnosu na bazno razdoblje b .
Ako je potrebno izmjeriti prosjecnu (relativno izrazenu) promjenu pojave u odredenom
razdoblju, izracunava se prosjecna stopa promjene.
Prosjecna stopa promjene pojave
s = (G 1) 100,


G=


n1


yn
1 ,
y1

(7)

pri cemu je G geometrijska sredina.


- nije uputno koristiti ako vremenski niz pokazuje
Prosjecnu stopu promjene takoder
izrazito velike oscilacije s vremenom. Time prva i posljednja frekvencija niza nisu
pouzdan pokazatelj dinamike razvoja pojave unutar cijelog niza.
Prognoza pojave pomocu prosjecne stope promjene
Ako pojava nema izrazene varijacije u vremenu i ako se pretpostavi da ce se i u
narednom (kratkorocnom) razdoblju mijenjati prema utvrdenoj
prosjecnoj stopi, moze
se predvidjeti buduca razina pojave u vremenu n + koristeci se formulom
yn+ = yn G ,
(8)
pri cemu je:
broj razdoblja nakon posljednjeg za koje se provodi prognoza,
yn+ prognozirana vrijednost pojave u razdoblju n + .
Individualni indeksi
U osnovnoj numerickoj analizi vremenskog niza nezaobilazno je koristenje
individualnih indeksa. Ako se pomocu indeksa mjeri dinamika razvoja jedne pojave tada
su to individualni indeksi. Skupni indeksi mjere dinamiku skupine pojava. Individualni
indeksi mogu biti verizni (lancani) i bazni. Indeksi su pozitivni brojevi, a mogu biti
manji od sto, veci od sto ili jednaki sto.
Veriznim indeksom se mjeri promjena pojave u tekucem razdoblju u odnosu na
prethodno razdoblje (izrazena relativno). To su stope promjene pojave u uzastopnim
Matematicko-fizicki list, LIX 2 (2008. 2009.)

71

vremenskim razdobljima povecane za 100. Verizni indeks pokazuje koliko jedinica


pojave u vremenu t dolazi na sto jedinica pojave u vremenu t 1 .
yt
100, t = 2, 3, 4, ..., n,
(9)
Vt =
yt1
Vt = st + 100.
(10)
Verizni indeksi se mogu graficki prikazati grafikonom jednostavnih stupaca 2 ili linijskim
grafikonom.
Bazni indeksi (indeksi na stalnoj bazi) izrazavaju promjenu pojave u tekucem
razdoblju u odnosu na bazno razdoblje (izrazena relativno). To su stope promjene pojave
u odnosu na bazno razdoblje povecane za 100. Bazni indeks pokazuje koliko jedinica
pojave u vremenu t dolazi na sto jedinica pojave u baznom vremenu b :
yt
It =
100, t = 1, 2, 3, 4, ..., n,
(11)
yb

It = st + 100.
(12)
Buduci da se pri izracunu baznih indeksa frekvencije niza dijele istim brojem, indeksi
na stalnoj bazi proporcionalni su velicinama iz kojih su izracunani (frekvencijama
vremenskog niza). Kao i verizni indeksi, oni su jednaki sto, manji od sto ili veci od sto.

Primjena osnovne analize vremenskog niza u poslovanju fondova


cijena udjela
fonda u kn
25.10.2006.
154.86
25.11.2006.
154.47
25.12.2006.
157.11
25.01.2007.
164.75
25.02.6007.
177.48
25.03.2007.
184.44
25.04.2007.
200.71
25.05.2007.
206.12
25.06.2007.
204.09
25.07.2007.
210.74
25.08.2007.
206.67
25.09.2007.
211.12
25.10.2007.
220.63
26.11.2007.
198.92
24.12.2007.
208.43
25.01.2008.
190.11
25.02.2008.
185.13
25.03.2008.
169.72
24.04.2008.
164.85
Tablica 1. Cijene udjela PBZ Equity fonda. Izvor: www.pbzinvest.hr
datum

Na temelju podataka o cijenama udjela PBZ Equity fonda 3 provedena je empirijska


analiza vremenskog niza za razdoblje od 25. listopada 2006. do 24. travnja 2008. godine.
Duljina vremenskog niza je 19. Cijene su biljezene gotovo istog dana u mjesecu, stoga
2 Pri tome se na osi apscisa nanosi aritmeti
cko mjerilo za vrijeme koje upucuju na promjenu u tekucem prema
prethodnom razdoblju.

72

Matematicko-fizicki list, LIX 2 (2008. 2009.)

je vremenski niz cijena udjela fonda trenutacni vremenski niz (tablica 1).
Cijena udjela u fondu formira se ovisno o kretanjima cijena dionica u koje fond ulaze,
ovisi o trzistu efektive, te o kretanju cijena drzavnih obveznica. Najvazniji cilj Fonda je
sigurno plasirati prikupljena sredstva, te osigurati stalnu likvidnost uloga imatelja, uz s to
vecu profitabilnost ulaganja. Osnovna analiza vremenskog niza je od posebne vaznosti
u donosenju poslovnih odluka svih onih koji su direktno ili indirektno vezani uz rad
fonda.
240

cijena, kn

220
200
180
160
140
120
24.04.2008.

25.02.2008.

24.12.2007.

25.10.2007.

25.08.2007.

25.06.2007.

25.04.2007.

25.02.6007.

25.12.2006.

25.10.2006.

100

Grafikon 1. Cijene udjela PBZ equity fonda. Izvor: www.pbzinvest.hr

datum
t
1
25.10.2006.
25.11.2006.
25.12.2006.
25.01.2007.
25.02.6007.
25.03.2007.
25.04.2007.
25.05.2007.
25.06.2007.
25.07.2007.
25.08.2007.
25.09.2007.
25.10.2007.
25.11.2007.
24.12.2007.
25.01.2008.
25.02.2008.
25.03.2008.
24.04.2008.

cijena
udjela
fonda
u kn
yt
2
154.86
154.47
157.11
164.75
177.48
184.44
200.71
206.12
204.09
210.74
206.67
211.12
220.63
198.92
208.43
190.11
185.13
169.72
164.85
Tablica 2.

verizni
indeksi

stope
promjene
prve
pojedinacne
u odnosu
diferencije stope cijene
na bazu
cijene
udjela
b = 25.04.2007.
udjela
yt
st
st
5
6
7

22.84
0.39
0.25
23.04
2.64
1.71
21.72
7.64
4.86
17.92
12.73
7.73
11.57
6.96
3.92
8.11
16.27
8.82
0.00
5.41
2.69
2.70
2.03
0.98
1.68
6.65
3.26
5.00
4.07
1.93
2.97
4.45
2.15
5.19
9.51
4.50
9.92
21.71
9.84
0.89
9.51
4.78
3.85
18.32
8.79
5.28
4.98
2.62
7.76
15.41
8.32
15.44
4.87
2.87
17.87
PBZ Equity fonda kroz godinu dana

bazni indeksi
25.04.06 = 100

Vt
It
3
4

77.16
99.75
76.96
101.71
78.28
104.86
82.08
107.73
88.43
103.92
91.89
108.82
100.00
102.70
102.70
99.02
101.68
103.26
105.00
98.07
102.97
102.15
105.19
104.50
109.92
90.16
99.11
104.78
103.85
91.21
94.72
97.38
92.24
91.68
84.56
97.13
82.13
Kretanje cijene udjela

Iz navedenog grafickog prikaza (grafikon 1) moze se uociti da su cijene udjela do


listopada prosle godine pokazale dinamiku ujednacenog rasta, a od tada se, s iznimkom
u sijecnju 2008. godine kada su se povecale, kontinuirano smanjuju. Uocenu dinamiku
kretanja moguce je kvantitativno izraziti osnovnim pokazateljima dinamike kako je
izlozeno u prethodnom dijelu rada (tablica 2). Pri tome je nuzno ukazati na ogranicenja
primjene nekih od pokazatelja koristenima u analizi navedenog vremenskog niza cijena
udjela.
Matematicko-fizicki list, LIX 2 (2008. 2009.)

73

Diferencija uz datum 25.04.2007. prikazuje da je na taj dan cijena udjela u Equity


fondu bila veca za 16.25 kn u odnosu na mjesec prije tj. u odnosu na 25.03.2007. No,
diferencija uz datum 25.08.2007. prikazuje da je cijena udjela toga dana bila manja za
4.07 kn u odnosu na 25.07.2007. Diferencije ostalih dana u mjesecu interpretiraju se na
isti nacin.
Pojedinacna stopa promjene cijene udjela Equity fonda na dan 25.05.2007. godine
iznosi 2.69. To znaci da je prinos toga dana bio veci za 2.69% u odnosu na 25.04.2007.
Dok je 25.06.2007. godine stopa promjene cijene iznosi 0.98 . To znaci da je prinos
tog dana bio manji za 0.98% u odnosu na prethodno razdoblje.
Stopa promjene u odnosu na bazno razdoblje pokazuje za koliko se relativno
promijenila razina pojave u vremenu t u odnosu na bazno razdoblje. Stopa promjene
25.11.2006. u odnosu na bazno razdoblje 25.04.2007. iznosila je 23.04 , s to znaci
da je prinos cijena udjela u Fondu tog datuma u odnosu na bazni datum bio nizi za
23.04%, dok je 25.10.2007. stopa promjene iznosila 9.92 u odnosu na bazno razdoblje,
s to je za 9.92% visa cijena udjela u Fondu u odnosu na bazno razdoblje. Tumacenje
stopa promjene u uzastopnim razdobljima jednako je tumacenju veriznih indeksa (tabela
2, stupac 3), a stope promjene u odnosu na bazno razdoblje tumace se isto kao i bazni
indeksi (tabela 2, stupac 4), odnosno bazni indeks cijene udjela 25.10.2006. iznosi 77.16
s to pokazuje da je na 100 kuna cijene udjela u fondu 25.04.2007. dolazilo 77.16 kuna
cijene udjela 25.10.2006., odnosno da je cijena udjela u fondu bila manja za 22.84%.
U odnosu na bazni datum 25.04.2007. najveci pad cijene udjela u fondu bio
je 25.11.2006., dok je najveci porast cijene u odnosu na bazni datum zabiljezen
25.10.2007.
Ranije je vec konstatirano, a grafikon 3 to i potvrduje,
da su se cijene udjela do
listopada 2007. godine uglavnom povecavale u odnosu na prethodni mjesec. Stupci na
grafikonu veriznih indeksa koji su iznad osi 100 pokazuju povecanje. Smanjenje u tom
razdoblju zabiljezeno je u studenome 2006., lipnju i kolovozu 2007. godine. Nakon toga
je uslijedilo razdoblje kada su verizni indeksi bili manji od 100, stupci na grafikonu
su stoga ispod osi 100, s to znaci da su se cijene uzastopno smanjivale u odnosu na
prethodni mjesec osim u prosincu kad su se povecale za nesto manje od 5%.

indeksni bodovi

115

105

95

04.08./03.08.

03.08./02.08.

02.07./01.08.

01.08./12.07.

12.07./11.07.

11.07./10.07.

10.07./09.07.

09.07./08.07.

08.07./07.07.

07.07./06.07.

06.07./05.07.

05.07./04.07.

04.07./03.07.

03.07./02.07.

02.07./01.07.

01.07./12.06.

12.06./11.06.

11.06./10.06.

85

Grafikon 3. Verizni indeksi cijena udjela PBZ Equity fonda. Izvor: izracun autora

Buduci da je iz grafickog prikaza i iz stopa promjene u uzastopnim razdobljima,


odnosno iz veriznih indeksa ocito da se cijena udjela uzastopno povecavala do listopada
2007., a od tada se smanjivala, nije uputno racunanje prosjecne prve diferencije niti
prosjecne stope promjene cijene za cijelo razdoblje. Kao s to je ranije navedeno, ove
se prosjecne mjere temelje na prvoj i posljednjoj frekvenciji niza koje u ovom slucaju
nisu dobar reprezentant dinamike cijena u cjelokupnom razdoblju. Stoga je vremenski
niz podijeljen u dva dijela: od listopada 2006. do listopada 2007. i od studenoga 2007.
do travnja 2008. godine i izracunate su prosjecne mjere promjene za svaki dio niza
posebno.

74

Matematicko-fizicki list, LIX 2 (2008. 2009.)

Tako je za prvo razdoblje (do listopada 2007. godine) prosjecna prva diferencija:
220.63 154.86
yn y1
=
= 5.48,
yI =
n1
13 1
a za razdoblje od studenoga 2007. do travnja 2008. to je:
164.85 198.92
= 6.82
yII =
61
Prosjecna stopa promjene cijene udjela PBZ Equity fonda za oba razdoblja je:

 



y
220.63
n
12
1 100 = 2.99
sI = (n1)
1 100 =
y1
154.86


5 164.85
1 100 = 3.69
sII =
198.92
Cijena udjela u fondu u razdoblju od 25. listopada 2006. do 25. listopada 2007.
povecavala se prosjecno mjesecno za 5.48 kuna ili gotovo 3%, a nakon toga u razdoblju
od studenoga 2007. do travnja 2008. se smanjivala prosjecno mjesecno za 6.82 kn ili za
3.7%.
Na temelju uocene dinamike cijena udjela u fondu, nije moguce pretpostaviti da
c e se cijena mijenjati u narednom razdoblju po posljednjoj utvrdenoj
prosjecnoj stopi,
stoga nije moguce niti prognoziranje njene razine na temelju posljednje cijene iz travnja
i prosjecne stope promjene. Prognoziranje je nuzno provesti slozenijim statistickim
prognostickim metodama i modelima koji nisu tema ovoga rada.

Zakljucak
Strategija ulaganja i izbora instrumenata od kojih ce se graditi portfelj Fonda sadrzi
nesto veci rizik ulaganja, ali s tim nosi i vece prinose u fondu. Fond je namijenjen
ulagateljima koji z ele plasirati dio svoje imovine u domace i strane dionice, ulagati na
dulji rok te ostvariti vece stope prinosa. Cilj fonda je ostvarivanje relativno velikog
prinosa kroz dulje razdoblje, uz postojanje kratkorocnog rizika. Kontinuirano pracenje i
analiziranje promjena na trzistu kapitala postaje imperativ.
Analiza cijene udjela PBZ Equity fonda u devetnaest mjeseci dala je niz korisnih
informacija. Tendencija rasta cijene udjela u Fondu je sve do listopada 2007. godine bila
rastuca, ali se nakon toga uocava poremecaj i kontinuirano smanjenje cijena koje moze
biti jednim dijelom posljedica zasicenosti trzista fondovima c iji se broj s vremenom
povecava.
Graficka i numericka analiza vremenskog niza osnovnim pokazateljima dinamike ima
s iroke, ali uvijek treba imati na umu i ogranicenje, mogucnosti primjene. Numericki
postupci nisu zahtjevni, a daju korisne informacije o dinamici promatrane pojave, s to
c ini izvor informacija za odlucivanje.

Literatura
[1]
[2]
[3]
[4]
[5]
[6]

K. BLACK , Business statistics for contemporary decision making, Willey, New York (2006).
S. PIVAC , B. S EGO , Statistika udzbenik i zbirka zadataka, Alka script, Zagreb (2005).

C , Primijenjena statistika, Skolska


knjiga, Zagreb (2006).
I. S O SI

I. SO SIC , Statistika, Skolska knjiga, Zagreb (2006).

C , V. SERDAR , Uvod u statistiku, XII. izdanje, Skolska


knjiga, Zagreb (2002).
I. S O SI
www.pbzinvest.hr

Matematicko-fizicki list, LIX 2 (2008. 2009.)

75

Povijest matrica i determinanti


Marko Juraic, Zagreb
Poceci koristenja matrica i determinanti sezu u drugo stoljece pr. Kr., no neki tragovi
govore da su se koristile cak i u cetvrtom stoljecu pr. Kr. Ipak pravi razvoj i koristenje
je zapocelo tek u 17. st.
Matrice i determinante su nastale proucavanjem sustava linearnih jednadzbi. Babilonci
su proucavali probleme koji su se mogli rijesiti koristenjem takvih jednadzbi, a neki od
njih su sacuvani na glinenim plocicama. Jedna takva plocica potjece iz oko 300. godina
pr. Kr. i sadrzi sljedeci problem:
Imamo dva polja c ija je ukupna povrsina 1800 m2 . Oba polja su za proizvodnju zita,
2
1
2
2
s tim da jedno daje kg/m , a drugo daje kg/m . Ukoliko je ukupan rod 1100 kg,
3
2
kolika je povrsina svakog polja?
- 200. i 100. godine pr. Kr., vise priblizili matricama nego
Kinezi su se, izmedu
- je tekst pod nazivom Nine Chapters on
Babilonci. Za vrijeme dinastije Han izraden
the Mathematical Art (Devet poglavlja matematickog umijeca) koji je prvi ikad naden
primjer koristenja matricnih metoda. Prvi problem je vrlo slican Babilonskom, a glasi
ovako:
Imamo tri vrste zita, od c ega tri mjere prvog, dvije drugog i jedna treceg zajedno
c ine 39 jedinica. Dvije mjere prvog, tri drugog i jedan treceg c ine 34 jedinica, a jedna
mjera prvoga, dvije drugoga i tri trecega c ine 26 jedinica. Koliko jedinica zita sadrzi
jedna mjera za svaki tip zita?
Ovaj problem su rjesavali tako da su postavili jednadzbe, cije koeficijente su poslozili
u stupce pridruzene nepoznanicama:
1
2
3
2
3
2
3
1
1
26 34 39
Danasnja metoda rjesavanja kaze nam da se koeficijenti pisu u redove, ali to sada nije
bitno. U uputama za rjesavanje ovog zadatka navedeno je: pomnozi srednji stupac s 3
i oduzmi desni stupac koliko god puta mozes. Zatim se isto napravi i s prvim stupcem
(pomnozi s 3 i oduzmi se treci stupac). To daje,
0
0
3
4
5
2
8
1
1
39 24 39
Zatim se prvi stupac pomnozi s 5 i od njega se oduzme srednji stupac onoliko puta
koliko je to moguce napraviti. Dobivamo,
0
0 3
0
5 2
36
1 1
99 24 39

76

Matematicko-fizicki list, LIX 2 (2008. 2009.)

Odavde se moze izracunati koliko je jedinica za trecu vrstu z ita, zatim za drugu, a onda
i za prvu (uvrstavanjem). Ova metoda se naziva Gaussova metoda eliminacije i nije bila
poznata sve do pocetka 19. st.
Godine 1545. Jerome Cardan, u djelu Ars Magna, iznosi pravilo za rjesavanje
sustava dviju linearnih jednadzbi koje se naziva majka svih pravila. Ovo pravilo je
istovjetno Cramerovom pravilu za rjesavanje (2, 2) sistema, no Cardan nije napravio
zavrsni korak. Nije dosao do definicije determinante, ali je bio na pravom putu.
Mnogi rezultati elementarne matricne teorije su se pojavili mnogo prije nego su
matrice postale predmet matematicke analize, tako da su mnoge zapocete teorije bile
zapravo put ka otkrivanju mnogo kompliciranijih izvoda i rezultata za koje se u to
vrijeme kada su stvorene nije ni pomisljalo.
Ideja o determinanti se gotovo istovremeno pojavila u Japanu i u Europi, ali prvi
ju je objavio Seki Takakazu (poznat kao Seki Kowa). On je 1683. g. napisao djelo
Method of solving the dissimulated problems (Metoda rjesavanja skrivenih problema)
koje sadrzi metode rjesavanja pomocu matrica ispisanih u tablicu i to na isti nacin kako
su to radili u staroj Kini. Bez da je spomenuo i jednu rijec koja bi mogla predstavljati
determinantu, dao je opcenite nacine izracunavanja determinanti s to se baziralo na
primjerima (determinante matrica reda (2, 2), (3, 3), (4, 4) i (5, 5)).
Iste godine se determinanta pojavila i u Europi. Godine 1683. Gottfried Wilhelm
Leibniz je pisao lHospitalu i objasnio mu da sustav jednadzbi
10 + 11x + 12y = 0
20 + 21x + 22y = 0
30 + 31x + 32y = 0
ima rjesenje jer je 102132+112230+122031 = 102231+112032+122130
s to je zapravo uvjet da je determinanta koeficijenata matrice jednaka 0. Leibniz je u
ovome primjeru koristio brojeve kao koeficijente, a oni zapravo predstavljaju znakove
koje mi danas pisemo kao a11 , a12 itd., odnosno prvi od dva indeksa predstavlja u kojoj
se jednadzbi nalazi koeficijent, a drugi oznacava nepoznanicu.
Leibniz je bio uvjeren da je kljuc uspjeha u matematici pravilno oznacavanje
koeficijenata, pa je on eksperimentirao upravo s nasumicnim odabirom pisanja. U
njegovim neobjavljenim radovima nadeno
je preko 50 razlicitih nacina pisanja sistema
koeficijenata koje je pisao kroz pedesetak godina pocevsi od 1678. Samo na dva mjesta
(1700. i 1710. godine) dobiven je rezultat sistema i tada je koristen zapis koji je napisao
u svom pismu lHospitalu.
Leibniz je za danasnju determinantu koristio naziv rezultanta, i dokazao je
razlicite rezultate u ovisnosti o determinanti s to poznajemo kao Cramerovo pravilo.
Isto tako je znao da se determinanta moze izracunati razvojem po bilo kojem stupcu, s to
se danas zove Laplaceov razvoj.
Tridesetih godina 18. st. Maclaurin je napisao Treatise of algebra (objavljeno
1748. g.) u kojemu je dokazao Cramerovo pravilo za (2, 2) i (3, 3) sisteme i pokazao
kako bi se mogao rijesiti (4, 4) sustav. Cramer je odredio opcenito pravilo za (n, n)
sisteme u djelu Introduction to the analysis of algebraic curves (Uvod u analizu
algebarskih krivulja) (1750. g.), te je u tom djelu izrazio z elju za pronalazenjem
jednadzbe ravnine koja je zadana odredenim
brojem tocaka. Pravilo je napisao na kraju
djela, ali ono nije bilo dokazano.
Proucavanje determinanata se sada pocelo razvijati.
Godine 1764. Bezout je otkrio metode za racunanja determinanti, a isto je ucinio
i Vandermonde 1771. g. Laplace je 1772. g. tvrdio da su Cramerove i Bezoutove
Matematicko-fizicki list, LIX 2 (2008. 2009.)

77

metode neprakticne te je napisao diskusije rjesenja linearnih jednadzbi bez racunanja


- ce je da je determinante nazivao
sustava, vec je samo racunao determinante. Iznenaduju
rezultante, jednako kao i Leibniz, a zapravo nije bio upoznat s njegovim radom.
Lagrange je 1773. g. proucavao identitete determinanti (3, 3) matrica, ali njegovi svi
- svog rada i rada Laplacea
komentari su bili nagadanja
i on nije mogao naci vezu izmedu
i Vandermondea. No ipak njegovo djelo sadrzi izraz za volumen tetraedra zadanog s
tockama O(0, 0, 0), M(x, y, z), M  (x , y , z ), M  (x , y , z ):
1
[z(x y y x ) + z (yx xy ) + z (xy yx )].
6
Izraz determinanta je prvi upotrijebio Gauss u djelu Disquisitiones arithmeticae
(1801. g.) dok je opisivao kvadraticne forme, a to ime je izabrao zato s to determinanta
odreduje
(eng. determines) svojstva kvadratnickog oblika. No njegov koncept nije
jednak onome koji mi poznajemo. U istom djelu Gauss je ispisao koeficijente svoje
kvadraticne forme u oblik pravokutnika. Opisao je mnozenje matrica (koje je smatrao
kao kompoziciju pa stoga nije dosegao koncept matricne algebre) i izracunavanje inverza
matrice.
Gaussovu eliminaciju, koja se prvi put pojavila u djelu Nine Chapters on
Mathematical Art (Devet poglavlja matematickog umijeca) 200. g. pr. Kr., Gauss je
koristio pri proucavanju putanje asteroida Pallas. Koristenjem tih podataka uzetih izmedu
1803. i 1809. g., sastavio je sistem od s est linearnih jednadzbi sa s est nepoznanica.
Nasao je sistematsku metodu za rjesavanje takvih jednadzbi s to se upravo i naziva
Gaussova metoda eliminacije.
Augustin Louis Cauchy je 1812. g. prvi koristio determinantu u modernom smislu.
Cauchyjev rad je najkompletniji rad o determinantama ikad uraden.
On je dokazao
prethodne rezultate i dao nove na temelju minora i adjunkta. Godine 1812. dokazao je
teorem o mnozenju determinanti po prvi put.
Godine 1826. Cauchy je napisao dijagonalizaciju matrice. Isto tako je uveo ideju o
simetricnim matricama i pokazao da one imaju realne svojstvene vrijednosti. Poslije je
dokazao i da se svaka realna simetricna kvadratna matrica moze dijagonalizirati.
Zanimljivo je da niti Cauchy niti Sturm nisu nasli opcenite relacije za ideje koje
su iznijeli, vec su ih koristili i vidjeli samo u kontekstu onoga cime su se oni bavili.
Jacobi (1830. g.), a zatim i Kronecker i Weierstrass (1850. i 1860. g.), bavili su
- ne opcenito vec u smislu linearne transformacije.
se matricnim rezultatima ali takoder
Jacobi je objavio tri rada o determinantama 1841. g. koji su bitni zato s to je po prvi
puta definicija determinante bila napisana preko algoritma, pa su rezultati bili tocni bilo
da se radilo samo o brojevima ili funkcijama. Nakon ovih radova ideja determinanti
postala je poznatom cijelom svijetu.
- 1841. g. objavio prvi engleski clanak o teoriji determinanti. Na
Cayley je takoder
svom papiru je koristio dvije vertikalne linije na lijevoj i desnoj strani redova cime je
oznacavao determinante, a takvo oznacavanje je ostalo i do danas.
Ime matrica prvi je koristio Sylvester 1850. g. On je definirao matricu kao pravilnu
skupinu vrijednosti i gledao ju kao nesto s to ima ulogu odrediti determinantu vrijednosti
koju sadrzava. Nakon povratka iz Amerike u Englesku 1851. g., Sylvester je postao
- zanimala. Kada je
odvjetnik i upoznao Cayleya, odvjetnika kojeg je matematika takoder
Cayley vidio vaznost matrica, 1853. g. objavio je c lanak u kojem se po prvi put pojavio
inverz matrice.
Cayley je objavio Memoir on the theory of matrices koja po prvi puta sadrzi apstraktnu
definiciju matrice 1858. g. Pokazao je da su sustavi koeficijenata za kvadraticne forme i

78

Matematicko-fizicki list, LIX 2 (2008. 2009.)

linearne transformacije zapravo specijalni slucajevi njegovog opcenitog koncepta. Cayley


- je dokazao da, u
je pokazao definicije zbrajanja, mnozenja i inverza matrica. Takoder
slucaju (2, 2) matrice, matrica ponistava svoj svojstveni polinom. Hamilton je nakon
toga dokazao specijalni slucaj za (4, 4) matrice i stoga se to otkrice danas naziva
Cayley-Hamiltonov teorem.
Godine 1870. Jordan je u djelu Treatise on substitutions and algebraic equations
objavio nesto s to se danas naziva Jordanova forma matrice. To je dijagonalna
blok-matrica na c ijoj se dijagonali nalaze tzv. Jordanovi blokovi (kvadratna matrica
specijalnog oblika).
Frobenius je 1878. g. napisao vrlo vazan rad On linear substitutions and bilinear
forms (O linearnim supstitucijama i bilinearnim formama) s tim da nije bio upoznat
s Cayleyvim radom. Nakon toga je, procitavsi Cayleya, dokazao Cayley-Hamiltonov
teorem za opceniti slucaj (1896), i izveo definiciju za rang matrice.
Pozitivnost kvadratne matrice je 1884. g. dao Sylvester. Dosao je do zakljucka da
je kvadratna forma pozitivna ako i samo ako su glavne minore odgovarajuce matrice
pozitivne.
Glavni ljudi koji su poceli ozbiljno promovirati teoriju matrica i determinanti bili
su Weierstrass (objavio je rad On determinant theory (O teoriji determinante)) koji je
- objavljeni
objavljen 1903. nakon njegove smrti) i Kronecker (njegovi radovi su takoder
nakon njegove smrti). Teoriju determinanti su matematicari odmah prihvatili, dok je za
teoriju matrica trebalo nesto vise vremena. Rad koji je pridonio potpunom shvacanju
teorije matrica bio je Introduction to higher algebra (Uvod u visu algebru) od Bochera
1907. g. Turnball i Aitken su napisali vrlo utjecajne radove 1930. godine, a nakon rada
Mirskya 1955. (An introduction to linear algebra (Uvod u linearnu algebru)), linearna
algebra je zauzela mjesto jedne od najvaznijih grana matematike.


Kalendar natjecanja u matematici


za ucenike osnovnih i srednjih skola 2009. g.

Skolska
natjecanja

Zupanijska
natjecanja
Klokan bez granica"
Drzavno natjecanje
Mediteransko matematicko natjecanje
Regionalna natjecanja
Medunarodna
matematicka olimpijada
Srednjoeuropska matematicka olimpijada

Matematicko-fizicki list, LIX 2 (2008. 2009.)

29. sijecnja
23. veljace
19. ozujka
od 29. ozujka do 1. travnja
travanj ili svibanj
8. svibnja
od 10. do 22. srpnja
od 24. do 29. rujna

79

S razredbenog ispita u Japanu


Roko Pesic, Zagreb
U jednom americkom c asopisu objavljen je sljedeci zadatak koji se pojavio na
razredbenom ispitu na jednom sveucilistu u Tokiju, ali bez rjesenja. Zadatak mi se
- a nadam se ce i vama biti interesantan.
svidao,
Zadatak. Zadana je pravilna c etverostrana
piramida i kugla sa sredistem koje lezi u bazi
piramide. Ona dira sve njezine bridove, a brid
osnovice je duljine a . Nadite:
a) visinu v piramide;
b) obujam V dijela koji je zajednicki kugli i
piramidi.
Rjesenje. a) Iz uvjeta zadatka se vidi da
srediste S kugle mora biti u sredistu baze
piramide (baza je kvadrat sa stranicom duljine
a), a to znaci da je polumjer kugle jednak
polovici duljine stranice a ,
a
R= .
(0)
2

Slika 1.

a 2
Promotrimo pravokutni trokut ASV : |AS| + |SV| = |AV| , gdje je |AS| =
,
2
|SV| = v (visina piramide), |AV| = b (bocni brid piramide), pa je
a2
(1)
v2 = b 2 .
2
- iz uvjeta zadatka slijedi da je visina |SE| trokuta ASV jednaka polumjeru R
Takoder
kugle, |SE| = R. Oznacimo li |AE| = x , tada je |EV| = b x .
S obzirom da su trokuti ASV , ASE i ESV pravokutni, vrijede ove jednakosti:
2

R2 = x(b x),
(2)

 a 2 2
2
a
.
(3)
= bx = x =
2
2b
Uvrstivsi jednakosti (0) i (3) u (2) dobivamo a = b , i to je piramida kojoj su svi
bridovi jednake duljine (polovica oktaedra). Iz jednakosti (1) tada dobivamo visinu
piramide

a 2
.
v=
2
b) Zajednickom dijelu obujma pripada gornja polukugla umanjena za cetiri kuglina
odsjecka koji ne pripadaju piramidi po jedan nad svakom pobockom piramide. Ti
odsjecci su odredeni
cetirima ravninama u kojima leze pobocke piramide jer njih,
(odsjecke) te ravnine odsijecaju od gornje polukugle.
Obujam kuglinog odsjecka je
1
Vo = h(3r2 + h2 ),
(4)
6

80

Matematicko-fizicki list, LIX 2 (2008. 2009.)

gdje je r polumjer baze, a h visina kuglinog odsjecka. Baza kuglinog odsjecka je krug
- kruznicom polumjera r . Ta kruznica dira bocne bridove piramide, a kako je
omeden
ova jednakostranicni trokut stranice duljine a , ta kruznica je upisana u jednakostranicni

trokut pa za nju vrijedi jednakost a = 2r tg , gdje je sredisnji kut jednakostranicnog


2
trokuta. Kako je = 360 : 3 tj. = 120 , uvrstavanjem te vrijednosti u prethodnu
jednakost dobivamo da je a = 2r tg 60 , tj. a = 2r 3 odnosno
a
(5)
r= .
2 3

Slika 2.

Slika 3.

Slika 4.

Za odsjecak kugle polumjera R, visine h i polumjera baze r (vidi sliku 3) vrijedi


jednakost:
(6)
r2 = h(2R h),
s to lako slijedi iz Pitagorinog poucka. Uvrstivsi (0) i (5) u (6), nakon sredivanja
a2
= 0. Njezina rjesenja su:
dobivamo kvadratnu jednadzbu u varijabli h , h2 ah +


12
1
1
h=a
+
0.9a,
(7a)
6
2
1
1

h=a
0.1a.
(7b)
2
6
Rjesenje (7a) nije ispravno, jer iz (0) bi slijedilo h > R, s to ne moze biti. Rjesenje
(7b) zadovoljava uvjet h < R, pa je to rjesenje ispravno.
Uvrstivsi izraze (5) i (7b) u formulu (4) za obujam kuglinog odsjecka konacno
dobivamo:


a3 1 7 6

.
(8)
Vo =
6
2
36
Zajednicki obujam c emo dobiti ako od obujma polukugle oduzmemo ukupni obujam
cetiriju kuglinih odsjecaka:


V1
2 3
a3 1 7 6
V=
4Vo = R 4

.
2
3
6
2
36
Nakon sredivanja
dobivamo zajednicki obujam kugle i piramide


a3 7 6 1
V=

.
2
27
2

Matematicko-fizicki list, LIX 2 (2008. 2009.)

81

Jedno svojstvo logaritamske funkcije i njegova primjena


Petar Vranjkovic, Zadar
U ovom ce c lanku biti govora o jednom manje poznatom svojstvu logaritamske
funkcije i njegvoj ucinkovitoj primjeni na rjesavanje nekih logaritamskih nejednadzbi.
To su one u kojima su baza logaritamske funkcije i njezin argument funkcije iste
varijable. Znamo da se takve nejednadzbe rjesavaju metodom razlikovanja slucajeva.
Pokazat c emo da se primjenom tog svojstva spomenute nejednadzbe rjesavaju znatno
lakse i jednostavnije.
Podimo
od definicije logaritamske funkcije. Funkcija f : R+ R odredena
formulom
f (x) = logb x
je definirana za b > 0 i b = 1 .
Primjer 1. Rijesimo nejednadzbu
log1+x (2x 6) 0

x R.

Rjesenje. Ovu nejednadzbu cemo rijesiti metodom razlikovanja slucajeva.


1

0<1+x <1
2x 6 > 0
1 + x = 1
2x 6 1
x

Prema tome, rjesenje zadane nejednadzbe je x

7
.
2

1+x >1
2x 6 > 0
1 + x = 1
2x 6 1
7
x
2

No, uocimo sljedece nejednadzbe (ostale su u 1 i 2 ):


1+x<1
2x 6 > 0

1+x>1
2x 6 > 0

1+x1<0
2x 6 1 0

1+x1>0
2x 6 1 0

(1 + x 1)(2x 6 1) 0
(1 + x 1)(2x 6 1) 0.
Dakle, u oba slucaja su umnosci istog predznaka. Na tom tragu se pojavila ideja o
novom svojstvu logaritamske funkcije.
Poucak. Neka je a > 0 , a = 1 i b > 0 .
(1)
Tada vrijedi
loga b 0 (a 1)(b 1) 0 ili loga b  0 (a 1)(b 1) 0 (2)
Dokaz. Provedimo dokaz za slucaj loga b 0.
Ako a i b zadovoljavaju (1) onda imamo:
loga b 0 ((0 < a < 1 b 1) (a > 1 0 < b 1))

82

Matematicko-fizicki list, LIX 2 (2008. 2009.)

((a 1 < 0 b 1 0) (a 1 > 0 b 1 0))


(a 1)(b 1) 0.
Analogno se dokazuje ekvivalentnost nejedankosti
loga b 0 (a 1)(b 1) 0.
Vratimo se sada primjeru 1 i rijesimo ga pomocu ovog poucka. Prema (1) imamo
1 + x > 0, 2x 6 > 0, 1 + x = 1.
Rjesenje ovog sustava je x > 3 .
(3)
Prema (2) imamo
(1 + x 1)(2x 6 1) 0,
odnosno x(2x 7) 0 tj. 2x2 7x 0 .
7
Rjesenje ove kvadratne nejednadzbe je x 0 ili x , a zbog (3), konacno rjesenje
2
7
je x .
2
U slucaju kada je ili baza ili argument logaritamske funkcije konstanta, onda je
postupak brzi i laksi. Evo jednog primjera.
Primjer 2. Rijesimo nejednadzbu
log 1 (4x + 1) < 0,
3

x R.

1
Rjesenje. Prema (1) je 4x + 1 > 0 , pa je x > , a prema (2) imamo
4


1
1 (4x + 1 1) < 0,
3
odnosno x > 0 . Konacno rjesenje je x > 0 .
Zadatak 1. Ako je a > 0 , b > 0 , c > 0 i a = 1 , onda vrijedi
loga b loga c (a 1)(b c) 0
loga b loga c (a 1)(b c) 0

(4)
(4*)

Dokaz. Dovoljno je dokazati prvu tvrdnju.


loga b loga c loga b loga c 0


b
b
loga 0 (a 1)
1 0 (a 1)(b c) 0.
c
c
Pri c emu je koristeno (2).
Evo jednog primjera.
Primjer 3. Nadimo
rjesenje nejednadzbe
log2x (4x 1) 2,

x R.

Rjesenje. Domenu rjesenja cemo dobiti rjesavanjem sustava


2x > 0, 2x = 1, 4x 1 > 0.
Dobijemo


1
1
x
, + , x = .
4
2
Matematicko-fizicki list, LIX 2 (2008. 2009.)

(5)

83

Zadanu nejednadzbu cemo transformirati tako da mozemo primijeniti nejednakost


(4). Dobijemo
(2x 1)(4x 1 (2x)2 ) 0,
odnosno, nakon sredivanja
(2x 1)3 0 .
1
Odavde imamo 2x 1 0 , tj. x . Uzimajuci u obzir domenu rjesenja, konacno
2


1 1
rjesenje je x
,
.
4 2
Primjer 4. Rijesimo nejednadzbu
log|x|


x2 9 1.

(6)

Rjesenje. Odredimo domenu rjesenja:


x = 1,

x = 0,

x2 9 > 0.

x = 1,

Ovi uvjeti su zadovoljeni za


x  , 3  3, + .
Nadalje, (6) mozemo zapisati
log|x|
Prema (4) imamo

(7)


x2 9 log|x| |x|.


(|x| 1)( x2 9 |x|) 0.

(8)

Dalje, mozemo rjesavati na standardni nacin, ali ovdje cemo koristiti drugu metodu.
U domeni rjesenja sigurno vrijedi

x2 9 + |x| 0.
(9)
Pomnozimo li (8) s (9), dobivamo
(|x| 1)(x2 9 x2 ) 0

tj.

|x| 1 0.

Rjesenje ove nejednadzbe je 1 x 1 , a iz (7) imamo konacno rjesenje x , tj.


nijedan realan broj x ne zadovoljava danu nejednadzbu.
Zadatak 2. Neka je
a > 0,

b > 0,

c > 0,

d > 0,

a = 1,

c = 1.

(10)

Tada je
1
2

84

loga b logc d 0 (a 1)(b 1)(c 1)(d 1) 0


loga b logc d 0 (a 1)(b 1)(c 1)(d 1) 0
loga b
0 (a 1)(b 1)(c 1)(d 1) 0, d = 1
logc d
loga b
0 (a 1)(b 1)(c 1)(d 1) 0, d = 1
logc d

(11)

ili

(11)
ili

(12)
(12)

Matematicko-fizicki list, LIX 2 (2008. 2009.)

Primjer 5. Rijesimo nejednadzbu


logx (1 x) log2x (2x) 0,

x R.

(13)

Rjesenje. Prema (10) i (13) dobivamo


x > 0, x = 1, 1 x > 0, 2 x > 0, 2 x = 1, 2x > 0.
Ovi uvjeti su zadovoljeni za
x  0, 1 .
(14)
Primjenom (11) nejednadzba (13) je ekvivalentna sljedecoj
(x 1)(1 x 1)(2 x 1)(2x 1) 0,
(15)
tj.
x(1 x)2 (2x 1) 0.
(16)
2
Kako je (1 x) > 0 za sve x  0, 1 , (16) je ekvivalentno s nejednadzbom
x(2x 1) 0,

1
, + , pa iz (14) dobivamo konacno rjesenje
cije je rjesenje x  , 0]
2


1
x
,1 .
2

Zadaci za vjezbu
1. log2 (4x) > 1 ,

Rj. x > 0 .

2. log3x (4x 1) log3x (9x2 1) ,

fi
Rj. x

1 4
,
3 9

fl
.

3. logx 2 log2 (4x) > 1 ,


4. log2x+4 x2 < 2 ,

Rj. x > 1 .
fi
fl
4
3
Rj. x ,
.
2
3

|x| 2
< 0,
Rj. x > 2 .
|x| + 3
x
x
Rj. x  0, 1 .
6. log 1 + logx > 0 ,
x 2
4
fl

fi

1
1+x
,1 .
7. log 3+x
> 0,
Rj. x
1x
2x
2
8. logcos x sin x logsin x cos x 0 ,
Rj. nema rjesenja.
5. logx+1

9. logsin x 4 logsin2 x 2 > 0 ,


10.

log2+x (1 + x)
0,
log1+x x

Rj. k < x < + k ,

x =

+ 2k , k Z .
2

Rj. x > 1 .

Matematicko-fizicki list, LIX 2 (2008. 2009.)

85

LHC: zasto mu se divimo?


Tome Anticic i Vuko Brigljevic 1 , Zagreb

Brzina je vazna
Fizika cestica istrazuje najmanje dijelove prirode i stanje svemira duboko u proslosti,
samo nekoliko trenutaka nakon Velikog praska. Jedan od glavnih nacina za dobivanje
laboratorijskih uvjeta za stvaranje novih, tezih c estica i za ispitivanje ponasanja prirode na
visokim temperaturama je sudaranjem visokoenergetskih cestica. Einsteinova jednadzba
koja povezuje masu i energiju objasnjava nam kako je to moguce:
1
m c2 ,
E= 
v2
1 2
c
gdje su m, v i E masa, brzina i energija cestice, a c brzina svjetlosti. S malo
razmisljanja moze se doci do zakljucka da priblizavanjem brzine v brzini svjetlosti c,
energija cestice raste prema beskonacnosti. Dakle, s vecim brzinama stvaraju se vece
energije, a time i uvjeti za proizvodnju cestica s vecom masom i proucavanje fizike na
visim temperaturama.

Slika 1. Shematski prikaz LHC-a


1 Dr. Tome Anti
cic i dr. Vuko Brigljevic su visi znanstveni suradnici u Zavodu za eksperimentalnu fiziku Instituta
- Boskovic u Zagrebu.
Ruder

86

Matematicko-fizicki list, LIX 2 (2008. 2009.)

Puno komaraca u Velikom hadronskom sudarivacu


- danasnjice za takvo ispitivanje materije je Veliki hadronski sudarivac
Najmocnije orude
(engl. Large Hadron Collider, skraceno LHC), smjesten oko 100 m ispod povrsine
na s vicarsko-francuskoj granici. Izgradnja LHC-a dovrsena je upravo ove godine, a
u proljece 2009. ocekuju se sudari na najvisim energijama za koje je konstruiran.
Pridjev veliki je posve na mjestu: LHC ubrzava protone (i druge cestice) u kruznom
tunelu s opsegom od c ak 27 km, i kao takav je daleko najveci sudarivac na svijetu.
Veliki je i zato s to, dijelom i zbog svojih impresivnih dimenzija, stvara najenergetskije
laboratorijske sudare. Naime, kroz LHC mogu kruziti dva snopa protona kroz odvojene
cijevi u suprotnim smjerovima s energijama od cak 7 TeV ( 1 TeV = 1012 eV, a
1 eV = 1.6 1019 J, energija koju jedan elektron dobije kada je na potencijalu od
jednog volta). Izrazeno brzinama, svaki proton c e se kretati brzinom jednakom cak
0.999999991 brzine svjetlosti. Dakle, svake sekunde c e protoni u snopu c ak 11 000 puta
obici cijeli prsten LHC-a. Izrazeno svakodnevnim primjerom, ta energija je otprilike
jednaka kinetickoj energiji leteceg komarca. Na prvi pogled to se i ne doima jako
impresivno, ali treba imati na umu da je na LHC-u ta energija koncentrirana u prostor
bilijun puta manji od dimenzije tog istog komarca.

Slika 2. Slikoviti prikaz stvaranja novih c estica (razno voce) iz energije

Jos impresivnija je energija po snopu. U svakom od njih je oko 3000 skupina cestica,
koji se sastoje od cak 100 milijardi protona. To znaci da c e svaki snop (od dva) na
LHC-u imati ukupnu energiju od oko 3.5 108 J, ekvivalentno 400 tonskom vlaku koji
putuje brzinom od 150 km/ sat!

Puno energije u Velikom hadronskom sudarivacu


LHC je dizajniran kao sudarivac, s to znaci da se ubrzane cestice sudaraju jedna u
drugu, za razliku od inzinjerski puno jednostavnijeg i jeftinijeg rjesenja usmjeravanja
snopa na nepokretnu metu. Razlog tome je razlicit iznos kolicine energije u sustavu
centra mase, to jest korisne kolicine energije za stvaranje novih cestica. U frontalnom
Matematicko-fizicki list, LIX 2 (2008. 2009.)

87

sudaru c estica iz suprotnih pravaca energija se zbraja, tako da se dobiva 14 TeV-a


na LHC-u za protonske snopove (snopovi teskih jezgara, poput olova, imat ce i vecu
energiju), dok se kod nepokretne mete korisna energija
mijenja kao korijen od energije
udarajuce cestice, to jest za velike energije E 2 mproton Esnop , pa se umjesto
14 TeV-a dobiva tricavih 0.114 TeV-a.

Slika 3. Slicnosti u operaciji televizora i sudarivaca

Mocni magneti
Kretanje c estice kroz tunel kontrolira se pomocu dvije vrste magneta: dipola i
kvadrupola. Dipolni magneti imaju, kao s to im ime i kaze, dva magnetska pola, i
zaduzeni su za drzanje cestica na kruznoj putanji. Naime, magnetsko polje ne povecava
brzinu cestice, vec je skrece, tako da c estica koja ide pod pravim kutom u odnosu na
je veca energija koja se z eli postici, to dipolni
polje, putuje kruznom putanjom. Sto
magneti (a ima ih 1232 na LHC-u) moraju proizvoditi jace magnetsko polje da bi mogli
cestice zadrzati unutar cijevi sudarivaca. Za energije koje se z ele postici na LHC-u,
magnetsko dipolno polje iznosi cak 8.3 T (tesla), s to zahtijeva supravodljive magnete i
hladenje
pomocu helija u superfluidnom stanju (koji ima iznimno dobru vodljivost za
toplinu, ali je strahovito tezak za manipuliranje) na temperaturi od samo 1.9 K (kelvina),
s to je hladnije c ak i od dalekog Svemira (2.7 K). Ukupno c e u svakom trenutku magneti
- s cak 120 tona helija!
biti hladeni
Da je LHC manji, magnetsko polje bi trebalo biti jos vece za uspostavu iste energije
snopa, s to je i razlog potrebe tunela s to veceg promjera:
p 0.3 B R,
gdje je p impuls cestice u GeV/ c2 (1 GeV = 109 eV), B je polje u teslama, a R
promjer u metrima.
Kvadrupolni magneti, s cetiri naizmjenicna pola, su zaduzeni za fokusiranje snopa.
Zbog sprecavanja nepozeljnih sudara, snop se krece u cijevima u kojima je uspostavljen
vakuum deset puta bolji od vakuuma na Mjesecu (Mjesec nema atmosferu).

88

Matematicko-fizicki list, LIX 2 (2008. 2009.)

Slika 4. Pogled na dipolne magnete unutar LHC-tunela

Gubici energije u Velikom hadronskom sudarivacu


Uz magnete, LHC ima i elektromagnetske komore koje sluze najprije za ubrzavanje
c estica i onda za drzanje istih na konstatnoj energiji kompenzirajuci gubitke energije.
Operacija sudarivaca je ustvari na puno nacina slicna televizoru (onom starinskog tipa s
katodnim cijevima), kao s to je objasnjeno na slici 3. Samo nekoliko c estica je prikladno
za ubrzavanje u sudarivacima. One moraju biti i stabilne i nabijene, s to izbor svodi
na protone, jezgre tezih elemenata ili elektrone. Spomenuti gubitak energije eliminira
elektrone kao cestice prikladne za LHC. Rijec je o takozvanom sinkrotonskom zracenju,
kada nabijena cestica uslijed kretanja u kruznoj putanji gubi energiju emitirajuci fotone:
1  Esnop 4
Esinkroton
.
R m
Ovaj gubitak energije je ocito iznimno osjetljiv na masu cestice, i za teze cestice kao
s to su protoni je puno manji (po jednom krugu rotacije) nego za oko 2000 puta lakse
elektrone. Ako protoni energije 7 TeV na LHC-u gube po krugu 7 103 eV, izracunajte
koliko bi elektron izgubio sinkrotonskim zracenjem! Odgovor ce vam nedvosmisleno
objasniti zasto se elektroni ne rabe na LHC-u. Izbor dakle ostaje samo na protonima (i
tezim ionima) kao jedinim prikladnim cesticama.

Slika 5. Prosireni prikaz impresivnog CMS (od engl. Compact Muon Solenoid) detektora.
Sicusna mrlja ispred CMS-a je ustvari prikaz c ovjeka u pravoj skali. CMS je napravljen,
- ostalog, od c ak 10 000 tona zeljeza, s to je vise nego u Eiffelovom tornju. Detektor je
izmedu
15 m visok, 21 m dugacak i ima masu od 12 500 tona.
Matematicko-fizicki list, LIX 2 (2008. 2009.)

89

Protoni se klasificiraju kao hadroni, vrsta c estica karakterizirana kvarkovskom strukturom


(vidi prvi okvir). Tako je konacno objasnjen i pridjev hadronski u LHC-u. Mana
protonskih sudara je upravo postojanje njihove strukture, zbog c ega je samo dio energije
koristan za stvaranje novih c estica. U principu bi se na LHC-u mogli sudarati protoni
i antiprotoni, s to bi imalo jos vecu korisnu energiju. Medutim,
protone je puno lakse
stvoriti i time je stvoreni broj sudara u sekundi (velicina proporcionalna tzv. luminozitetu)
iznimno veci.
Okvir 1. Standardni model elementarnih cestica
Elementarne cestice (fermioni)

Postoje tri generacije elementarnih cestica. Cestice


druge i trece generacije su u svemu slicne
- su nestabilne i nakon kratkog
onima iz prve generacije osim po tome s to su teze. Takoder
vremena se raspadaju u cestice prve generacije. Materijalni svijet oko nas se sastoji iskljucivo
od cestica prve generacije, dok se one ostalih generacija stvaraju u prirodnim ili umjetnim
visokoenergetskim procesima i brzo raspadaju.
leptoni
elektricni
naboj

kvarkovi

elektron
gradi atom
zajedno s
jezgrom

elektronski neutrino
jako slabo medudjeluje
i moze proci kroz
cijelu zemlju a da
se ne zaustavi,
milijarde njih prolaze
kroz nas svake
sekunde; ima jako
malu masu
mionski neutrino
slican elektronskom
neutrinu, stvara se
zajedno s mionima
u nekim raspadima
tau-neutrino
otkriven tek 2000.
godine kao
zadnji od svih
fermiona

mion
tezi rodak
elektrona

tau
jos tezi,
jako
nestabilan

2
+
3

1
3

generacija

gornji (Up)
sastavni dio
protona i
neutrona
zajedno sa
down kvarkom

donji (Down)
sastavni dio
protona i
neutrona

carobni
(Charm)
tezi rodak
up kvarka

strani (Strange)
tezi rodak
down
kvarka

vrsni (Top)
najteza od
svih poznatih
cestica, tezak
kao atom
zlata

dubinski (Bottom)
- down
jos tezi rodak
kvarka, vazan za
proucavanje asimetrije
materija-antimaterija

Kvarkovi se nikada ne nalaze sami. Uvijek su vezani u hadrone. Jedini stabilni hadroni su
proton i neutron (i neutron zapravo samo ukoliko se nalazi u atomskoj jezgri, izvan nje se
raspada nakon nekoliko minuta). Proton se sastoji od dva up-kvarka i jednog down-kvarka, a
neutron od dva down-kvarka i jednog up-kvarka.
Svi leptoni i kvarkovi su fermioni: njihov spin, to jest intrinsicna kutna kolicina gibanja, je
visekratnik Planckove konstante podijeljene s 2. Fermioni zadovoljavaju Paulijev princip: dva
identicna fermiona ne mogu se nalaziti istodobno u istom stanju. Taj zahtjev je neophodan
uvjet za bogatstvo struktura koje vidimo u svijetu, od atomske jezgre do kompleksnih DNK
molekula z ivota.
Za svaki fermion, kvark ili lepton, postoji i odgovarajuca anticestica, jednake mase, jednakog
trajanja z ivota i spina, ali sa suprotnim elektricnim nabojem. Tijekom velikog praska nastajale
su jednake kolicine cestica i anticestica. Medutim,
anticestice su potpuno nestale iz naseg
svemira, a razlog za to jos nije utvrden.

90

Matematicko-fizicki list, LIX 2 (2008. 2009.)

Divovski detektori za otkrivanje mikrosvijeta


Na vise mjesta na LHC-tunelu protoni iz suprotnih pravaca se fokusiraju unutar
desetak mikrona i sudaraju. Oko tih lokacija izgradeni
su masivni detektorski sustavi
koji detaljno analiziraju stvorene c estice i iz toga pokusavaju razumjeti fiziku koja
se pri sudaru dogodila. Detektori su koncipirani slojevito: cestice koje proizlaze iz
tocke sudara prolaze kroz razne slojeve. Svaki od njih mjeri specificnu karakteristiku
prolazecih cestica: kolicinu gibanja, elektricni naboj, energiju ili brzinu. Razne vrste
cestica ce ostaviti razlicite potpise u detektoru. Dok ce neke kao elektroni ili fotoni biti
zaustavljene vec u prvim slojevima detektora, druge ce, poput miona, proci kroz gotovo
cijeli detektor i jedini ostaviti trag u golemim mionskim komorama u vanjskom sloju
detektora (vidi sliku 6). Svaki od tih divovskih eksperimenta je rezultat kolaboracije
vise tisuca znanstvenika i tehnicara.

Slika 6. Presjek CMS detektora s raznim slojevima detektora.


Prikazan je i odziv detektora na razne vrste c estica.

Carobne
rijeci: Higgs, dodatne dimenzije, supersimetrija, tamna tvar,. . .
LHC je stvoren jer nase znanje Svemira nije konacno. Tzv. Standardni model cestica
i sila ujedinjuje nase trenutno znanje fizike c estica, vidi okvir 1 i 2. Standardni model
je testiran na brojnim eksperimentima i bio je iznimno uspjesan u predvidanju
novih
cestica i njihovih svojstava. Medutim,
puno nerijesenih problema ostaje, i LHC je
upravo dizajniran da postigne energiju gdje se neki odgovori moraju pojaviti. Prvo
veliko otvoreno pitanje je pitanje porijekla mase, koje je od kljucne vaznosti za
samu konzistentnost standardnog modela. U verziji u kojoj postoje samo vec poznate
cestice, masa svih cestica bi treba biti jednaka nuli, s to ocito ne odgovara cinjenicama.
Standardni model se moze nadograditi uvodenjem
Higgsovog polja, s cijom interakcijom
c estice dobivaju masu (jeste li se kada upitali: zasto tijela imaju masu?). To polje
podrazumijeva i najmanje jednu jos neotkrivenu c esticu, tzv. Higgsov bozon. Teorija
- sve njegove karakteristike osim jedne, njegove mase. Medutim,
predvida
iz ranijih
mjerenja i teorijskih razloga znamo da mu masa mora biti veca od otprilike 120 i manja
od 200 vodikovih atoma. Ako postoji i ako mu je masa u tom podrucju, Higgsov bozon
c e nastati u proton proton sudarima na LHC-u i eksperimenti ce ga otkriti. Otkrice
Higgosvog bozona je jedan od prvenstvenih ciljeva dva najveca LHC-eksperimenta,
CMS-a i ATLAS-a.
Matematicko-fizicki list, LIX 2 (2008. 2009.)

91

Medutim,
otkrice Higgsovog bozona ne moze biti kraj price. Naime, standardni
model c ak i s Higgsovim poljem ne daje kompletnu sliku svih fundamentalnih sila, te
sadrzi i puno proizvoljnih parametara, i ostavlja niz otvorenih pitanja: zasto se priroda
ponavlja po tri puta (vidi okvir 1)? Krije li se dublji princip reda iza tri generacija
elementarnih cestica? Zasto su mase elementarnih cestica toliko razlicite jedna od druge
(top kvark je vise od 300 tisuca puta tezi od elektrona)? Kako ukljuciti gravitaciju u
sliku? Jedna moguca nadogradnja tog modela je tzv. teorija supersimetrije, koja postulira
- bozona i fermiona, i koja predvida
- da svaka postojeca cestica
temeljnu simetriju izmedu
ima i svog superpartnera. Ako je ta teorija tocna, LHC ce najvjerojatnije naci najlaksu
supersimetricnu c esticu, koja je ujedno i glavni kandidat za objasnjenje tamne tvari, tj.
23% gustoce energije svemira koja ne potjece od nama poznatih c estica (odgovornih za
tek 4%). Postoje jos egzoticnije stvari koje ce mozda biti otkrivene, kao s to su dodatne
dimenzije prostora, ili odstupanje zakona gravitacije od 1/r2 ovisnosti na jako malim
skalama, s to bi moglo dovesti do jako zanimljivog fenomena produkcije mini crnih rupa
na LHC-u.
Okvir 2. Standardni Model elementarnih cestica
Elementarne sile (bozoni)
- elementarnih cestica. Bez njih ne bi bilo
Sile su odgovorne za medudjelovanje
izmedu
strukturirane tvari, reakcija, raspada. Danas poznajemo cetiri osnovne sile. Sile se prenose
putem izmjene cestica nosioca sila.
elektromagnetska
osjetljive cestice kvarkovi i nabijeni
leptoni
vezane pojave
drzi elektrone oko
atomskih jezgri,
spaja atome u
molekule, i
zasluzna je
za svojstva
plinova,
tekucina, i
krutih tvari

slaba
svi kvarkovi
i leptoni
uzrok
radioaktivnosti

nositelj sile

W + , W , i Z 8 gluona

foton

jaka
kvarkovi
veze kvarkove u
protonu, neutronu i
drugim hadronima,
drzi atomsku jezgru
na okupu i pri tome
je jaca od ogromne
elektricne odbojne
sile izmedu
protona

gravitacija
sve cestice
s masom
prva poznata
sila, ali na
mikroskali
jos uvijek
najlosije
shvacena;
ne uklapa se
u teorijski
opis standardnog
modela
graviton? (postulat)

Elektromagnetska i slaba sila su prepoznate kao razlicite manifestacije jednog istog principa
i sjedinjene u elektroslabu silu. U okviru standardnog modela opisane su elektroslaba i jaka
sila. Ukljucenje gravitacije u tu sliku ostaje jedan od velikih izazova fizike na pocetku 21.
stoljeca.
Nosioci sila su bozoni, s to znaci da je njihov spin (vidi okvir 1) cjelobrojan (0,1,2,. . . )
visekratnik Planckove konstante.

Potraga za novom fizikom: trazenje igle u puno plastova sijena


Kako ce fizicari naci Higgsov bozon, supersimetricne c estice ili neki drugi znak nove
fizike u LHC sudarima? Kao prvo treba napomenuti da je rijec o vrlo rijetkim procesima
koje ce fizicari trebati izdvojiti iz ogromne kolicine podataka. U srcu CMS i ATLAS
detektora ce se svakih 25 nanosekundi krizati snopovi protona i u svakom krizanju c e
doci do nekoliko proton-proton sudara koji ce ostaviti tragove u detektoru, dakle 40

92

Matematicko-fizicki list, LIX 2 (2008. 2009.)

milijuna puta svake sekunde. Ocekuje se da ce stopa proizvodnje Higgsovog bozona,


ovisno o teorijskom modelu, biti reda velicine jedan na dan (po svakom eksperimentu),
Trazenje igle u plastu sijena u usporedbi
s to znaci da se trazi jedan od 10 13 dogadaja!
izgleda djecjom igrom: ako je volumen tipicne iglice 5 mm 3 , a volumen plasta sijena 50
m 3 , trazenje nove fizike u LHC sudarima odgovara trazenju jedne igle u tisucu plastova
sijena! Pri tome je prvi izazov uopce sacuvati podatke zanimljivih sudara. Svaki sudar
ce u detektoru generirati otprilike 1 MB informacije, a 40 milijuna njih 40 TB svake
sekunde! Takvu kolicinu podataka nije moguce pohraniti i od njih c e se samo 100 do 200
- su zauvijek izgubljeni i
najzanimljivijih sacuvati za daljnju analizu. Odbaceni dogadaji
- koji sadrze tragove
stoga fizicari moraju biti sasvim sigurni da smo sacuvali dogadaje
Higgsovog bozona ili drugih oznaka jos nepoznate fizike. Za izbor najzanimljivijih
- je odgovoran veliki racunalni grozd sastavljen od nekoliko tisuca racunala koji
dogadaja
u nekoliko milisekundi mora analizirati svaki sudar trazeci oznake zanimljivih dogadaja.
Tu fizicarima zakoni fizike ipak dolaze u pomoc i bitno im olaksavaju posao. U vecini
proton-proton sudara c e se dva protona tek okrznuti i samo lagano rasprsiti, te ce
produkti tih sudara uglavnom letjeti jako blizu osi samog snopa. S druge strane, ako se
u sudaru stvori tezi objekt poput Higgsovog bozona ili supersimetricne cestice, od kojih
znamo iz rezultata ranijih eksperimenata da ukoliko postoje moraju biti tezi od otprilike
100 atoma vodika, raspasti c e se u puno lakse vidljive ili nevidljive cestice. Iz vec
- mase i energije slijedi da ce se masa teskog objekta u raspadu
spomenute relacije izmedu
pretvoriti u veliku kolicinu gibanja laganih produkata raspada. A zato jer je u principu
- se ocekuju cesto cestice
svaki pravac leta produkata raspada moguc, iz takvih dogadaja
s velikim kutom rasprsenja prema pravcu snopa, ili velikom transverzalnom kolicinom
gibanja u odnosu na pravac snopa. Taj jednostavan argument je kljuc izbora zanimljivih
dogadaja:
traze se sudari s cesticama velike transverzalne energije ili kolicine gibanja.

Slika 7. Unutrasnja struktura protona: proton je sastavljen od dva donja i jednog gornjeg
kvarka. Ali unutar protona mozemo naci i gluone koji vezu kvarkove, a i dodatne parove
kvark-antikvark koji se mogu na trenutak stvoriti i ponovo nestati.

Kad su podaci sudara jednom snimljeni, postaju objekt duge i cesto komplicirane
analize fizicara koji ce krenuti u potragu za oznakama nove fizike. Trazene nove cestice,
npr. Higgsov bozon, se obicno nece mjeriti direktno u detektoru, jer su jako nestabilne.
Raspasti c e se vrlo brzo nakon stvaranja i produkti njihovog raspada, obicno vec poznate
cestice, su oni koji ce ostaviti vidljiv trag u raznim slojevima detektora. Ako je npr.
masa Higgsovog bozona jednaka masi 150 do 200 atoma vodika, cesto ce se raspasti
- elektrona,vidi okvir 1)
u 2 elektrona i 2 miona. Elektroni i mioni (mion je tezi rodak
ostavljaju karakteristican trag u detektoru (vidi sliku 6). Iz mjerenja njihovih impulsa
moze se jednostavno odrediti masa Higgsovog bozona. Pri tome se koristi samo zakon
- mase, energije i impulsa
ocuvanja energije i impulsa i relativisticka relacija izmedu
2
2 2
2 4
jedne cestice: E = p c + m c (ovo je jednadzba s pocetka c lanka u drugacijem
obliku). Mala komplikacija je takozvana pozadina: bit ce puno drugih dogadaja
s 2
elektrona i 2 miona u konacnom stanju, zapravo ce ih obicno biti za nekoliko redova
velicine vise od dogadaja
s Higgsovim bozonom (signal). Tu se otvara prostor za
mastu i kreativnost fizicara koji c e kroz lukavi odabir zahtjeva uspjeti odbaciti skoro
- i sacuvati one s Higgs-signalom. Katkad ce se za tu proceduru
sve pozadinske dogadaje
Matematicko-fizicki list, LIX 2 (2008. 2009.)

93

koristiti slozenim alatima poput neuronskih mreza. Nova fizika se moze otkriti iako se
stvaraju nove c estice koje nevidljivo pobjegnu iz detektora bez da ostave ikakav trag,
ali mogu sa sobom nositi znacajnu kolicinu energije i kolicinu gibanja. To bi npr. bio
slucaj za cesticu tamne tvari, koja samo vrlo slabo djeluje s normalnom materijom.
Koristeci se ponovo zakonima ocuvanja kolicine gibanja i energije i c injenicom da
detektor potpuno okruzuje tocku sudara moze se izmjeriti sveukupni impuls i energiju i
primijetiti da nesto nedostaje. Iz ostatka detektora moze se odrediti energija, a mozda i
masa i druge osobine bjegunca. Mozemo dakle otkriti i nesto s to uopce nismo vidjeli,
zakoni ocuvanja su stvarno vazni u fizici!
Konacno je vrijedno spomenuti jos jednu karakteristiku sudara proton-proton koja
dodatno otezava mjerenja na LHC-u: okruzenje tih sudara je jako prljavo! Da bismo
razumjeli o cemu je rijec moramo se sjetiti da proton nije jednostavan i elementaran
objekt. Sastoji se od tri kvarka, ali zapravo je i slozeniji od toga (vidi okvir 1). U
protonu mozemo naci kvarkove, ali i gluone, pa cak i antikvarkove. Kad se u sudaru
proton-proton stvara Higgsov bozon, ili par top-kvarkova (za njih znamo sigurno da
- dvije elementarne cestice (kvark,
postoje!), to je produkt elementarne reakcije izmedu
antikvark ili gluon) iz dva protona. Ali tu su i ostali sastavni dijelovi protona, ostali
kvarkovi i gluoni, koji nisu sudjelovali u glavnom procesu, u kojem igraju na jedan
nacin samo ulogu gledatelja. Nakon tog glavnog procesa, i kvarkovi-gledatelji ce letjeti
dalje i zakomplicirati konacno stanje s dodatnim cesticama. Nalazenje tragova iz raspada
- svim tim cesticama (njih cesto vise od 100) predstavlja dodatni
neke nove cestice medu
izazov u fascinantnom intelektualnom pothvatu potrage za novom fizikom
- proucavati i stanje
Osim sudara proton-proton, sudarima iona olova LHC ce takoder
materije koja je postojala nekoliko mikrosekundi nakon velikog praska, tzv. kvarkovskogluonske plazme. U tom stanju kvarkovi i gluoni su se slobodno kretali u iznimno vrucoj
i gustoj juhi, za razliku od danas kada ih se samo moze naci u cesticama kao s to su
protoni ili neutroni. U sudarima olovo-olovo stvorena temperatura bit ce vise od 100 000
puta veca nego ona u sredistu Sunca. Eksperiment optimiziran za ova istrazivanja je
ALICE. Pomocu c etvrtog eksperimenta na LHC-u koji se zove LHC-b, pokusat ce se
odgonetnuti tajne asimetrije tvari u svemiru, to jest naci odgovor na pitanje zasto nema
jednake kolicine materije i antimaterije u svemiru.
Eksperimenti na CERN-u ukljucuju i po vise tisuca znanstvenika iz cijelog svijeta
(vecinom fizicara) koji c e imati pune ruke posla sljedecih 10 do 20 godina da pokusaju
iz dobivenih podataka odgonetnuti s to vise tajni Svemira. I desetak hrvatskih fizicara
je ukljuceno u taj pothvat. Mozete ih kontaktirati ako mozda i vi z elite biti dio ovog
najveceg znanstvenog projekta u povijesti. . .


Nadopuna. U c lanku J. Siftara


Abelova nagrada 2008. g. John G. Thompson i
Jacques Tits u br. 1/ 233 ispustene su slike dobitnika nagrade, pa ih sada donosimo.

John G. Thompson

94

Jacques Tits
Matematicko-fizicki list, LIX 2 (2008. 2009.)

Geometrija i problemi kretanja


Ljiljana Sudar, Leskovac
Neki fizicki problemi mogu se rijesiti na vise nacina, primjenom razlicitih podrucja
matematike. Problemi kretanja se, npr. najcesce rjesavaju algebarski. Taj nacin rjesavanja
zahtijeva za dano kretanje poznavanje eksplicitne ovisnosti brzine i prijedenog
puta od
vremena.
Medutim,
i bez poznavanja te eksplicitne ovisnosti moguce je i efikasnije i elegantnije
rijesiti mnoge probleme kretanja pomocu nestandardne, veoma mocne metode, koja se
bazira na primjeni grafova i geometrije.
Ukoliko se pri kretanju intenzitet brzine za isti vremenski interval uvijek mijenja
za istu vrijednost (tj. ubrzanje je konstantno), samo na osnovu grafova i elementarnog
poznavanja geometrije mogu se postaviti relacije iz kojih se dobiva rjesenje problema.

Slika 1. Grafovi ovisnosti brzine o vremenu

Ako se intenzitet brzine za svaki mali vremenski interval t uvijek povecava za istu
vrijednost v (tj. ubrzanje je konstantno), graf brzine je pravac koji zaklapa ostar kut
v2 v1
v
=
s pozitivnim dijelom t -osi (sl. 1a). S grafa se vidi: tg =
= a,
t
t2 t1
gdje je a intenzitet ubrzanja. Ako se kroz tocku K koja je srediste duzine DC
povuce pravac paralelan t -osi, pravokutni trokuti DD1 K i KC1 C su identicni, pa
imaju jednake povrsine. Zato ce osjencana povrsina trapeza ABCD (u oznaci P) biti
jednaka povrsini pravokutnika ABC1 D1 (PABC1 D1 ) osnovice t = t2 t1 i visine v tj.
put S za vrijeme t , bit
P = PABC1D1 = v t , pa kako je v t elementarni prijedeni
ce P = S tj. povrsina ispod odgovarajuceg dijela grafa brzine predstavlja elementarni
- put.
prijedeni
Ako se intenzitet brzine za svaki mali vremenski interval t uvijek smanjuje za
istu vrijednost v (tj. ubrzanje je konstantno), graf brzine je pravac koji zatvara
tupi kut s pozitivnim dijelom t -osi (sl. 1b). S grafa je ocigledno = i
v
tg = tg( ) = tg =
= a , gdje je a intenzitet ubrzanja, pa slijedi
t
tg = a , gdje je kut koji pravac zatvara s negativnim dijelom t -osi. Analogno
se, kao u prethodnom slucaju, moze pokazati da je elementarni prijedeni
put tijela za
vrijeme t (S) osjencana povrsina trapeza ABCD (P) s osnovicama AD i BC i
|AD| + |BC|
t .
visinom t = t2 t1 , tj. S = P =
2
Matematicko-fizicki list, LIX 2 (2008. 2009.)

95

Ako se intenzitet brzine tijekom vremena ne mijenja, pravac je paralelan s t -osi


(sl. 1c). Prijedeni
put tijela za vrijeme t (u oznaci S) je, ocigledno, osjencana
povrsina pravokutnika ABCD (P) osnovice t i visine v, tj. S = P = t v.
Zbroj elementarnih prijedenih
putova je ukupan prijedeni
put tijela i predstavlja
povrsinu ispod odgovarajuceg dijela grafa brzine.
Metoda rjesavanja problema kretanja pomocu grafa, primjenom geometrije zahtijeva,
osim dobrog razumijevanja grafickog prikaza problema kretanja, i to imajuci u vidu:
- put tijela i
1) da je povrsina ispod grafa brzine prijedeni
2) da je nagib pravca, kojom se prikazuje ovisnost brzine tijela o vremenu, definiran
ubrzanjem tijela koje se krece.
Slijede primjeri koji ilustriraju ovu metodu.
- za dvostruko dulje vrijeme nego drugu
Primjer 1. Prvu polovinu puta tijelo prijede
polovinu. Srednja brzina duz cijelog puta je 6 m/ s. Kolika je srednja brzina na prvoj, a
kolika na drugoj polovini puta? [2]
Rjesenje.
Dan je graf brzine tijela u ovisnosti o vremes
tijelo prelazi
nu. Drugu polovinu puta s2 =
2
brzinom v2 za vrijeme t . Prvu polovinu puta
s
tijelo prelazi brzinom v1 za vrijeme 2t
s1 =
2
(tj. dva puta dulje vrijeme nego drugu polovinu
puta), pa je ukupno vrijeme kretanja tijela 3t i
citav put
(1)
s = s1 + s2 .
Put s1 jednak je povrsini pravokutnika OABC ,
osnovice |OA| = 2t i visine |OC| = v1 , (u oznaci POABC ), tj.
s
(2)
s1 = = POABC = v1 2t.
2
Put s2 jednak je povrsini pravokutnika ADEF , osnovice |AD| = t i visine |AF| = v2 ,
(u oznaci PADEF ), tj.
s
s2 = = PADEF = v2 t.
(3)
2

Citav
put s tijelo bi preslo i da se sve vrijeme (3t) kretalo srednjom brzinom vsr = 6
- put s u tom slucaju bio jednak povrsini
(u m/ s), pa je s grafa ocigledno da bi prijedeni
pravokutnika ODGH , osnovice |OD| = 3t i visine OH = vsr , (u oznaci PODGH ), tj.
s = PODGH = vsr 3t = 18t.
(4)
Iz relacija (2) i (3) dobivamo
(5)
2v1 = v2 .
Zamjena (2), (3) i (4) u (1), uz uvjet (5) daje v1 = 4.5 i v2 = 9 (u m/ s).
Primjer 2. Ivan i Luka hodaju s istog mjesta u suprotnim smjerovima oko nogometnog
igralista koje je dugacko 110 m, a s iroko 70 m. Luka hoda brzinom 5 m/ s, a Ivan
brzinom 4 m/ s. Koliko metara ce svaki od njih prevaliti kad se prvi put sretnu? Nakon
koliko vremena ce to biti? [3]

96

Matematicko-fizicki list, LIX 2 (2008. 2009.)

Rjesenje.

Na sl. a) su dani grafovi brzina Luke (L) i Ivana (I) u ovisnosti o vremenu.
Do trenutka (t) kad se prvi put sretnu Luka je presao put (sL ) jednak povrsini
pravokutnika OACD, osnovice |OA| = t i visine |OD| = 5 (u m/ s), (u oznaci POACD ),
pa je
sL = POACD = 5t.
(1)
Ivan je do prvog susreta presao put jednak povrsini pravokutnika OABE , osnovice
|OA| = t i visine |OE| = 4 (u m/ s), (u oznaci POABE ), pa je
sI = POABE = 4t.
(2)
U trenutku susreta (t) je zbroj prijedenih
putova Luke i Ivana jednak opsegu igralista
(O) duzine a = 110 m i s irine b = 70 m (vidi sl. b), tj.
sL + sI = O.
(3)
Kako je opseg igralista O = 2a + 2b = 360 (u m), kao i relacija (1), (2) i (3) dobiva
se: 9t = 360 , tj. t = 40 (u s) s to znaci da je vrijeme prvog susreta Luke i Ivana 40 s
nakon istovremenog polaska s istog mjesta (na sl. b) to mjesto je vrh M pravokutnika;
Luka iz njega krece oko nogometnog igralista u smjeru kretanja kazaljki na satu, a Ivan
u suprotnom smjeru, na s to ukazuju strelice). Na osnovu relacije (1), Luka je do prvog
susreta presao put sL = 5t = 200 (u m), a na osnovu relacije (2) Ivan je presao put
sI = 4t = 160 (u m).
Primjer 3. Padobranac skace iz aviona te nakon 3 s otvara padobran. Nakon otvaranja
padobrana on naglo (trenutno) usporava te nastavlja padati brzinom od 5.4 m/ s. Deset
sekundi nakon njegovog skakanja iz aviona skace drugi padobranac. Nakon koliko
vremena on mora otvoriti padobran da bi zajedno stigli do tla? Pretpostavite da je otpor
zraka prije otvaranja padobrana zanemariv. [3]
Rjesenje.

Grafovi I i II daju ovisnost brzina od vremena oba padobranca. Nulti trenutak


vremena je onaj kada skace prvi padobranac. Prve tri sekunde on slobodno pada tj.
krece se s ubrzanjem a = g , a zatim konstantnom brzinom 5.4 (u m/ s). Nakon deset
sekundi iskace i drugi. On slobodno pada tj. krece se s ubrzanjem a = g sve do trenutka
Matematicko-fizicki list, LIX 2 (2008. 2009.)

97

t kada sustize prvog padobranca, otvara padobran pa njegova brzina trenutno postaje 5.4
(u m/ s) (vidi graf ). Od trenutka t oba padobranca se krecu brzinom 5.4 (u m/ s) sve
dok istovremeno ne stignu do tla.
Zbog istog ubrzanja padobranaca dok slobodno padaju (a1 = a2 = g), nagib duzina
OB i CE prema pozitivnom dijelu t -osi je isti, pa iz pravokutnih trokuta OAB i
CDE imamo
v2
v1
=
= g.
(1)
3
t 10
- put prvog padobranca, s1 , je u trenutku sustizanja (t) jednak zbroju povrsina
Prijedeni
trokuta OAB, osnovice |OA| = 3 (u s) i visine |AB| = v1 , i pravokutnika ADFG,
osnovice |AD| = t 3 i visine |DF| = 5.4 (u m/ s), tj.
3v1
s1 =
+ 5.4(t 3).
(2)
2
- put drugog padobranca, s2 , je u trenutku sustizanja (t) jednak povrsini trokuta
Prijedeni
CDE , osnovice |CD| = t 10 i visine |DE| = v2 , tj.
v2 (t 10)
s2 =
.
(3)
2
- putovi padobranaca su jednaki u trenutku sustizanja (t) tj.
Prijedeni
(4)
s1 = s2
pa je prema relacijama (2) i (3):
v2 (t 10)
3v1
+ 5.4(t 3) =
.
(5)
2
2
Zamjenom relacije (1) u (5) dolazi se do kvadratne jednadzbe:

30.24
10.8
= 0.
+ 91 +
t2 t 20 +
g
g
Ako se uzme g = 9.81 (u m/ s2 ), kvadratna jednadzbe glasi:
t2 21.1t + 94.08 = 0.
Njena resenja su: t1 = 14.7 i t2 = 6.4 (u s). Drugi padobranac otvara padobran nakon
t 10 sekundi (vidi sliku), pa je ocigledno da fizicki smisao ima samo drugo rjesenje
za koje je t 10 = 4.7 (u s).
Primjer 4. Kuriri iz mjesta A i B krecu jedan drugome u susret, pri c emu se svaki
krece jednoliko, ali razlicitom brzinom u odnosu na onog drugog. Posto su se sreli, da
bi stigli u nasuprotna mjesta, jednome je potrebno jos 16, a drugome 9 sati. Koliko
- citav put izmedu
- A i B?
je potrebno vremena jednom, a koliko drugom, da prijedu
(Zadatak Lewisa Carrolla, engleskog matematicara i pisca knjige za djecu Alisa u
zemlji c udesa.) [4]
Rjesenje.

98

Matematicko-fizicki list, LIX 2 (2008. 2009.)

Dan je graf brzina oba kurira u ovisnosti o vremenu. Ne smanjujuci opcenitost


rjesavanja mozemo pretpostaviti da je brzina vA kurira iz mjesta A veca od brzine vB
kurira iz mjesta B tj. vA > vB . Nulti trenutak vremena je onaj kada se kuriri pocinju
kretati jedan drugome ususret, a t je trenutak njihovog susreta. U trenutku susreta (t)
zbroj prijedenih
putova kurira jednak je udaljenosti d mjesta A i B (vidi sl. b)) tj.
sA + sB = d.
(1)
Kurir iz A je do susreta presao put sA , jednak povrsini pravokutnika OKMN ,
osnovice |OK| = t i visine |ON| = vA , (u oznaci POKMN ), pa je
sA = POKMN = vA t.
(2)
Kurir iz B je do susreta presao put sB , jednak povrsini pravokutnika OKLP , osnovice
|OK| = t i visine |OP| = vB , (u oznaci POKLP ), pa je
sB = POKLP = vB t.
(3)
Zamjenom relacija (2) i (3) u (1) dobiva se
vA t + vB t = d.
(4)
Kako je vA > vB kurir iz A je manje udaljen od mjesta B nego kurir iz B od mjesta A,
pa c e po uvjetu zadatka njemu trebati 9 sati da stigne u B, dok c e kuriru iz B trebati
16 sati da stigne u A. S grafa je ocigledno da kurir iz A stize u B u trenutku t + 9 ,
presavsi put jednak udaljenosti d mjesta A od mjesta B, s to na grafu odgovara povrsini
pravokutnika OQRN, osnovice |OQ| = t + 9 i visine |ON| = vA , (u oznaci POQRN ), tj.
(5)
d = POQRN = vA (t + 9).
Kurir iz B stize u A u trenutku t + 16 presavsi put jednak udaljenosti d mjesta B od
mjesta A, s to na grafu odgovara povrsini pravokutnika OSTP , osnovice |OS| = t + 16
i visine |OP| = vB , (u oznaci POSTP ) tj.
(6)
d = POSTP = vB (t + 16).
d
d
Iz relacija (5) i (6) mozemo izraziti brzine: vA =
i vB =
.
t+9
t + 16
Njihovom zamjenom u relaciju (4) dolazi se do kvadratne jednadzbe:
d
d
t+
t = d tj. t2 = 144 odnosno t = 12 (u h).
t+9
t + 16
Kurir iz A krecuci se brzinom vA preci ce c itav put od A do B za vrijeme:
t + 9 = 21 (u h)
a kurir iz B krecuci se brzinom vB preci ce c itav put od B do A za vrijeme:
t + 16 = 28 (u h).
Primjer 5. Tijelo je baceno brzinom 20 m/ s vertikalno uvis. Na visini 15 m nalazi
se horizontalna ploca od koje se tijelo savrseno elasticno odbije. Nakon koliko vremena
(od trenutka izbacaja) se tijelo vrati natrag? (g = 9.81 ms2 ). [5]
Rjesenje: Graf predstavlja ovisnost brzine
tijela od vremena. Nulti trenutak vremena je
trenutak izbacaja tijela vertikalno uvis. Brzina
tijela se tokom kretanja navise smanjuje i u
trenutku t1 , kada tijelo ima brzinu v1 , udara u
horizontalnu plocu. Prema uvjetu zadatka udar
tijela o plocu, koja je na udaljenosti d = 15 m
od mjesta izbacaja tijela, savrseno je elastican
Matematicko-fizicki list, LIX 2 (2008. 2009.)

99

pa brzina v1 mijenja samo smjer dok njezin intenzitet ostaje isti. Tijelo se nakon udara
put tijela
o plocu krece nanize i u trenutku t2 vraca na mjesto izbacaja. Prijedeni
vertikalno uvis do udara u plocu (s1 ) jednak je udaljenosti ploce od mjesta izbacaja (d)
tj. s1 = d = 15 (u m).
Na grafu tom putu odgovara povrsina pravokutnog trapeza OABC , s osnovicama
|OC| = 20 (u m/ s) i |AB| = v1 i visinom |OA| = t1 , (u oznaci POABC ), pa je
20 + v1
t1 = 15 (u m).
(1)
s1 = POABC =
2
Na grafu prijedenog puta tijela vertikalno dolje do mjesta izbacaja (s2 ) odgovara
povrsina pravokutnog trapeza ADEB, s osnovicama |AB| = v1 i |DE| = v2 i visinom
|AD| = t2 t1 = t , (u oznaci PADEB ), pa je
v2 + v1
s2 = PADEB =
t = 15 (u m).
(2)
2
2
Ubrzanje tijela prema gore i prema dolje je gravitacijsko (g = 9.81 m/ s ) i odreduje
nagib duzine BC prema negativnom dijelu t -osi i nagib duzine BE prema t -osi (vidi
sliku) pa je: FBC = KBE i vrijedi
v2 v1
20 v1
=
.
(3)
g=
t1
t
Ako oko duzine AB rotiramo pravokutni trapez ADEB za 180 , on ce prijeci u osjencani
trapez AD1 E1 B s osnovicama |AB| = v1 i |D1 E1 | = v2 i visinom |AD1 | = |AD| = t
i imat c e, naravno, istu povrsinu kao i trapez ADEB. Sa slike je ocigledno razlika
- povrsina
povrsina pravokutnog trapeza OABC i osjencanog trapeza AD1 E1 B takoder
pravokutnog trapeza OD1 E1 C s osnovicama |OC| = 20 (u m/ s) i |D1 E1 | = |DE| = v2
i visinom |OD1 | = t1 t tj.
20 + v2
(t1 t).
(4)
POABC PADEB =
2
Zamjenom relacija (1) i (2) u (4) dobiva se:
20 + v2
(t1 t) odakle je t1 = t
0=
(5)
2
pa iz (3) dobivamo v2 = 20 (u m/ s).
Zato graf brzine tijela u ovisnosti od vremena, zapravo, izgleda ovako.
S grafa i iz relacije (5) se dobiva
t2 = t1 + t = 2t1 .
S grafa su ocigledne i relacije:
20 v1
g=
i
t1
20 + v1
t1 = 15.
POABC =
2

(6)
(7)
(8)

Iz relacija (7) i (8) se dobiva kvadratna jednadzba: gt12 40t1 + 30 = 0 cija su


rjesenja: t1 = 0.99 (u s) i t1 = 3.09 (u s). Rjesenje t1 = 3.09 nema fizicki smisao
jer se iz (7) za njega dobiva negativna vrijednost za velicinu brzine, s to je nemoguce
(v1 = 20 gt1 = 10.3). Na osnovu rjesenja t1 = 0.99 (u s) i relacije (6) dobiva se
proteklo vrijeme od trenutka izbacaja do povratka tijela
t2 = 2t1 = 2 0.99 = 1.98 (u s)
Ova nestandardna metoda rjesavanja problema kretanja moze se veoma uspjesno
primijeniti i na sve vrste hitaca (vertikalan, kosi, horizontalan), kao i na slaganje
kretanja, pa se naizgled jako teski zadaci mogu lako i brzo rijesiti.

100

Matematicko-fizicki list, LIX 2 (2008. 2009.)

Zadaci za vjezbu
- put od 3 m.
1. Tokom s este sekunde, krecuci se jednoliko usporeno, motociklista prijede
2
Ubrzanje kojim se pri tome kretao ima intenzitet od 2 m/ s . Kolika mu je pocetna brzina? (Rj.
14 m/ s.) [2]
2. Balon se krece vertikalno uvis stalnom brzinom 5 m/ s. Poslije 20 s od balona se odvoji
kuglica.
a) Kolika je najveca visina do koje ce kuglica dospjeti? (Rj. 101.27 m.)
b) Poslije kojeg vremena ce kuglica pasti na Zemlju ako se ono mjeri od trenutka odvajanja
od balona? (Rj. 5.05 s.)
c) Kolika je srednja brzina kuglice na citavom putu? (Rj. 8.08 m/ s.) [2]
3. Ako se pocetna brzina tijela poveca 1.2 puta, pri nepromijenjenom usporenju, zaustavni
put se poveca za 22 m. Koliki je bio zaustavni put prije navedene promjene? (Rj. 50 m.) [2]

4. Cestica,
koja izlijece iz nekog izvora, najprije prelazi 200 m stalnom brzinom v , a zatim
usporava s 2 m/ s2 . Pri kojoj je vrijednosti brzine v minimalno
vrijeme kretanja cestice od
r
l
). [6]
izlijetanja iz izvora do zaustavljanja? (Rj. 20 m/ s; tmin = 2
g
5. Tri kuglice se puste slobodno padati, svaka sljedeca sa zakasnjenjem od jedne sekunde.
Kolike su njihove medusobne
udaljenosti pet sekundi nakon pustanja prve? (Rj. 44.145 m;
34.335 m; 78.48 m.) [1], [2]

Literatura
[1] LJILJANA SUDAR , Geometrija pomaze fizici, Zbornik sa XXVI republickog seminara o
nastavi fizike, Drustvo fizicara Srbije, Vrnjacka Banja 2008., str.123130.
[2] JEVREM JANJIC , MIROSLAV PAVLOV , BRANKO RADIVOJEVIC , Fizika za I razred srednjeg
obrazovanja i vaspitanja, Naucna knjiga, Beograd 1987. str. 129132.

[3] Zadaci i rjesenja rjesenja iz fizike, zadatak OS259.,


zadatak 1357., MFL 1/ 229, god.
LVIII, Zagreb 2007.2008., str. 45, 46.
[4] MILAN S ARIC , BOGOLJUB MARINKOVIC , Razliciti nacini resavanja zadataka II, problemi
kretanja, Materijali za mlade matematicare sv. 83 (10), ArhimedesR. Kasanin, Beograd
Beli Manastir 1995., str. 16.
[5] Kvalifikacijski ispiti, zadatak F25., MFL 2/ 218, god. LV, Zagreb 2004.2005., str. 126.
ALUKOVIC , MILAN RASPOPOVIC , Fizika 1M, Krug, Beograd 2001., str. 9.
C
[6] NATA SA


Bangov teorem
Sve strane tetraedra imaju isti opseg ako i samo ako su
one medusobno
sukladne.
Sve strane tetraedra imaju jednake povrsine ako i samo
ako su one medusobno
sukladne.

Matematicko-fizicki list, LIX 2 (2008. 2009.)

101

Teleskop MAGIC carobni instrument astrocesticne fizike


Dario Hrupec 1 , Koprivnica

Astrocesticna fizika
Astrocesticna je fizika novo, interdisciplinarno, znanstvenoistrazivacko podrucje koje
ukljucuje fiziku c estica, astronomiju i astrofiziku te kozmologiju (dijelom takoder,
nuklearnu fiziku i gravitaciju). Ona istrazuje, izmedu ostalog, daleke i egzoticne objekte
poput pulsara 2 i supermasivnih crnih rupa. U blizini takvih objekata postoje polja
(elektricno, magnetsko i gravitacijsko) ekstremne jakosti koja uzrokuju ubrzavanje
cestica do vrlo visokih energija te emisiju razlicitih valova (elektromagnetskih i
- do Zemlje i ondje budu opazeni, nazivamo

gravitacijskih). Cestice
i valove, koji dodu
prenositeljima (engl. messenger). Istovremeno opazanje razlicitih prenositelja koji dolaze
iz istog kozmickog objekta, s to je danas u astronomiji trend, nazivamo kombiniranim
pristupom (engl. multimessenger approach) [1].

Gama-astronomija
Jedna od najuspjesnijih grana astrocesticne fizike danas je gama-astronomija. Ona
istrazuje kozmicke gama-zrake, odnosno elektromagnetsko zracenje najvisih energija koje
dolazi iz svemira [2]. Nize energijsko podrucje kozmickih gama-zraka (do par desetaka
GeV 3 ) dostupno je, neposredno, samo detektorima na satelitima. Trenutno je u orbiti
nekoliko satelita koji opazaju kozmicke gama-zrake, no najveci i najperspektivniji medu
njima je nedavno lansirani satelit Fermi (bivsi GLAST) [3]. Vise energijsko podrucje
kozmickih gama-zraka (od par desetaka GeV do par desetaka TeV) dostupno je,
posredno, detektorima na Zemlji. Visokoenergijske gama-zrake (kao i visokoenergijske
nabijene cestice, takozvane kozmicke zrake) izazivaju u atmosferi velike pljuskove
sekundarnih c estica. Nabijene sekundarne cestice u pljusku, cija je brzina veca od

zracenje.
brzine svjetlosti 4 u zraku, emitiraju posebnu vrstu svjetlosti Cerenkovljevo

Cerenkovljevo
zracenje iz pljuska sekundarnih cestica u atmosferi moguce je opaziti
- Boskovic u Zagrebu, e-mail: dario.hrupecirb.hr
Autor je visi asistent u Institutu Ruder
Pulsar je brzorotirajuca neuronska zvijezda, kozmicki objekt ogromne gustoce, sredisnji ostatak eksplozije
supernove.
3 GeV je gigaelektronvolt, 109 eV, a TeV teraelektronvolt, 1012 eV.
4 Brzina svjetlosti u vakuumu, c , grani
cna je brzina u prirodi. Medutim,
kroz prozirnu tvar (npr. staklo, vodu
c

ili zrak) svjetlost putuje sporije, brzinom v = , gdje je n indeks loma tvari. Cestica
se kroz tvar moze gibati
n
brzinom koja je veca od v , ali jos uvijek manja od c .
1
2

102

Matematicko-fizicki list, LIX 2 (2008. 2009.)


posebnom vrstom teleskopa Cerenkovljevim
teleskopom. Danas postoji nekoliko

sustava Cerenkovljevih
teleskopa, H.E.S.S. u Namibiji [4], MAGIC na Kanarima [5],

VERITAS u Arizoni te CANGAROO III u Australiji. Cerenkovljevi


teleskopi su iznimno
uspjesni znanstveni instrumenti koji gotovo svakodnevno donose nova otkrica. Vodecu
ulogu u zemaljskoj gama-astronomiji danas drze dva europska 5 sustava teleskopa,
- zajednicki rade na buducem
MAGIC i H.E.S.S. Kolaboracije MAGIC i H.E.S.S. takoder

velikom sustavu, od par desetaka do mozda i stotinu, Cerenkovljevih


teleskopa CTA
(Cherenkov Telescope Array) koji bi trebao proraditi za pet do deset godina [6].

Slika 1. Teleskop MAGIC na kanarskom otoku La Palmi. Izvor: Robert Wagner, MPI, Munchen.

Uvod u teleskop MAGIC


Teleskop MAGIC (Major Atmospheric Gamma Imaging Cherenkov) najveci je

Cerenkovljev
teleskop, promjera 17 m, i trenutno jedan od znanstveno najproduktivnijih
instrumenata iz podrucja astrocesticne fizike. Smjesten je na vrhu kanarskog otoka La
Palma, na visini 2200 m, unutar opservatorija Roque de los Muchachos. Teleskop je
dovrsen krajem 2003. godine i opaza punim kapacitetom od 2004. godine. Pri samom
je zavrsetku drugi teleskop, MAGIC II, koji bi trebao proraditi pocetkom 2009. godine.
Oba ce teleskopa moci raditi samostalno ili u paru. Kolaboracija MAGIC broji oko 150
clanova, cak iz 17 institucija. Od ove godine c lanica kolaboracije MAGIC je i hrvatska
- Boskovic,
grupa (Croatian MAGIC Consortium) od s est fizicara s Instituta Ruder

Sveucilista u Splitu i Sveucilista u Rijeci. Za razliku od optickih teleskopa, Cerenkovljev


teleskop nema cjeloviti reflektor nego segmentirani, sastavljen od mnostva manjih zrcala,
kako bi se relativno jeftino dobila s to veca sabirna povrsina. Teleskop MAGIC ima

teleskopa ne
ukupnu povrsinu reflektora od 236 m2 . Nadalje, kamera Cerenkovljevog
- CCD (Charge-Coupled Device), kao kod vecine optickih teleskopa,
sastoji se od uredaja
nego od fotomultiplikatora 6 . To su, zasad, jedini prikladni fotosenzori za biljezenje vrlo

kratkotrajnih bljeskova Cerenkovljeve


svjetlosti. Kamera teleskopa MAGIC II sadrzavat

c e, po prvi put u povijesti Cerenkovljevih


teleskopa, napredne fotosenzore tipa HPD
(Hybrid Photo-Detector). U razvoju fotosenzora HPD vaznu su ulogu odigrali i hrvatski
fizicari, posebice prof. dr. Daniel Ferenc sa Sveucilista u Kaliforniji, Davis.
5

Oba su sustava zemljopisno smjestena na africkom kontinentu, zbog klime, no vode ih europske kolaboracije.
Fotomultiplikator je iznimno osjetljiv detektor svjetlosti koji moze opaziti cak pojedinacne cestice svjetlosti,
fotone. Posebne elektrode u cijevi, takozvane dinode, djeluju kao pojacivac, ili multiplikator, signala.
6

Matematicko-fizicki list, LIX 2 (2008. 2009.)

103

Ciljevi i rezultati teleskopa MAGIC


Popis ciljeva teleskopa MAGIC iznimno je opsezan. U prvom redu tu su
izvangalakticki objekti i pojave: aktivne galakticke jezge i provale gama-zracenja
(engl. gamma ray burst, GRB), zatim galakticki objekti: pulsari, dvojni sustavi
kompaktnih kozmickih objekata 7 te ostaci supernova. Vezano uz kozmologiju, medu
ciljevima su primjerice izvangalakticka pozadinska svjetlost te, danas jako aktualna,
tamna tvar.

Slika 2. Umjetnicki prikaz aktivne galakticke jezgre. Vidi se akrecijski disk i dva relativisticka
mlaza. U samom sredistu je supermasivna crna rupa. Izvor: NASA.

- vaznijim
Popis dosad ostvarenih rezultata je velik i izvan opsega ovog teksta. Medu
otkricima, koja su nedavno objavljena u uglednom c asopisu Science, mogu se spomenuti:
otkrice visokoenergijskih gama-zraka iz udaljenog kvazara 3C279 8 te otkrice pulsnog
gama-zracenja iz pulsara Crab 9 .

Literatura
[1] D. HRUPEC , Kombinirani pristup u astronomiji, MFL LVI 4 (2005/ 2006)
[2] D. HRUPEC , Gama-astronomija posljednji elektromagnetski prozor u svemir, MFL LVI
1 (2005/ 2006)
[3] http://fermi.gsfc.nasa.gov/
[4] http://www.mpi-hd.mpg.de/hfm/HESS/
[5] http://magic.mppmu.mpg.de/
[6] http://www.cta-observatory.org/
7 Kompaktni kozmi
cki objekti su objekti vrlo velike gustoce, bijeli patuljci, neutronske zvijezde ili crne rupe,
koji nastaju nakon smrti obicnih zvijezda. Nase Sunce ima relativno malu masu pa ce, kad potrosi sve svoje
nuklearno gorivo, zavrsiti kao bijeli patuljak za otprilike pet milijardi godina.
8 Objavljeno u casopisu Science, 27. lipnja 2008.
9 Objavljeno u ScienceXpress, 16. listopada 2008.

104

Matematicko-fizicki list, LIX 2 (2008. 2009.)

Profesor Strogi bio je toga dana iznimno


dobrog raspolozenja. Poceo je veselo:
Proslo je vec sedam godina ovoga
stoljeca, a i osma je pri kraju. Danas cemo
ih malo srediti.
Strogi nas godinama sreduje.
Bojim se da c e nas i ovaj puta puno
srediti! culo se otpozadi.

Cuo
sam vas! Ali danas nemam
vremena za prodiku. Treba raditi. Evo, na
ploci pisem sve brojke po velicini. Vas
- nekih
je zadatak da osam puta izmedu
brojki stavite znakove racunskih radnji,
a po potrebi i zagrade, tako da dobijete
izraze kojima su redom vrijednosti 2001,
2002, 2003, 2004, 2005, 2006, 2007, 2008.
Posebno cu ocijeniti ona rjesenja koja nisu
na mome popisu rjesenja. Rjesavanje vam
mogu olaksati ove jednakosti: 91 22 =
2002 , 667 3 = 2001 , 654 3 = 1962 . Na
posao!

Sela A, B i C c ine vrhove pravilnog


trokuta.U selu A z ivi 30 s kolaraca, u selu
B ih je 20, dok je u selu C njihov broj
najmanji i iznosi 10. Svi s kolarci isli su u
s kolu u prilicno udaljenom mjestu i zato su
z itelji tih triju sela odlucili sagraditi s kolu
u svojoj blizini.

Gdje treba sagraditi tu s kolu da zbroj


udaljenosti, koje svi s kolarci moraju prijeci
od svojih sela do s kole, bude najmanji?
Brat i sestra Veselic odlucili su odmah
rijesiti domacu zadacu kako bi imali vise
slobodnog vremena za odmor. Ali na
pocetku nije sve krenulo najbolje.
Nas profesor Dobric ovaj put je
pretjerao ljutio se za svojim radnim
Matematicko-fizicki list, LIX 2 (2008. 2009.)

stolicem brat Branko. Dati za domacu


- zadnje znamenke
zadacu da se nadu
ovakvih potencija 550 , 8800 , 99000 ! Zar
on misli da cu ja citav vikend mnoziti
silne petice, osmice i devetke?
Za drugim stolicem sestra Snjezana se
smjeskala:
Da, da, na velikim si mukama! Samo
si zaboravio uputu naseg dobrog profesora.
Na moras stvarno mnoziti bazu po bazu
da bi doznao kojom znamenkom zavrsava
svaka od tih potencija.
je Branko zaboravio?
Sto
Ako sredista susjednih strana kocke povezemo duzinama, dobiva se geometrijsko
tijelo koje ima 6 vrhova.

Evo pitanja:
1) Koje je to tijelo?
2) Koliki je obujam toga tijela ako je
duljina brida kocke 1?
Na crtezu je krug razdijeljen sa 4 pravca
na 9 dijelova. Pokazite da je moguce
odabrati 4 pravca koji krug dijele na manji,
odnosno veci broj dijelova od 9. Svaki
pravac treba pritom obodnu kruznicu sjeci
u dvije tocke.

Koji se najmanji, a koji najveci broj dijelova moze dobiti na taj nacin? Pokusajte
rijesiti zadatak ako je broj pravaca 5 ili 6.
Zdravko Kurnik, Zagreb

105

Redakcija, iz tehnickih razloga, daje ovo


upozorenje:
Krajnji rok za primanje rjesenja iz ovog
broja je 28. veljace 2009. Rjesenja (i imena
rjesavatelja) bit ce objavljena u br. 4/ 236.
Ujedno molimo da pripazite na upute rjesavateljima koje su na str. 143.

A) Zadaci iz matematike
U zadatku 3131. pojavila se greska pa
navodimo ispravnu formulaciju.
3131. Izracunaj sumu
2
1
+ 4
14 + 12 + 1
2 + 22 + 1
2008
+... +
.
20084 + 20082 + 1
3133. Ako su p i p2 + 8 prosti brojevi
dokazi da je i p2 + 4 prost broj.
3134. Za dani prirodan broj n pokazi da je
64n+1 63n 64 djeljivo s 3969. Opcenito,
dokazi da je za svake prirodne brojeve n i a
broj (a + 1)n+1 an (a + 1) djeljiv s a2 .
3135. Dokazi da su za svaki prirodan broj
n 5 prvih n decimala broja
p
(n + n2 + 1)n
jednake nuli.
- sva pozitivna rjesenja jednazbe
3136. Nadi
q
p
p
x 1 y2 + y 2 z2 + z 3 x2 = 3.
3137. Omjer duljina stranica paralelograma
je p : q , a omjer duljina njegovih dijagonala
je m : n . Odredi kutove paralelograma.
3138. Neka je P proizvoljna tocka unutar
s iljastokutnog trokuta ABC i k opisana mu
kruznica. Pravci AP , BP i CP po drugi put
sijeku k u tockama A1 , B1 i C1 . Oznacimo
s A2 , B2 i C2 ortogonalne projekcije tocke P
na pravce BC , CA i AB , tim redom. Dokazi
da su trokuti A1 B1 C1 i A2 B2 C2 slicni.
3139. Tri kruznice polumjera p , q i r sa
sredistima A , B i C , tim redom, dodiruju se

106

izvana u tockama D , E i F . Dokazi da je


omjer povrsina trokuta DEF i ABC jednak
2pqr
.
(p + q)(q + r)(r + p)
3140. U ravnini je zadan konveksni cetverokut ABCD . S njegove unutarnje strane nacrtani
su jednakostranicni trokuti ABQ i CDN , a s
njegove vanjske strane jednakostranicni trokuti
BCM i ADP . Dokazi da je cetverokut MQPN
paralelogram.
3141. Ako za kutove , i trokuta
vrijedi
sin2 + sin2 + sin2 = 2,
dokazi da je on pravokutan.
3142. Za bilo koje tocke A , B , C i D u
prostoru dokazi nejednakost
|AC|2 +|BD|2 +|AD|2 +|BC|2 |AB|2 +|CD|2 .
3143. U koordinatnoj ravnini dane su tocke
je skup svih
A(a, 0) i B(2a, 0) , a > 0 . Sto
tocaka P(x, y) takvih da je <
)PBA = 2<
)BAP ?
Koje tocke na osi x pripadaju tom skupu?
3144. Odredi produkt
!
n
k
Y
2 2
1 tg n
.
2 +1
k=1

3145. Bridovi AB , AC i AD tetraedra


ABCD su u parovima okomiti i njihove duljine
su redom a , b i c . Odredi polumjer sfere
opisane tom tetraedru.
3146. U nekom gradu ima 10 000 bicikala,
svaki je oznacen brojem od 1 do 10 000 tako
da nikoja dva od njih nemaju isti broj. Kolika
je vjerojatnost da je svaka znamenka broja
slucajno izabranog bicikla razlicita od 8?

B) Zadaci iz fizike
OS 286. Kroz bakrenu zavojnicu tece
struja jakosti 0.4 A kad je napon na njenim
krajevima 5.4 V. Nakon mjerenja ucenici
su odmotali z icu s kalema i izvagali je.
Masa joj je iznosila 11 g. Koliki su bili
promjer i duljina bakrene z ice? Gustoca bakra
iznosi 8900 kg/ m3 , a elektricna otpornost
0.017 106 m.
Matematicko-fizicki list, LIX 2 (2008. 2009.)

OS 287. Nekad su se kotaci za zaprezna


kola pravili stavljanjem z eljeznih obruca na
drvene kotace. Na kotac promjera 80 cm
stavljao se z eljezni obruc kojem je promjer bio
4 mm manji od promjera kotaca. Za koliko
stupnjeva je trebalo zagrijati obruc da bi se
mogao staviti na kotac? Koeficijet linearnog
s irenja z eljeza iznosi 1.2 105 K1 .
OS 288. Markov otac z eli prije zime
napuniti cisternu s uljem za lozenje. Njihova
cisterna je pravokutnog oblika, duljine 2 metra,
s iroka 1 metar i visoka 1.5 metara. Napunjena
je uljem do visine 40 cm. Koliko litara ulja
Markov otac mora naruciti da bi napunio
cisternu do vrha? Koliko ce se nakon punjenja
povecati hidrostatski tlak na dnu cisterne?
Gustoca ulja za lozenje je 850 kg/ m3 .
OS 289. Jedan djecak vuce sanjke
na kojima sjedi drugi djecak mase 42 kg.
Sila vucenja iznosi 30 N. Masa sanjki je
6 kilograma. Maksimalna misicna sila djecaka
koji vuce sanjke iznosi 64 N. Hoce li on moci
vuci sanjke ako na njih sjedne treci djecak
mase 50 kg?
1406. Na kojoj je visini ubrzanje slobodnog
pada jednako 25% od ubrzanja slobodnog pada
na povrsini Zemlje?
1407. Metak mase m = 6 g ispaljen je
horizontalno u blok mase M = 2.8 kg koji
miruje na horizontalnoj podlozi koeficijenta
trenja = 0.3 . Metak se zabije u blok i ostaje
u njemu, a blok se pomakne za l = 0.65 m i
- pocetnu brzinu metka!
zaustavi se. Nadi

1409. Elektron se krece brzinom 0.8c .


Kolika je masa elektrona i ukupna energija
(m0 = 9.1 1031 kg) .
1410. Tanki homogeni s tap mase m1 = 2 kg
i duljine l = 2 m ucvrscen je o jedan kraj
pomocu zgloba, a o drugi je privezan na
oprugu tako da je ravnotezni polozaj s tapa
okomit. Opruga ima zanemarivu masu, a
njezina konstanta elasticnosti je k = 6.1 N/ m.
Izracunaj period malih oscilacija s tapa.

1411. Izracunaj doskok skijasa skakaca koji


polijece brzinom od 20 m/ s pod kutom 15
prema horizontalnoj ravnini, uz pretpostavku
da se otpor zraka zanemaruje i da se padina na
koju skijas skace moze aproksimirati kosinom
koja s horizontalnom ravninom zatvara kut od
45 . (Doskok je udaljenost od tocke polijetanja
do tocke u kojoj skijas dodirne padinu.)
1412. U valjkastu posudu polumjera
r = 10 cm ulije se 1 litra vode. U posudu
spustimo staklenu kocku ( = 4000 kg/ m3 ) .
Koliku je silu potrebno upotrijebiti da se kocka
podigne s dna posude, ako je njezina masa
m = 0.75 kg?

C) Rjesenja iz matematike

1408. Na horizontalnoj podlozi lezi kugla


mase M = 2 kg spojena preko opruge konstante
elasticnosti k = 500 N/ m s cvrstim zidom.
Metak mase m = 20 g i brzine v1 = 600 m/ s
zabije se u kuglu i ostaje u njoj.

a) Koliko ce se stisnuti opruga?


b) Odredi period titranja sistema!
Matematicko-fizicki list, LIX 2 (2008. 2009.)

U rjesenju zadatka 3093. pojavila se greska


pa donosimo ispravno rjesenje.
3093. Ako su a , b i c pozitivni realni
brojevi, dokazi nejednakost
1 1
c+a
a+b
1
b+c
+ 2
+
+ + .
a
b
c
a2 + bc
b + ca c2 + ab
Rjesenje.
1 1 1
c+a
a+b
b+c

0
+ +
a b c a2 +bc b2 +ca c2 +ab
.
abc(a2 + bc)(b2 + ca)(c2 + ab)
. . .

107

a4 b4 +b4 c4 +c4 a4 a4 b2 c2 b4 c2 a2 c4 a2 b2
0.
abc(a2 +bc)(b2 +ca)(c2 +ab)
( )
Koristeci nejednakost
x2 + y2 + z2  xy + yz + zx

1
[(x y)2 + (y z)2 + (z x)2 ]  0
2

imamo za x = a2 b2 , y = b2 c2 , z = c2 a2 :
(a2 b2 )2 + (b2 c2 )2 + (c2 a2 )2
 a2 b2 b2 c2 + b2 c2 c2 a2 + c2 a2 a2 b2
a4 b4 + b4 c4 + c4 a4
a4 b2 c2 b4 c2 a2 c4 a2 b2  0.
Zato je ispravna nejednakost () .
Jednakost vrijedi ako i samo ako je
a = b = c.
Edin Ajanovic (3),
Prva bosnjacka gimnazija, Sarajevo, BiH
3105. Dokazi da niz brojeva an =
3n + 4, n N ne sadrzi nijedan kvadrat
prirodnog broja.
Rjesenje. Pretpostavimo suprotno, tj. da za
neki n vrijedi 3n + 4 = m2 , m N . Buduci
da 3  | m2 , onda ni 3  | m ,
3n = m2 4,
3n = (m + 2)(m 2).
Jer je 3 prost broj, oba faktora moraju biti
djeljiva s 3 ili jedan od njih mora biti jednak
1.

3106. Neka su a , b , c realni brojevi takvi


da je abc = 0 . Ako je
b
c
c
a
a
b
x= + , y= + , z= + ,
c
b
a
c
b
a
dokazi
x2 + y2 + z2 xyz = 4.
Rjesenje.
2

x + y2 + z2 xyz = 4,
x2 + y2 + z2 = 4 + xyz,
2
2
2

c
a
b
c
a b
+
+
+
+
+
c
b
a
c
b a

b
c
a
c
a
b
=4+
+
+
+
,
c
b
a
c
b
a
c2
c2
a2
a2
b2
b2
+2+ 2 + 2 +2+ 2 + 2 +2+ 2
2
c
b
a
c
b
a

ab
a
a b
b
c2
+ 2 +
+
+
,
=4+
a
ab
b
b a
c
b2
c2
c2
a2
a2
b2
+ 2 + 2 + 2 + 2 + 2 +6
2
c
b
a
c
b
a
a2
b2
c2
c2
a2
b2
= 4 + 1 + 2 + 2 + 2 + 2 + 2 + 2 + 1.
a
c
c
b
a
b
Na kraju i lijevo i desno imamo iste clanove
koji se svi medusobno
eliminiraju.
David Devald (1),
SS Izidora Krsnjavoga, Nasice
3107. Nadi- sva cjelobrojna rjesenja jednadzbe
(x2005)(x2006)(x2007)(x2008) = 3024.

Kad bi oba broja bila djeljiva s 3, tad bi


i njihova razlika bila djeljiva s 3, no njihova
je razlika jednaka 4, stoga barem jedan od tih
brojeva nije djeljiv s 3. Dakle, preostaje jedino
da je jedan od tih faktora jednak 1.

Rjesenje. Zamjenom x 2005 = a jednadzba postaje


a(a 1)(a 2)(a 3) = 3024,

Za m + 2 = 1 je m = 1 s to nije moguce,
jer je m prirodan broj.

Supstitucijom a2 3a = b dobivamo
b(b + 2) = 3024,

Za m 2 = 1 je m = 3 s to nije moguce,
jer m nije djeljiv s 3.
Dakle, niz (an ) ne sadrzi nijedan kvadrat
prirodnog broja.
Kristijan Kvaternik (1),
V. gimnazija, Zagreb

108

(a2 3a)(a2 3a + 2) = 3024.

b2 + 2b 3024 = 0.
Rjesenja ove jednadzbe su b1 = 54
b2 = 56 .
Za b1 = 54 je

a2 3a = 54,
Matematicko-fizicki list, LIX 2 (2008. 2009.)

z2 + zx + x2
3 1
1
.

+
zx
2 z
x
Zbrajanjem tih triju nejednakosti dobivamo
zadanu nejednakost:
p
p

x2 +xy+y2
y2 +yz+z2
z2 +zx+x2
+
+
xy
yz
zx

1 1
1
1 1
3 1
+ + + + +

2 x
y
y
z
z
x

1
1
1
+ +
.
 3
x
y
z

a2 3a 54 = 0,
a1 = 9, a2 = 6,
x1 = 2014, x2 = 1999.
Za b2 rjesenje je
a2 3a = 56,
a2 3a + 56 = 0.
Jednadzba nema realnih rjesenja.
Dakle, x {1999, 2014}

David Devald (1), Nasice

Kristijan Kavternik (1), Zagreb


3108. Za pozitivne realne brojeve x , y , z
dokazi nejednakosti:

q
3
x2 + xy + y2
(x + y) ;
a)
2
b)
p
p

x2 +xy+y2
y2 +yz+z2
z2 +zx+x2
+
+
xy
yz
zx

1 1
1
+ +
3
.
x
y
z

Rjesenje. a)

x2 + xy + y2 

3
(x + y) ,
2

3 2
(x + 2xy + y2 ),
4
x2 2xy + y2  0,
x2 + xy + y2 
2

(x y)  0.
b) Iskoristit cemo dokazanu nejednakost iz
a):

3
(x + y).
2
Podijelimo tu nejednakost s xy :
p

x2 + xy + y2
3x+y

,
xy
2 xy
p

x2 + xy + y2
3 1 1

+
.
xy
2 x
y
x2 + xy + y2 

Analogne nejednakosti dobivamo za parove y


i z , odnosno z i x :
p

y2 + yz + z2
3 1 1

+
,
yz
2 y
z
Matematicko-fizicki list, LIX 2 (2008. 2009.)

U decimalnom zapisu broja (5 +


3109.
26)n odredi prvih n znamenaka poslije
decimalne tocke.

Rje
senje. Neka je = 5 + 26 , =
5 26 . Brojevi i su rjesenja koordinatne
jednadzbe
x2 10x 1 = 0.
Ako je

n = an + bn 26,

tada je

n = an bn 26,

pa je

n + n = 2an = An
prirodan broj. Odatle slijedi
An n = n

|An n | = |n | =

26)n

<

1
.
10n

(5 +
- n i An po
Prema tome, razlika izmedu
1
. Kako
apsolutnoj je vrijednosti manja od
10n
je < 0 za parno n prvih n znamenaka poslije
decimalne tocke su devetke, a za neparno n
prvih n znamenaka poslije decimalne tocke su
nule.
Ur.
3110. Odredi cjelobrojno rjesenje jednadzbe
p

3
3
3
1 + 2 + . . . + x3 1 = 400.
( x je oznaka za najveci cijeli broj koji nije
veci od x .)

3
Rjesenje. Kako je 3 8 = 23 = 2 , prvih
7 clanova ima vrijednost 1, pa je njihov zbroj

109

3
7 1 = 7 . Zatim, 3 27 = 33 = 3 pa svi
clanovi od 8. do 26. imaju vrijednost 2. Njihov

3
zbroj je 19 2 = 38 . 3 64 = 43 = 4 , pa svi
clanovi od 27. do 63. imaju vrijednost 3. Njihov

3
zbroj je 373 = 111 . Dalje 3 125 = 53 = 5 ,
pa svi clanovi od 64. do 124. imaju vrijednost
4. Njihov zbroj je 61 4 = 244 . Zbrojimo
7 + 38 + 111 + 244 = 400 . Zaklju
cimo da je

3
zadnji clan jednak 3 124 = x3 1 iz
cega slijedi x3 = 125 , odnosno x = 5 .
David Devald (1), Nasice
3111. Ako je zbroj cijelih brojeva a , b , c
jednak 0, dokazi da je 2(a4 + b4 + c4 ) kvadrat
cijelog broja.

Uz oznake kao na slici dani uvjet mozemo


zapisati u obliku 2b x = 2(a + x) . Iz
slicnosti ABD i BCE slijedi a : x =
a2
2b : a , odnosno x =
. Uvrstavanjem
2b
x u pocetnu jednadzbu dobivamo kvadratnu
jednadzbu 4b2 4ab 3a2 = 0 . Za pozitivno
b
3
rjesenje dobivamo
= . Zbog cos =
a
2
a
1
= slijedi = 70 32 , a = 90 =
2b
3
19 28 .
Vanja Ubovic (2),
Gimnazija P. Preradovica, Virovitica

Rjesenje. Kako je a + b + c = 0 imamo

3113. Ako su , , kutovi s iljastokutnog


trokuta, dokazi da vrijedi nejednakost

a2 + b2 + c2 = 2(ab + bc + ca).

sin + sin + sin 2(cos + cos + cos ).

Tada je
(a2 + b2 + c2 )2 = 4(ab + bc + ca)2 ,
a4 + b4 + c4 + 2(a2 b2 + b2 c2 + c2 a2 )

Rjesenje.

= 4(a2 b2 + b2 c2 + c2 a2 ) + 8abc(a + b + c),


tj. a4 + b4 + c4 = 2(a2 b2 + b2 c2 + c2 a2 ).
Sada je
2(a4 + b4 + c4 )
= a4 + b4 + c4 + 2(a2 b2 + b2 c2 + c2 a2 )
= (a2 + b2 + c2 )2 .
Edin Ajanovic (3), Sarajevo, BIH
3112. Tocka D je noziste visine spustene
iz vrha A na krak BC jednakokracnog trokuta
ABC . Ako je |AC| + |CD| = 2(|AB| + |BD|) ,
koliki su njegovi kutovi?
Rjesenje.

Po poucku o sinusima vrijedi a = 2R sin ,


b = 2R sin , c = 2R sin . Za a = |AH|
imamo
|AC | = a cos <
)HAC1 = a cos <
)A1 AB

= a sin tj.
= a cos
2
|AC |
b cos
=
sin
sin
2R sin cos
=
= 2R cos .
sin

a =

Analogno se dobije
b = 2R cos ,

c = 2R cos .

Iz nejednakosti o trokutu vrijedi


a + b > c,
b + c > a,

110

Matematicko-fizicki list, LIX 2 (2008. 2009.)

c + a > b,

odakle je
2(a + b + c ) > a + b + c,

1+

 (1 +

tj.

1
sin cos

2)2 = 3 + 2 2.

Ur.
3114. Kompleksni brojevi zadovoljavaju ove
uvjete:
z1 + z2 = i 1,
z1 z2 = i.
Ne racunajuci z1 i z2 , odredite z1 z2 .
Rjesenje. Primijetimo da je
z
z1 z2 = 1 |z2 |2 .
z2
Iz (z1 + z2 )2 = 2i = 2z1 z2 , dobivamo da je
z1 2 + 4z1 z2 + z2 2 = 0
ili u ekvivalentnom obliku

2
z
z1
+ 4 1 + 1 = 0.
z2
z2
z1
realno i negativno. Zato
Vidimo da je
z
- 2 realno i negativno. Kako
je z1 z2 takoder
je |z1 z2 | = |z1 z2 | = 1 , konacno dobijemo
z1 z2 = 1 .
Ur.

vrijedi
3115. Dokazi da za 0,
2
nejednakost

1
1
1+
3 + 2 2.
1+
sin
cos
Prvo rjesenje.
fi
fl

0,
= sin , cos > 0 ,
2

1
1
1+
1+
sin
cos

1
1
1
+
+
=1+
sin
cos
sin cos
2
1
1+
+
sin cos
sin cos
Matematicko-fizicki list, LIX 2 (2008. 2009.)

1+

2
sin 2

Ur.

2(cos + cos + cos > sin + sin + sin .


Dakle, vrijedi stroga nejednakost.

Drugo rjesenje. Koristit cemo Huygensovu


nejednakost: Ako su realni brojevi xi > 0 ,
i = 1, 2, . . . , n , tada vrijedi nejednakost
n

(1+x1 )(1+x2 ) . . . (1+xn )  1+ n x1 x2 . . . xn .


Iz dane nejednakosti slijedi

1
1
1
1+
 1+
1+
sin
cos
sin cos

AG
2

1+
 (1+ 2)2 = 3+2 2.
sin cos
( )
Radi poznate nejednakosti

2
= 2,
0 < sin + cos  2
2
odnosno
1
1
 ,
sin + cos
2
slijedi () . Jednakost
vrijedi
ako
i samo ako

2
, tj. ako i samo ako je
je sin = cos =
2
= 45 .
Vanja Ubovic (2), Virovitica
3116. Koliko se najvise lovaca moze
postaviti na s ahovsku plocu n n tako da
se nikoja dva ne napadaju.
Rjesenje. Promotrimo dijagonale s ahovske
ploce n n .

(Tih dijagonala ima 2n 1 .)


Na svakoj od tih dijagonala smije stajati
najvise jedan lovac. No na dijagonali 1 i 2n1

111

ne mogu istovremeno stajati dva lovca. (To su


polja na suprotnim kutovima ploce.)
Zakljucak: najvise mozemo staviti (2n
1) 1 = 2n 2 lovaca.
No postoji razmjestaj 2n 2 lovaca uz
zadane uvjete:

2n
2n + 1

,
n+1
a2n 1
a2 1

pa je
Sk =

k
X
n<0

2n
+1

n
a2

k
X
n=0

2n + 1
2n
2n+1
n
2
a 1
a
1

1
2k+1
,
2k+1
a1
a
1

2k+1
1
k+1
S = lim Sk = lim
k
k a 1
a2 1
1
=
.
a1
=

jer je lim
Ur.
3117. Nadi- vjerojatnost da slucajno izabrani dvoznamenkasti broj bude djeljiv barem ili
s 3 ili sa 7.
- u kojem je
Rjesenje. Neka je A dogadaj
- u
izabran broj djeljiv s 3, a B dogadaj
kojem je izabran broj djeljiv sa 7. Treba naci
- A
vjerojatnost p(A + B) , pri cemu se dogadaji
i B ne iskljucuju. Dvoznamenkastih brojeva
11, 12, . . . , 99 ima 90. Od njih je 30 djeljivo
- A ), 13 sa
s 3 (povoljni slucajevi za dogadaj
7 (povoljni slucajevi za dogadaj B ) i 4 su
djeljiva i s 3 i sa 7 (povoljni slucajevi za AB ).
Zato je
p(A + B) = p(A) + p(B) p(AB)
30 13
4
13
=
+

=
= 0.43.
90 90
90
30
Edin Ajanovic (3), Sarajevo
3118. Nadi- sumu reda

X
2n
,
n
2
a +1
n=0

gdje je a > 1 .
Rjesenje. Za a > 1 imamo
2n (a2n 1)
2n (a2n + 1) 2n + 1
2n
= 2n+1
=
n+1
+1
a
1
a2 1

n
a2

112

2k + 1
= 0.
1

k+1
k a2

Edin Ajanovic (3), Sarajevo

D) Rjesenja iz fizike
OS 278. Lukina baka stanuje na c etvrtom
- katova je 3 metra.
katu. Visinska razlika izmedu
Luka je izmjerio da lift putuje do c etvrtog kata
12 sekundi. Snaga elektromotora koji pokrece
kabinu lifta je 6 kW. Kolika je korisnost motora
ako je masa kabine, zajedno s ljudima u njoj,
500 kg?
Rjesenje.
h = 4 3 m = 12 m
t = 12 s
Pu = 6 kW
m = 500 kg

=?
G = m g = 500 kg 10 N/ kg
G = 5000 N
W = G h = 5000 N 12 m
W = 60 000 J
W
60 000 J
Pk =
=
t
12 s
Pk = 5000 W = 5 kW
Matematicko-fizicki list, LIX 2 (2008. 2009.)

Pk
5 kW
=
Pu
6 kW
= 0.83 = 83%

A1 = a a = 2 cm 2 cm

A1 = 4 cm2

Arijan Golub (8),


OS Augusta Cesarca, Krapina
OS 279. Napravi njihalo od konca i
utega proizvoljne mase. Odredi period takvog
njihala za duljine konca 10, 20, 30, 40
i 50 cm. Nacrtaj dijagram koji prikazuje
ovisnost perioda njihala o duljini konca. Je li
ta ovisnost linearna?
Rjesenje.
l (cm)

t (s)

50
40
30
20
10

10
10
10
10
10

14.3
12.6
11.1
9.53
6.8

t
(s)
N
1.43
1.26
1.11
0.95
0.68

T=

Ovisnost perioda o duljini niti nije linearna.


Ana Paic (7),

OS Fausta Vrancica, Sibenik


OS 280. Stolica ima c etiri noge
kvadratnog presjeka. Njihova je debljina 2 cm.
Masa stolice je 4 kilograma. Kolika bi trebala
biti masa c ovjeka koji bi sjeo na tu stolicu da
tlak pod njenim nogama iznosi 0.5 MPa?
Rjesenje.
n=4
a = 2 cm
ms = 4 kg
p = 0.5 MPa = 500 000 Pa
mc =?
Matematicko-fizicki list, LIX 2 (2008. 2009.)

A = 4 A1
A = 16 cm2 = 0.0016 m2
G=F =pA
G = 500 000 Pa 0.0016 m2
G = 800 N
Gs = ms g = 4 kg 10 N/ kg
Gs = 40 N
Gc = G Gs
Gc = 760 N
G
760 N
mc = c =
g
10 N/ kg
mc = 76 kg
Juraj Krsnik (8),
OS Augusta Cesarca, Krapina
OS 281. Keopsova piramida je najstarije
od sedam starih svjetskih c uda. Sagradena
je oko 2560 g.p.n.e. To je najveca i
najteza gradevina
ikad sagradena
na Zemlji.
Sagradena
je od 2 300 000 kamenih blokova
prosjecne mase 2.5 tona. Sad je visoka
135.75 metara, a duljina stranica osnovice
iznosi 229 metara. Izracunaj koliki postotak u
ukupnom volumenu piramide otpada na kamen,
a koji na prostorije i hodnike. Gustoca kamena
krecnjaka od kojeg su isklesani blokovi iznosi
2600 kg/m3 .
Rjesenje.
m = 2.5 t 2 300 000
= 5 750 000 t = 5 750 000 000 kg
h = 135.75 m
a = 229 m

= 2600 kg/ m3
Volumen piramide:
Vp = S

h
3

Vp = a a

h
3

Vp = 229 229

135.75
3

113

Vp = 2 372 955 m3
Volumen kamena:
m
=
V
m
V1 =

2500 kg
V1 =
2600 kg/ m3
V1 = 0.96 m3
Vk = 0.96 2 300 000
Vk = 2 208 000 m3
2 208 000 m3
Vk
=
= 0.93 = 93%
Vp
2 372 955 m3
93% otpada na kamen, a 7% na prostorije i
hodnike.
Ana Paic (7),

OS Fausta Vrancica, Sibenik


1392. Konac namotan na kalem unutarnjeg
radijusa r i vanjskog R lezi na horizontalnoj
povrsini i moze se kotrljati bez klizanja.
Kolikom brzinom i u kojem smjeru c e se kalem
gibati ako se kraj konca povlaci brzinom v
kao na slici?

v
,
Rr

v = R =

vR
.
Rr
Ur.

1393. Dva tijela masa m1 i m2 spojena


su krutom s ipkom duljine l i postavljena
na horizontalnu podlogu. Koeficijenti trenja
- tijela i podloge su 1 i 2 . Sipka

izmedu
je zanemarive mase dok su tijela zanemarivo
malenih dimenzija u odnosu na duljinu s ipke.
U sredistu s ipke djeluje sila iznosa F okomito
na nju i paralelno s horizontalnom podlogom
kao na slici. Koliki mora biti iznos te sile da
bi s ipka ostala okomita na smjer sile?

Rjesenje. Sile trenja na mase m1 i m2


djeluju u smjeru suprotnom od sile F . Teziste
sistema nalazi se u tocki koja je za
lm2
m1 + m2
udaljena od m1 ; udaljenost tezista od m2 je
l1 =

lm1
.
m1 + m2
Bez gubljenja opcenitosti, mozemo pretpostaviti da je m1 > m2 (teziste se tada nalazi
lijevo od centra, tj. polovice s ipke). Ukupan
moment sile oko tezista je:

l
l1
M = Ftr1 l1 Ftr2 l2 + F
2
lm2
lm1
2 m2 g
= 1 m1 g
m1 + m2
m1 + m2

lm2
l

+F
2
m1 + m2

l
lm m
lm2
= g 1 2 (1 2 ) + F

.
m1 +m2
2 m1 +m2
l2 =

Rjesenje. Kalem se kotrlja bez klizanja, pa


mora postojati sila trenja Ftr koja djeluje u
suprotnom smjeru od brzine v (s hvatistem u
tocki dodira s podlogom). Buduci da se kalem
ne ubrzava, konac se povlaci silom F istog
iznosa (dakako, suprotnog smjera). Moment
sile trenja bit ce veci od momenta sile F , pa
ce kalem rotirati u smjeru kazaljke na satu. Za
vrijeme t kraj konca ce se pomaknuti za v t ,
centar kalema ce se pomaknuti za v t = R t
(gdje je kutna brzina rotacije kalema), a
konac ce se dodatno zamotati za duljinu
r t . Stoga vrijedi:
v t R t = r t,
v = R r,

114

Ako z elimo da s ipka ostane okomita na smjer


sile, moment sile mora biti jednak nuli (u
Matematicko-fizicki list, LIX 2 (2008. 2009.)

f (x + y + 2f ) = (x + f )(y + f ),

protivnom ce doci do rotacije s ipke):


M = 0 =
g

lm1 m2
(1 2 ) = F
m1 + m2

l
lm2

2
m1 + m2

f x + f y + 2f 2 = xy + f x + f y + f 2 ,

lm2
l
lm1 m2

(2 1 ) :
F=g
m1 + m2
2
m1 + m2
2lm1 m2
(2 1 ).
=g
l (m1 + m2 ) 2lm2
Ur.
1394. Dijete mase m njise se na njihaljci
duljine l s kutom maksimalnog otklona od
ravnoteznog polozaja . Dijete vice i time
proizvodi zvuk frekvencije f . Koji raspon
frekvencija c uje roditelj koji stoji ispred
njihaljke daleko u odnosu na svoju visinu?
Rjesenje. Maksimalnu brzinu njihanja dijete
postize u trenutku kada je u najnizem polozaju
jer se tada sva potencijalna gravitacijska
energija pretvara u kineticku:

s to je i trebalo pokazati.
Kristijan Kvaternik (1),
V. gimnazija, Zagreb
- dvije identicne
1396. Udaljenost izmedu
metalne kuglice na kojima su razlicite kolicine
naboja je 2 cm. Radijusi kuglica su mnogo
manji od njihove medusobne
udaljenosti.
Privlacna sila izmedu kuglica je 4 105 N.
Nakon s to su kuglice spojene vodljivom zicom
i zatim odspojene, one se medusobno
odbijaju
silom od 2.25105 N. Odredite pocetne naboje
na kuglicama.
Rjesenje.
F1 = 4 105 N
F2 = 2.25 105 N
r = 0.02 m

mv2
,
mgl mgl cos =
2
p
v = 2gl(1 cos ).

Q1 , Q2 =?

Minimalnu frekvenciju racunamo pomocu Dopplerovog izraza u situaciji kada se dijete tom
brzinom udaljava od roditelja, a maksimalnu
kada mu se priblizava:
vzvuka
f=f
vzvuka v
v
pzvuka
.
=f
vzvuka 2gl(1 cos )
Ur.
1395. Dokazi da je za sfericna zrcala
umnozak udaljenosti predmeta i slike od zarista
uvijek jednak kvadratu zarisne duljine.
Rjesenje. Oznacimo li s a i b udaljenosti
predmeta i slike od tjemena zrcala, s x i y
udaljenosti predmeta i slike od fokusa, vrijedi:
a = x+f,

f 2 = xy,

b= y+f.

Iz jednadzbe sfernog zrcala:


1 1
1
= +
f
a b
imamo:
f (a + b) = ab,
Matematicko-fizicki list, LIX 2 (2008. 2009.)

Bez smanjenja opcenitosti moze se pretpostaviti


da je kuglica naboja Q1 prije dodira nabijena
pozitivno, a kuglica s nabojem Q2 negativno.
Tada je:
1 Q1 Q2
F1 =
,
40 r2
Q1 Q2 = 40 r2 F1 .
Pri spajanju kuglica doslo je do prelaska
odredenog
pozitivnog naboja Q s kuglice s
nabojem Q1 na kuglicu s nabojem Q2 , tako da
je naboj na kuglicama nakon spajanja jednak
(kuglice su identicne; dakle, imaju jednaki
kapacitet).
1 (Q1 Q)(Q2 + Q)
,
F2 =
40
r2
Q1 Q = Q2 + Q.
Ove tri jednadzbe cine sistem s tri nepoznanice
(Q, Q1 , Q2 ). Eliminiranjem Q2 dobivamo:
(Q1 Q)2 = 40 r2 F2 ,
Q1 (Q1 2Q) = 40 r2 F1 .
Oduzimanjem ove dvije jednadzbe dobivamo:
Q2 = 40 r2 (F2 F1 ),

115

Q = 1.67 109 C,
p
Q1 = 40 r2 F2 + Q = 2.67 109 C,

Rjesenja zabavne matematike

Q2 = Q1 2Q = 0.67 109 C.
Ur.
1397. Elektricna zarulja od 110 V i 60 W
spojena je na bateriju elektromotorne sile od
120 V. Unutarnji otpor baterije je 60 . Hoce
li zarulja svijetliti maksimalnim intenzitetom u
ovakvom spoju?
Rjesenje.
U = 110 V,
P = 60 W,
E = 120 V,
r = 60 .
Otpor z arulje racunamo iz njenih karakteristika:
U2
U2
= R =
= 201.7 .
R
P
Struja koja ce teci kroz z arulju kada je spojimo
na bateriju je:
E
= 0.46 A.
I=
R+r
Na z arulji ce se razvijati snaga
P = UI =

P = I 2 R = 42.7 W,
s to je manje od nominalne snage. Dakle, z arulja
nece svijetliti maksimalnim intenzitetom.
Ur.
1398. Dvije identicne metalne kugle zagrijavaju se tako da im se polumjer poveca za
isti iznos. Jedna od njih lezi na stolu, a druga
je objesena o nit. Obje kugle su toplinski izolirane od stola i niti. Kojoj kugli je dovedeno
vise topline i zasto?
Rjesenje. Kugli koja lezi na stolu pri
povecavanju polumjera raste i gravitacijska
potencijalna energija (jer joj se teziste podize)
pa je ukupna toplina koju joj treba dovesti
veca!
Ur.

116

Tanja je nabrojila 61 pravokutni trokut, a


Perica 40 paralelograma.

Prema rijecima profesora Kubusa lako


zakljucujemo sljedece:
Svaki matematicar dopisuje se na istom
jeziku s barem tri druga. Od ove trojice ili se
dva dopisuju na tome istom jeziku, ili se sva
trojica dopisuju drugim jezikom.
Svaki matematicar dopisuje se na istom
jeziku s barem s est drugih. Od ove s estorice
ili se dva dopisuju na tom istom jeziku, ili se
sva s estorica dopisuju na ostala dva jezika, a
tada se zakljucivanje izvodi kao u prethodnom
slucaju.
Dakle, postoje najmanje tri matematicara
koji se medusobno
dopisuju na istom jeziku.

Broj osvojenih bodova je 190. Ako je


a broj bodova posljednjeg na tablici i r
razlika bodova svaka dva susjedna igraca,
onda su igraci osvojili redom a , a + r ,
a + 2r ,. . . , a + 19r bodova. Zbroj tih bodova
je 20a + 190r , pa mora biti 20a + 190r = 190 .
Jedini par prirodnih brojeva koji zadovoljava
ovu jednakost je a = 0 , r = 1 . Pobjednik je
osvojio 19 bodova, a posljednji na tablici 0
bodova.

Ucenici su otkrili da se broj 666 666, nakon


nalazenja njegova rastava na proste faktore u
obliku 666 666 = 2 3 3 7 11 13 37 , moze
na tri nacina prikazati kao umnozak dvaju
troznamenkastih brojeva:
666 666 = 693 962 = 777 858 = 814 819.
Matematicko-fizicki list, LIX 2 (2008. 2009.)

Matematicka konferencija u c ast akademiku Josipu Pecaricu


Sanja Varosanec 1 , Zagreb
Sudionici matematickih natjecanja cesto se susrecu s jednom posebnom vrstom
zadataka s matematickim nejednakostima. Nejednakost trokuta, Cauchy-SchwarzBunjakowskijeva nejednakost, Jensenova, Holderova, Minkowskijeva nejednakost,
- aritmeticke i geometrijske sredine, te nejednakosti izmedu
- drugih
nejednakost izmedu
sredina, samo su neke od osnovnih nejednakosti koje uz puno maste i blistavih ideja
koristimo pri rjesavanju takvih zadataka. No, nejednakosti se ozbiljno proucavaju i

u matematickoj znanstvenoj zajednici. Siroj


je javnosti mozda manje poznato da u
Zagrebu z ivi i djeluje jedno od najvecih svjetskih matematickih imena iz podrucja

matematickih nejednakosti akademik Josip Pecaric. Cinjenica


da je u 2008. godini
profesor Pecaric navrsio 60. godina bila je povodom za organizaciju medunarodne
konferencije Mathematical Inequalities and Application 2008 (MIA 2008). Konferencija
cija je organizacija pocela dvije godine prije odrzana je od 8. do 14. lipnja 2008.
u Trogiru uz sudjelovanje 135 matematicara iz 27 zemalja od sjevera Europe pa
do Australije. Malo je koja matematicka konferencija u Hrvatskoj privukla toliki broj
sudionika iz cijeloga svijeta.

Mathematical Inequalities and Applications 2008


June 8 -14, 2008, Trogir, Croatia

Conference in honour of Professor Josip Peari on the occasion of his 60 th birthday

Plenary Speakers
Peter S. Bullen
Victor Burenkov
Sever S. Dragomir
Roland Glowinski
Frank Hansen
Alois Kufner
Lech Maligranda
Roy Mathias

Scientific Committee

Marko Mati
Ingram Olkin
Zsolt Ples
Ivan Peri
Lars-Erik Persson
Andrs Prkopa
Gord Sinnamon
Vladimir D. Stepanov

Lars-Erik Persson (Chairman)


Shoshana Abramovich
Ravi P. Agarwal
Tsuyoshi Ando
Drumi Bainov
Catherine Bandle
Richard A. Brualdi
Peter S. Bullen
Victor Burenkov
Wing-Sum Cheung
Yeol Je Cho
Neven Elezovi
Carla Giordano
Roland Glowinski
Pankaj Jain
Anna Kaminska
Alois Kufner
Andrea Laforgia
Chi-Kwong Li
Lech Maligranda
Marko Mati
Ram N. Mohapatra
Edward Neuman
Constantin Niculescu
Ludmila Y. Nikolova
Ryskul Onarov
Ingram Olkin
Bohumir Opic
Charles E. M. Pearce
Ivan Peri
Andrs Prkopa
Saburou Saitoh
Gord Sinnamon
Hari M. Srivastava
Vladimir D. Stepanov
George P. H. Styan
Nenad Ujevi
Sanja Varoanec

Organizing Committee
Aleksandra imeija
Neven Elezovi
Milica Klarii Bakula
Marko Mati
Sanja Varoanec

INFORMATION: http://mia2008.ele-math.com

CONTACT: mia2008@math.hr

Faculty of Natural Sciences, Mathematics and Kinesiology, University of Split

DEADLINE FOR ABSTRACTS: May 1, 2008

Faculty of Textile Technology, University of Zagreb

Croatian Mathematical Society

foto: Jelena Popi

Okosnicu konferencije c inilo je 15 jednosatnih predavanja pozvanih znanstvenika


P. S. Bullen (Kanada) osvrnuo se na najzanimljivija Pecariceva otkrica, V. Burenkov
(Velika Britanija) iznio je najnovije rezultate o multiplikativnim visedimenzionalnim
nejednakostima u tezinskim Lebesgueovim prostorima, S. S. Dragomir (Australija) dao je
1

Dr. sc. Sanja Varosanec je redovita profesorica PMF-Matematickog odjela Sveucilista u Zagrebu.

Matematicko-fizicki list, LIX 2 (2008. 2009.)

117

pregled aproksimacija Riemann-Stieltjesovog integrala, R. Glowinski (USA) diskutirao


je rjesenja neglatkog problema svojstvenih vrijednosti, o operatorskim nejednakostima s
primjenama predavanje je odrzao F. Hansen (Danska), a o nejednakostima Hardyjevog
- kvazi-Banachovih Lp - Lq prostora bile
ske. Norme izmedu
tipa A. Kufner iz Ce

su tema predavanja L. Maligrande ( Svedska),


a M. Matic (Split) pozabavio se
Eulerovim tipovima nejednakosti. Profesor I. Olkin (USA) u svom je predavanju
- momenata, sredina, matricnih i
napravio zanimljivo povezivanje nejednakosti izmedu
probabilistickih nejednakosti, a o nejednakostima za konveksne funkcije viseg reda culi
smo od A. Prekope (USA). Sredine i to generalizirane kvazi-aritmeticke bile su tema
predavanja Zs. Palesa (Madarska),
a mjesovite sredine bile su tema predavanja I. Perica

(Zagreb). Predsjednik znanstvenog odbora konferencije Lars-Erik Person ( Svedska)


odrzao je predavanje o Hardyju i njegovom uplivu na teoriju nejednakosti, dok je
G. Sinnamon prezentirao rezultate o tezinskim Fourierovim nejednakostima, a V. D.
Stepanov (Rusija) o nejednakostima za integralne operatore.

Ostali sudionici konferencije svoje su rezultate izlozili u vidu kratkih 20-minutnih


priopcenja. Zbog velikog broja prijavljenih priopcenja (njih 107) rad je bio organiziran
u tri paralelne sekcije, a sve se odvijalo svakodnevno od 9 sati ujutro pa do
veceri, uz pauzu za rucak. Predavanja su obuhvatila gotovo sve aspekte matematickih
nejednakosti. Izlagalo se o analitickim nejednakostima, o geometrijskim, integralnim,
operatorskim i matricnim, o varijacijskim, o nejednakostima u apstraktnim strukturama, o
primjenama nejednakosti u metodi konacnih elemenata, u ocjenama numerickih formula,
pri rjesavanju funkcijskih jednadzbi, nelinearnih sustava diferencijalnih jednadzbi, te
o problemima konveksnog programiranja. A naravno, odrzati konferenciju u Trogiru
i ne obici neke znamenitosti u okolici bilo bi nedopustivo. Sudionici konferencije
posjetili su Nacionalni park Krka, te obisli Split i Trogir. Na stranici konferencije
(http://mia2008.ele-math.com) nalaze se detalji o konferenciji, a tamo je osim
sluzbenih informacija moguce naci i niz slika koje docaravaju atmosferu.
Buduci da je konferencija MIA 2008 organizirana u c ast 60. rodendana
profesora
Pecarica, razumljivo je da su se predavaci u velikoj mjeri dotakli i rezultata rada
profesora Pecarica s to je potpuno i razumljivo buduci da je on jedan od vodecih
strucnjaka u podrucju matematickih nejednakosti i jedan od najproduktivnijih svjetskih
matematicara s vise od 700 objavljenih ili prihvacenih za tisak znanstvenih radova, sa 70ak radova na konferencijama i u knjigama, 60-ak iz elektrotehnike, geofizike, geologije,
gradevinarstva,
fizike i povijesti, 14 matematickih monografija te dva visokoskolska

118

Matematicko-fizicki list, LIX 2 (2008. 2009.)

udzbenika. Bio je deseti matematicar u svijetu koji je na MathSciNetu imao preko 500
radova. Tako, na primjer, ISI Web of Knowledge u bazi SCIE prikazuje da se radovi
J. Pecarica citiraju 542 puta u 310 radova u casopisima te baze, a u monografiji P.
S. Bullena Handbook of Means and Their Inequalities, Mathematics and Applications
(Kluwer, 2003) Pecaric je najcitiraniji autor (7 knjiga i 105 c lanaka). Ovi brojevi
citiranosti kazuju koliki je odjek pojedinih radova u znanstvenoj zajednici, tj. koliko se
puta drugi matematicari pozivaju na te radove. Njegovim je imenom nazvano nekoliko
identiteta i nejednakosti, a o primjenama Mond-Pecariceve metode na operatore u
Hilbertovim prostorima objavljena je monografija, dok se druga upravo priprema. Za
njegov je rad karakteristicna suradnja preko 150 suradnika iz 26 zemalja, a pod
njegovim je vodstvom izradeno
ili je u izradi 30-tak magistarskih i doktorskih radnji.
Poznato je da svega cetrdesetak matematicara u svijetu ima preko sto suradnika.
- 2. rujna 1948. u Kotoru. Tamo je zavrsio osnovnoskolsko
Josip je Pecaric roden
i srednjoskolsko obrazovanje, a 1972. je diplomirao na Elektrotehnickom fakultetu u
Beogradu s radom iz podrucja nuklearne fizike. Na istom je fakultetu i magistrirao, a
1982. obranio je doktorsku disertaciju iz podrucja matematike s naslovom Jensenove
i srodne nejednakosti. U komisiji su bili profesori Boas i Ky Fan. Dio Boasovog
izvjesataja navodimo u originalu:
Let me say to begin with that I would consider Pecarics thesis acceptable in
any american university, and a better thesis than most of thesis I have seen.
Pecaric has, in the first place, systematized a large amount of material that
appeared in a haphazard way over the years and had become rather chaotic. This
synthesis is a valuable piece of work in itself.
In the second place, Pecaric has shown great ingenuity in finding simpler proofs
of some inequalities and appropriate generalizations of others. Consequently he
has made many inequalities more understandable. He has also been able to unify
some inequalities that originally seemed to be quite unrelated.
Finally, he has made many original contributions to the field.
Do 1987. godine radio je u Geomagnetskom institutu u Grockoj te na Gradevinskom fakultetu u Beogradu, a onda s
obitelji preseljava u Zagreb te se zaposljava na Tekstilno-tehnoloskom fakultetu
Sveucilista u Zagrebu gdje i danas radi
kao redoviti profesor. Od tada do danas
nekoliko je puta kao gostujuci profesor boravio na sveucilistima u Australiji, Italiji,

Pakistanu i Svedskoj.
Godine 1998. pokrenuo je medunarodni
znanstveni casopis
Mathematical Inequalities and Applications (izdavac Element, Zagreb) koji je vec
nakon dvije godine izlazenja uvrsten na Scientific Citation Index Expanded (SCIE) listu,
a prije par godina pokrenuo jos dva casopisa Journal of Mathematical Inequalities i
Operators and Matrices. Urednistva ovih triju casopisa c ini 90-ak uglednih matematicara.
Osim toga objavio je 21 publicisticku knjigu od knjige Srpski mit o Jasenovcu pa do
Za hrvatske vrednote i preko 200 c lanaka u kojima progovara i poucava o povijesti i
sadasnjosti Hrvatske zastupajuci ideje samostalne Hrvatske.
Za svoj doprinos u znanosti dobio je Drzavnu nagradu za znanost za 1996. godinu,
te je odlikovan ordenom Reda Danice hrvatske s likom R. Boskovica. Od 2000. godine
profesor Pecaric redoviti je clan Hrvatske akademije znanosti i umjetnosti.
Matematicko-fizicki list, LIX 2 (2008. 2009.)

119

49. medunarodna
matematicka olimpijada
Ovogodisnja Medunarodna
matematicka olimpijada (IMO) odrzana je u Madridu, u
srpnju 2008. Sudjelovalo je 535 natjecatelja iz 97 drzava. Hrvatsku su predstavljali Nina
Kamcev (XV. gimnazija, Zagreb), Adrian Satja Kurdija, (V. gimnazija, Zagreb), Ines
Marusic (V. gimnazija, Zagreb), Sara Muhvic (III. gimnazija, Osijek), Melkior Ornik
zic (V. gimnazija, Zagreb). Voditelji ekipe bili su
(XV. gimnazija, Zagreb) i Goran Zu

Zeljko
Hanjs i Mea Bombardelli s Matematickog odjela PMF-a u Zagrebu.
Ucenici su krenuli u Madrid 14. srpnja zajedno s voditeljicom Meom Bombardelli,

dok je voditelj ekipe Zeljko


Hanjs otputovao ranije u Spanjolsku
i u gradu La Granja
sudjelovao u izboru zadataka za natjecanje. Tamo nas je docekalo jako s panjolsko sunce
s to nas nije sprijecilo da vec prvi dan s vodicem podemo
u s etnju gradom.
Natjecatelji su bili smjesteni u nekoliko studentskih domova. Na istom katu s nama
su bile smjestene i ekipe iz Makedonije i Vijetnama. S Makedoncima smo otpocetka
ostvarili veoma dobru komunikaciju, dok je s Vijetnamcima bilo malo teze jer njihov
engleski nije bio ni upola tako dobar kao njihovo znanje matematike. Sljedeci dan, na
svecanom otvaranju, upoznali smo se i s ostalim ekipama iz nase regije Bosancima,
Crnogorcima, Slovencima i Srbima, s kojima smo se i kasnije druzili.
Na otvaranju smo, osim tradicionalnih govora o vaznosti matematike, imali prilike
vidjeti i nekoliko cirkuskih nastupa. Sutradan je bio prvi dan natjecanja, na kojem su
zadaci, po misljenju mnogih sudionika, bili laksi nego inace, ali je ipak bilo tesko
nakon prvog dana osigurati bilo kakvu medalju. Nakon drugog dana natjecanja pocelo
je spekuliranje o bodovima koje smo postepeno saznavali tijekom sljedeca dva dana,

dok smo isli na izlete promatrajuci razne ljepote Spanjolske,


kao s to su Segovia, San
Lorenzo de Escorial ili Toledo.
Treci dan nakon natjecanja na Internetu su bili objavljeni cjelokupni rezultati: Adrian
zic i Melkior Ornik osvojili su broncane medalje (prag je bio 15
Satja Kurdija, Goran Zu
bodova, a oni su imali redom 20, 19 i 17), dok su Nina Kamcev i Sara Muhvic dobile
pohvalu za tocno rijesen jedan zadatak (i to na razlicitim zadacima!). Po ukupnom
broju osvojenih bodova Hrvatska je bila 38., s to je najbolji uspjeh u zadnje tri godine.
Kao i obicno, (po broju osvojenih bodova) pobijedili su Kinezi s pet zlatnih medalja
i jednom srebrnom, dok je druga bila ekipa Rusije, ciji je svaki predstavnik osvojilo
zlatnu medalju.
Uvijek se moze bolje Nini (koja je nakon drugog dana rjesavanja zadataka
otputovala u Vijetnam na Medunarodnu
olimpijadu iz fizike, gdje je osvojila broncanu
medalju) je nedostajao tek bod za broncu, a Adrianu dva boda za srebro. Ipak, sve to
moze se nadomjestiti sljedece godine, kada c e se odrzavati jubilarna 50. IMO u Bremenu
u Njemackoj.
Trojica ucenika ostvarila su maksimalan broj bodova dvojica iz Kine i jedan iz
SAD-a (ali kineskog porijekla!). Oni su na svecanosti zatvaranja dobili pljesak veci cak
i od princa od Asturije, koji je dosao da bi dodjeljivao medalje i svojom prisutnoscu
uvelicao svecanost.
Nakon zatvaranja isli smo na zajednicki banket i druzenje na Univerzitetu Carlos III.
Od raznih ekipa dobili smo sitne poklone za uspomenu, a i mi smo drugima dijelili
olovke. Mnogima je bilo tesko oprostiti se od prijatelja i simpatija koje su stekli na ovoj
olimpijadi, ali sve je proslo u prijateljskoj atmosferi. Bilo je lijepo vratiti se u Hrvatsku
i nadati se jos boljem uspjehu na sljedecoj Olimpijadi.
Adrijan Satja Kurdija i Melkior Ornik

120

Matematicko-fizicki list, LIX 2 (2008. 2009.)

Zadaci
Prvi dan

Spanjolska,
srijeda, 16. srpnja 2008.
1. Tocka H je ortocentar s iljastokutnog trokuta ABC . Kruznica sa sredistem u
polovistu tranice BC prolazi kroz H i sijece pravac BC u A1 i A2 . Slicno, kruznica sa
sredistem u polovistu od CA prolazi kroz H i sijece pravac CA u B1 i B2 ; kruznica sa
sredistem u polovistu od AB prolazi kroz H i sijece pravac AB u C1 i C2 . Dokazi da
sve tocke A1 , A2 , B1 , B2 , C1 , C2 leze na kruznici.
(Rusija)
2. a) Dokazi da vrijedi:
y2
z2
x2
+
+
1
(x 1)2
(y 1)2
(z 1)2
za sve realne brojeve x , y, z, od kojih je svaki razlicit od 1, i zadovoljavaju uvjet
xyz = 1 .
b) Dokazi da jednakost vrijedi za beskonacno mnogo trojki racionalnih rojeva x , y,
z, od kojih je svaki razlicit od 1, i zadovoljavaju uvjet xyz = 1 .
(Austrija)
3. Dokazi da postoji beskonacno mnogoprirodnih brojeva n takvih da je n2 + 1
djeljivo prostim brojem koji je veci od 2n + 2n.
(Litva)

Drugi dan

Spanjolska,
srijeda, 17. srpnja 2008.
- sve funkcije f : (0, ) (0, ) ( f je funkcija sa skupa pozitivnh ealnih
4. Nadi
brojeva u skup pozitivnh realnih brojeva) takvih da vrijedi
w2 + x2
(f (w))2 + (f (x))2
=
,
f (y2 ) + f (z2 )
y2 + z2
za sve pozitivne realne brojeve w, x , y, z, koji zadovljavaju uvjet wx = yz.
(Republika Koreja)
5. Neka su n i k prirodni brojevi takvi da je k n i k n paran broj. Dano je 2n
z arulja oznacenih s 1 , 2 , . . . , 2n i svaka od njih moze biti ili upaljena ili ugasena. Na
pocetku su sve z arulje ugasene. Promatraj nizove koraka: u svakom koraku tocno jedna
z arulja promijeni svoje stanje (ako je bila upaljena, ugasi se, a ako je bila ugasena, upali
se). Neka je N broj takvih nizova od kojih svaki ima k koraka i na kraju su sve z arulje
od 1 do n upaljene,dok su sve z arulje od n + 1 do 2n ugasene. Neka je M broj nizova
od kojih svaki ima k koraka, i na kraju su sve z arulje od 1 do n upaljene,dok su sve
z arulje od n + 1 do 2n ugasene, i nijedna od z arulja n + 1 do 2n nije se niti palila niti
N
.
gasila. Odredi omjer
M
(Francuska)
Matematicko-fizicki list, LIX 2 (2008. 2009.)

121

6. Neka je ABCD konveksan c etverokut takav da je |BA| = |BC|. Oznacimo kruznice


upisane rokutima ABC i ADC s 1 i 2 , tim redom. Pretpostavi da postoji kruznica
koja dodiruje polupravac BA u tocki iza A i polupravac BC u tocki iza C , a isto
tako dodiruje pravce AD i CD. Dokazi da se zajednicke vanjske tangente kruznica 1
i 2 sijeku na kruznici .
(Rusija)
Vrijeme za rad svakog dana: 4 sata i 30 minuta
Svaki zadatak vrijedi 7 bodova
Ranglista
nagrade

broj

I II III poh. bod.


Kina
Rusija
SAD
Juzna Koreja
Iran
Tajland
Sjeverna Koreja
Turska
Tajvan
Madarska
Japan
Vijetnam
Poljska
Bugarska
Ukrajina
Brazil
Peru
Rumunjska
Australija
Njemacka
Srbija
Kanada
Ujedinjeno Kraljevstvo
Italija
Kazahstan
Bjelorusija
Izrael
Hong Kong
Mongolija
Francuska
Indija
Singapur
Nizozemska
Uzbekistan
Litva
Indonezija
Meksiko
Hrvatska
Argentina
ska
Ce
Grcka
Gruzija

Spanjolska
Juznoafricka Republika
Kolumbija
Slovacka
Turkmenistan
Azerbajdzan
Moldavija

122

5
6
4
4
1
2
2
3
2
2
2
2
2
2
2
1
1
1
1
1

1
2
2
5
3
4
1
4
3
3
2
3
1
2
5
3
4
5
2
3
2
4
3
2
3
1
3
2
1
1
2
1
1
1
1
1

1
2

1
2
1
1
2
1
3
2
1
2
2
1
3
4
2
3
3
2
2
1
1
4
5
3
2
4
2
2
1
3
3
1
2
5
3
3
4
3

2
1
2
1
1
1
3
2
4
2
3
4
1
3
4
1
1
1
2

217
199
190
188
181
175
173
170
168
165
163
159
157
154
153
152
141
141
140
139
139
135
134
132
128
125
120
107
105
104
103
98
94
94
92
88
87
86
85
85
85
84
82
79
77
76
76
74
74

nagrade

broj

I II III poh. bod.


Bosna i Hercegovina
Slovenija

Svicarska
1

Svedska
1
Danska
2
Kostarika
Malezija
1
Austrija
Norveska
1
Belgija
1
Makedonija
Luksemburg (5)
Tadikistan
Litva
1
Makau
Maroko
Armenija
Portugal
Albanija
(3)
Cile
1
Irska
Cipar
Novi Zeland
Estonija
Finska
Banglades (4)
Island (5)
Salvador (4)
Lanka
Sri
Trinidad i Tobago
Kirgistan 5
Kuba (1)
1
Ekvador
Kambodza
Crna Gora (3)
Paragvaj (4)
Filipini
Urugvaj (5)
Tunis (4)
Honduras (2)
Gvatemala (4)
Lihtenstajn (2)
Venecuela (2)
Portoriko (3)
Saudijska Arabija
Bolivija (5)
Ujedinjeni Arapski Emirati (4)
Kuvajt (5)

3
2
1
2
1
1
2
2
1
2
1
2
1
1
1
1
1
1
1

1
1

1
1
2
3
1
3
4
4
1
1
2
3
2
1
2
4
1
1
1
2
1
3
1
1
1
3
1
1
1
1
2
1
1
2
1
1

68
68
68
67
66
65
65
63
62
61
61
60
60
58
58
58
56
55
53
49
45
42
42
41
40
33
31
31
29
28
28
27
26
25
24
24
23
22
20
17
16
16
16
9
8
5
5
3

Matematicko-fizicki list, LIX 2 (2008. 2009.)

Druga srednjoeuropska matematicka olimpijada,


ska, 4. 10. rujna 2008.
Olomouc, Ce
Ilko Brnetic, Zagreb
Prosle smo godine s velikim zadovoljstvom docekali radanje
novog regionalnog
natjecanja u kojem sudjelujemo Srednjoeuropske matematicke olimpijade. Ove smo
godine imali priliku potvrditi uspjesan nastavak jedne lijepe price.
Ekipu za natjecanje smo birali u dvije faze i uz dosta zajednickih priprema. Na
Drzavnom natjecanju smo uz odabir s estero ucenika za Medunarodnu
matematicku
olimpijadu, odabrali i s iri krug od dvanaest potencijalnih kandidata za nastup na
Srednjoeuropskoj matematickoj olimpijadi u Olomoucu. Nakon dijela dopisnih i kratkih
zajednickih priprema u dva navrata odrzanih u Zagrebu, odrzano je i izborno dvodnevno
natjecanje. U ekipu su se konacno plasirali: Ivana Antolis (II. razred), Ivo Bozic (II.
razred), Matko Ljulj (I. razred) i Irma Telarovic (III. razred) iz XV. gimnazije u Zagrebu,
Grgur Valentic (I. razred) iz V. gimnazije u Zagrebu i Borna Cicvaric (II. razred)
iz Gimnazije Andrije Mohorovicica u Rijeci. Moramo naglasiti da ostali kandidati
nisu mnogo bodovno zaostali u tom zanimljivom natjecanju, kao i to da sudionici
medunarodne
matematicke olimpijade i maturanti, po pravilu, nisu imali pravo nastupa
na ovom natjecanju. Nakon izbora pripreme su nastavljene do pocetnog tjedna mjeseca
srpnja, a nakon ljetnog odmora, c lanovi ekipe okupili su se u Zagrebu nekoliko dana
prije polaska na put.
Put u Olomouc, jedno od vodecih sredista pokrajine Moravske, vodio nas je preko
Slovenije i Austrije. Zahvaljujuci nenadanim popravcima na pruzi izmedju Maribora i
Graza, umjesto predvidena
dva presjedanja, imali smo ih c ak pet, no, nakon 11 sati
voznje stigli smo na odrediste. Nakon toga slijedilo je pet i pol dana intenzivnog rada,
natjecanja, druzenja,. . .
Natjecanje i smjestaj bili su organizirani u Sveucilistu Jan Palacky. Ove godine
natjecalo se svih devet drzava koje su dogovorom i inicirale ovo natjecanje. Upravo je
sudjelovanje Madarske
i Njemacke, koje prosle godine nisu sudjelovale u natjecanju,
pridonijelo znacajnom jacanju kvalitete natjecanja koje se sastoji od dva dijela:
pojedinacnog i ekipnog.
U pojedinacnom dijelu sudjelovalo je 52 natjecatelja, po 6 iz svake drzave sudionice,
samo iz Slovenije ih je sudjelovalo c etvero. Nasi su ucenici osvojili tri broncane medalje
(Ivo Bozic, Matko Ljulj i Grgur Valentic) i jednu pohvalu (Borna Cicvaric). Ivo Bozic
bio je nadomak srebrne medalje, nedostajao mu je tek bod, a Borna Cicvaric je jedini
od nasih ucenika rijesio najtezi, geometrijski zadatak. U ekipnom dijelu natjecanja nasi
su ucenici bili tek osmi, no uz neveliki zaostatak za bolje plasiranim ekipama.
Rezultatski gledano, najuspjesnije su bile tri objektivno najjace ekipe, Madarska,
Poljska i Njemacka, dok su sve ostale, izuzev Slovenije, imale vrlo slicne rezultate. Mi
- ekipu na natjecanju.
smo s time vrlo zadovoljni, jer smo imali najmladu
skoj; domacini su priredili bogat program izleta,
Proveli smo vrlo ugodne dane u Ce
tako da smo uz upoznavanja vrlo lijepog Olomouca, grada bogate povijesti i kulture,
upoznali Kromeriz, grad sa spomenicima pod zastitom UNESCO-a, poznat ponajprije
po dvorcu i lijepim vrtovima, lijepu s pilju Javoricko, te dvorac Bouzov.
Sljedece godine, natjecanje c e se odrzati u Poljskoj.
Matematicko-fizicki list, LIX 2 (2008. 2009.)

123

Na kraju navodimo i zadatke s natjecanja (uz napomenu da je vrijeme rjesavanja


svakog dana bilo 5 sati), kao i rezultate.

Zadaci s pojedinacnog natjecanja


1. Neka je (an )
n=1 niz prirodnih brojeva takav da je an < an+1 , n  1 . Za
svaku c etvorku indeksa (i, j, k, m), gdje je 1 i < j k < m i i + m = j + k , vrijedi
nejednakost ai + am > aj + ak . Odredi najmanju mogucu vrijednost broja a2008 .
2. Promatramo s ahovsku plocu dimenzija n n , gdje je n > 1 prirodan broj. Na
koliko nacina se moze postaviti 2n 2 jednakih kamencica na tu s ahovsku plocu (na
razlicita polja) tako da nikoja dva od njih ne leze na istoj dijagonali sahovske ploce?
(Dva su kamencica na istoj dijagonali sahovske ploce ako su sredista tih polja paralelna
jednoj od dijagonala n n kvadrata.)
3. Neka je ABC jednakokracan trokut u kojem je |AC| = |BC|. Njegova upisana
kruznica dodiruje AB i BC u tockama D i E redom. Pravac (razlicit od AE ) prolazi
kroz A i sijece upisanu kruznicu u F i G. Pravac AB sijece pravce EF i EG redom u
K i L . Dokazi da je |DK| = |DL|.
4. Odredi sve cijele brojeve k takve da su za sve cjelobrojne n , brojevi 4n + 1 i
kn + 1 relativno prosti.

Zadaci s ekipnog natjecanja


- sve funkcije f : R R takve da vrijedi
1. Nadite
xf (x + xy) = xf (x) + f (x2 )f (y)
za sve realne brojeve x i y.
2. Na ploci je napisano n 2 prirodnih brojeva. U svakom koraku biramo par
brojeva s ploce i zamijenimo svaki od njih njihovom sumom. Odredite sve vrijednosti
n za koje mozemo, nakon konacnog broja koraka, dobiti n -torku s jednakim brojevima.
3. Dan je s iljastokutni trokut ABC . Neka je E tocka sa suprotne strane pravca AC
u odnosu na tocku B, i neka je D tocka unutar duzine AE . Pretpostavimo da vrijedi
<
)ADB = <
)CDE , <
)BAD = <
)ECD i <
)ACB = <
)EBA. Dokazite da tocke B, C i E
leze na istom pravcu.
4. Neka je n prirodan broj. Ako je suma svih pozitivnih djelitelja broja n potencija
- potencija broja 2 . Dokazite!
broja 2 , onda je i broj tih djelitelja takoder

124

Matematicko-fizicki list, LIX 2 (2008. 2009.)

Rezultati pojedinacnog i ekipnog natjecanja

drzava
Madarska
Poljska
Njemacka
ska
Ce
Hrvatska

Svicarska
Austrija
Slovacka
Slovenija

zlatna
medalja
3
1
1
0
0
0
0
0
0

srebrna
medalja
3
4
3
1
0
0
0
0
0

broncana
medalja
0
1
1
1
3
3
3
2
0

poredak
1.
1.
1.
4.
5.
6.
7.
8.
9.

drzava
Poljska
Madarska
Njemacka
Austrija
Slovacka

Svicarska
ska
Ce
Hrvatska
Slovenija

broj
bodova
32
32
32
26
25
24
22
21
9



Matematicko-fizicki list, LIX 2 (2008. 2009.)

125

Medunarodno
matematicko natjecanje
Klokan bez granica 2008. g.
Medunarodno
matematicko natjecanje Klokan bez granica odrz alo se deseti put pod pokroviteljstvom Ministarstva prosvjete i
s porta i Hrvatskog matematickog drustva 20. travnja ove godine. S priblizno istim zadacima natjecalo se 5 000 000 ucenika
u 42 zemlje svijeta, s to ovo natjecanje c ini najvecim s kolskim
natjecanjem u svijetu. Prema odjecima koji su stigli do nas,
vjerujemo da je natjecanje postiglo svoju svrhu i zainteresiralo
ucenike za rjesavanje zadataka iz matematike. U Hrvatskoj su se
natjecali ucenici u 320 osnovnih i 120 srednjih skola u svim z upanija u s est kategorija:
Leptirici, Ecolier, Benjamin, Cadet, Junior, Student. Ukupno se natjecalo 29 806
ucenika.
Sudjelovalo je 6743 ucenika II. i III. razreda osnovne s kole (L), 8365 ucenika IV. i
V. razreda osnovne s kole (E), 6166 ucenika VI. i VII. razreda osnovne s kole (B), 4526
ucenika VIII. razreda osnovne i I. razreda srednje s kole (C). 2935 ucenika II. i III.
razreda srednje s kole (J) i 1071 ucenik IV. razreda srednje s kole (S).
Najveci broj bodova koji se mogao ostvariti bio je 72 u kategoriji Leptirici, te 120
bodova za sve ostale kategorije. Prilikom dolaska na natjecanje svaki je ucenik dobio
poklon, a 10% najboljih natjecatelja dobili su nagrade. Bodovni prag najboljih 10%
natjecatelja bio je: za Leptirice 50 bodova; Ecolier 87.50 bodova; Benjamin
77.50 bod; Cadet 55.00 bodova; Junior 60.25 bodova; Junior matematicki
program 75.00 bodova; Student 48.75 bodova; Student matematicki program 66.25
bodova.
Sljedeci zadaci mogu vas upoznati s proslogodisnjem natjecanjem i korisno posluziti
za pripreme za novo natjecanje koje c e se odrzati 19. ozujka 2009. godine.
Koordinator natjecanja Klokan bez granica, Neda Lukac, prof.

Zadaci za ucenike 8. razreda osnovne skole i 1. razreda srednje skole (Cadet)


Pitanja za 3 boda:
1. U gusarskoj s koli, svaki ucenik mora sasiti svoju crno-bijelu zastavu. Pri tome
mora biti ispunjen uvjet da crni dio cini tri petine zastave. Koliko od prikazanih zastava
ispunjava taj uvjet?

A. nijedna

B. jedna

C. dvije

D. tri

E. cetiri

2. U razrednom odjeljenju ima 9 djecaka i 13 djevojcica. Polovina djece u odjeljenju


ima prehladu. Koliko djevojcica najmanje ima prehladu?
A. 0
B. 1
C. 2
D. 3
E. 4

126

Matematicko-fizicki list, LIX 2 (2008. 2009.)

3. 6 klokana pojedu 6 vreca sijena u 6 minuta. Koliko klokana c e pojesti 100 vreca
sijena za 100 minuta?
A. 100

B. 60

C. 6

D. 10

E. 600

4. Brojeve 2, 3, 4 i jos jedan nepoznati prirodni broj treba upisati


u kvadratice na slici. Zbroj brojeva u prvom retku mora biti 9, a zbroj
brojeva u drugom retku 6. Nepoznati broj je
A. 5

B. 6

C. 7

D. 8

E. 4

5. Trokut i kvadrat na slici imaju jednake opsege. Koliki je


opseg cijelog lika (peterokuta)?
A. 12 cm
B. 24 cm
C. 28 cm
E. ovisno o duljinama stranica trokuta

D. 32 cm
4cm

6. Cvjecarka Rina ima 24 bijele, 42 crvene i 36 z utih ruza. Koliko najvise jednakih
kitica ona moze sloziti ako z eli upotrijebiti sve ruze?
A. 4

B. 6

C. 8

D. 10

E. 12

7. Koliko se kvadrata moze nacrtati spajanjem tocaka na slici?


A. 2

B. 3

C. 4

D. 5

E. 6

8. Tri pravca prolaze istom tockom. Velicine dvaju


kutova su 108 i 124 , kao s to se vidi na slici. Koliko
stupnjeva ima kut osjencan sivom bojom?
A. 52

B. 53

C. 54

D. 55

E. 56

108q
124q

Pitanja za 4 boda:
9. Na kocki su odsijeceni vrhovi, kao na slici. Koliko bridova
ima nova figura na slici?
A. 26

B. 30

C. 36

D. 40

E. neki drugi odgovor

10. Danijel ima 9 novcica (svaki vrijednosti 2 lipe), a njegova sestra Ana 8 novcica
vrijednosti 5 lipa svaki. Koliko najmanje novcica trebaju razmijeniti medusobno
Danijel
i Ana da bi imali jednake novcane iznose?
A. 4

B. 5

C. 8

D. 12

E. nije moguca takva razmjena

11. Tom i Jerry razrezali su 2 sukladna pravokutnika. Tom je dobio 2 pravokutnika,


svaki opsega 40 cm, a Jerry 2 pravokutnika, svaki opsega 50 cm. Koliki je bio opseg
pocetnih pravokutnika?
A. 40 cm

B. 50 cm

C. 60 cm

Matematicko-fizicki list, LIX 2 (2008. 2009.)

D. 80 cm

E. 90 cm

127

12. Jedna strana kocke razrezana je po njenim dijagonalama (kao na


slici). Koja od sljedecih mreza nije mreza kocke na slici?
1

A. 1 i 3

B. 1 i 5

C. 3 i 4

D. 3 i 5

E. 2 i 4

13. Na pravcu su istaknute tocke A, B, C i D. Poznate su sljedece duljine:


|AB| = 13 , |BC| = 11 , |CD| = 14 i |DA| = 12 . Kolika je udaljenost dviju najudaljenijih tocaka?
A. 14

B. 38

C. 50

D. 25

E. drugi odgovor
Q

14. U pravokutnik su upisane 4 kruznice


duljine polumjera 6 cm, kao na slici. Tocka P je
vrh pravokutnika, a tocke Q i R diralista kruznica
i stranica pravokutnika. Kolika je povrsina trokuta
PQR?
B. 45 cm2
C. 54 cm2
A. 27 cm2

D. 108 cm2

E. 180 cm2

15. U kutiji se nalazi 7 karata. Na kartama su napisani brojevi od 1 do 7 (na svakoj


karti tocno jedan broj). Matija uzima iz kutije nasumce 3 karte, a Luka 2 karte (2 su
karte ostale u kutiji ). Tada Matija kaze Luki: Znam da je zbroj brojeva na tvojim
kartama paran. Zbroj brojeva Matijinih karata iznosi:
A. 10

B. 12

C. 6

D. 9

E. 15

16. Francuski matematicar August de Morgan imao je x godina u godini x2 . Umro


je 1899. godine. Kada se rodio de Morgan?
A. 1806.

B. 1848.

C. 1849.

D. 1899.

E. drugi odgovor

Pitanja za 5 bodova:
17. Simetrala kuta <
) C uz osnovicu |BC| jednakokracnog trokuta ABC sijece krak
AB u tocki D. Ako je |BC| = |CD|, kolika je velicina kuta <
)CDA?
A. 90

B. 100

C. 108

D. 120

E. nemoguce je odrediti
3

18. Drvena kocka 11 11 11 nastala je ljepljenjem 11 jedinicnih kocaka. Koliko


se najvise jedinicnih kocaka moze vidjeti gledajuci iz iste tocke gledanja?
A. 328

B. 329

C. 330

D. 331

E. 332

19. U Malim astronomima djevojke c ine vise od 45%, a manje od 50%. Koji je
najmanji moguci broj djevojaka u toj grupi?
A. 3

128

B. 4

C. 5

D. 6

E. 7

Matematicko-fizicki list, LIX 2 (2008. 2009.)

20. Djecak uvijek govori istinu c etvrtkom i petkom, uvijek laze utorkom, a ostale
dane u tjednu govori istinu ili laze bez pravila. Pitali su ga, sedam dana uzastopno, za
njegovo ime i odgovori prvih 6 dana bili su: Ivan, Branko, Ivan, Branko, Petar, Branko.
je djecak odgovorio sedmog dana?
Sto
A. Ivan

B. Branko

C. Petar

D. Katarina

E. drugi odgovor

21. Martina i Ivica su krenuli planinariti. U selu, u podnozju planine, procitali su


oznaku na kojoj pise da do vrha ima 2 sata i 55 minuta (pjesacenja). Napustili su
selo u 12 sati. U 13 sati stali su radi kratkog odmora i procitali novu oznaku na kojoj
pise da do vrha ima samo 1 sat i 15 minuta. Nakon 15 minuta nastavili su put istom brzinom kao i prije i nisu se vise zaustavljali do vrha. U koje su vrijeme stigli na vrh?
A. 14 : 30

B. 14 : 00

C. 14 : 55

D. 15 : 10

E. 15 : 20

22. Nazovimo tri prosta broja specijalnim ako je njihov umnozak 5 puta veci od
njihovog zbroja. Koliko specijalnih trojki prostih brojeva postoji?
A. 0

B. 1

C. 2

D. 4

E. 6

23. Zadana su dva skupa A i B peteroznamenkastih prirodnih brojeva. U skupu A


su brojevi ciji je umnozak svih znamenki 25, a u skupu B brojevi ciji je umnozak svih
znamenki 15. Koji skup ima vise brojeva? Koliko puta vise?
A. skup A,

5
puta
3
D. skup B, 2 puta

B. skup A, 2 puta

C. skup B,

E. oba skupa imaju jednaki broj clanova

5
puta
3

24. Najveci zajednicki djelitelj dvaju prirodnih brojeva m i n je 12, a njihov najmanji
- 5 brojeva: n , m , n , m , m n koliko njih
zajednicki visekratnik je kvadrat. Izmedu
3 3 4 4
su kvadrati?
A. 1

B. 2

C. 3

D. 4

E. nemoguce je odrediti

Zadaci za ucenike 2. i 3. razreda srednje skole (Junior)


Pitanja za 3 boda:
1. Dano je 5 kutija, a u svakoj od njih nalaze se kartice sa slovima A, E , I , O, U
kao s to je prikazano na slici. Petar je izbacio neke kartice tako da je na kraju svaka
kutija sadrzavala jednu karticu, a razlicite kutije sadrzavale su razlicita slova. Koja je
kartica ostala u kutiji broj 2?

A. A

B. E

C. I

Matematicko-fizicki list, LIX 2 (2008. 2009.)

D. O

E. U

129

2. Franjo i Gabrijela natjecali su se u trcanju na 200 metara. Gabrijela je stazu


pretrcala za pola minute, a Franjo za jednu stotinu sata. Tko je i za koliko sekundi bio
brzi?
A. Gabrijela za 36 sekundi
B. Franjo za 24 sekunde
C. Gabrijela za 6 sekundi
D. Franjo za 4 sekunde
E. Imaju ista vremena
3. Na proslavi ovogodisnje Nove godine, Borna je obukao majicu s brojem godine i
pred ogledalom izveo je stoj na rukama. Koju je sliku u ogledalu vidio njegov prijatelj
Vinko stojeci iza Borne, normalno na nogama?
A.

B.

C.

D.

E.

4. Zadani su brojevi a = 2 (4), b = (2) (3), c = 2 8 , d = 0 (6) i


e = (12) : (2). Koliko od njih nije jednako broju 6?
A. 0

B. 1

C. 2

D. 4

E. 5

5. Kolika je duljina duzine AB, ako je duljina


stranice svakog od kvadrata na slici 1 metar?

A. 5
B. 13
C. 5 + 2

D. 5
E. nijedna od danih

A
6. Koliko se najmanje slova moze maknuti iz rijeci KANGOUROU tako da su
preostala slova abecedno poslagana?
A. 1

B. 2

C. 3

D. 4

E. 5

7. U kutiji je 7 karata. Brojevi od 1 do 7 napisani su na njima i to tocno jedan broj


na jednoj karti. Prvi igrac iz kutije na slucajan nacin odabere 3 karte, a drugi igrac uzme
2 karte. Tada prvi igrac kaze drugome: Znam da je zbroj brojeva na tvojim kartama
paran. Koliki je zbroj brojeva na kartama prvog igraca? (Igraci ne vide preostale karte
u kutiji.)
A. 10

B. 12

C. 6

D. 9

E. 15

8. Tom i Jerry dobili su jednake papire u obliku pravokutnika. Svaki od njih je svoj papir razrezao na dva jednaka pravokutnika. Tom je izrezao pravokutnike s opsegom 40 cm,
a Jerry dva pravokutnika s opsegom 50 cm. Koliki je bio opseg pocetnog komada papira?
A. 40 cm

B. 50 cm

C. 60 cm

D. 80 cm

E. 100 cm

Pitanja za 4 boda:
9. Na prvom testu Ana je osvojila 1 bod od mogucih 5. Na sljedecim testovima
osvojila je sve bodove od mogucih 5. Koliko je jos testova trebala pisati da joj prosjecan
broj bodova bude 4 po testu?
A. 2

130

B. 3

C. 4

D. 5

E. 6

Matematicko-fizicki list, LIX 2 (2008. 2009.)

10. Marko ima 10 kartica, a na svakoj od njih se nalazi tocno jedan od ovih brojeva:
3, 8, 13, 18, 23, 28, 33, 48, 53, 68. Koliko najmanje kartica Marko moze izabrati tako
da zbroj brojeva na njima bude jednak 100?
A. 2

B. 3

C. 4
D. 5
E. to je nemoguce uciniti
11. Sedam je patuljaka rodeno istog dana u 7 uzastopnih godina. Tri najmlada
zajedno imaju 42 godine. Koliko godina zajedno imaju tri najstarija patuljka?
A. 51

B. 54

C. 57

D. 60

E. 63

12. Dva pravilna s esterokuta na slici su jednaka. Kolika je povrsina osjencanog dijela paralelograma?
A.

1
2

B.

1
3

C.

1
4

D.

1
5

E.

1
6

prirodnih brojeva oznaceno je slovima na brojevnom pravcu. Poznato je da su


13. Sest
barem dva od njih djeljivi s 3 i barem dva od njih djeljivi s 5. Koji su brojevi djeljivi s 15?

A
A. A i F

B. B i D

E
C. C i E

D. svi

F
E. samo jedan od njih

14. Koliko najvise znamenaka treba obrisati iz 1000-znamenkastog broja


20082008. . . 2008 tako da zbroj preostalih znamenaka bude 2008?
A. 746

B. 510

C. 524

D. 1020

E. 130

15. Na slici je dan jednakokracan trokut u kojem je


|AB| = |AC|. Ako je PQ okomito na AB, kut BPC je 120 i
kut ABP je 50 , koliki je kut PBC ?
A. 5

B. 10

C. 15

D. 20

E. 25

16. Koliko postoji parova realnih brojeva c iji su zbroj, produkt


i kvocijent medusobno
jednaki?
A. nema takvih parova

B. 1 par

C. 2 para

D. 4 para

E. 8 parova

Pitanja za 5 bodova:
17. Svaka znamenka pocevsi od trece (brojano slijeva) u s esteroznamenkastom
broju jednaka je zbroju prethodne dvije znamenke (koje se nalaze slijeva). Koliko
s esteroznamenkastih brojeva ima to svojstvo?
A. nijedan

B. 1

C. 2

D. 4

E. 6

18. Imam drvenu kocku cije tri strane su plave, a tri crvene. Kad je prerezem u
3 3 3 = 27 jednakih malih kocaka, koliko od njih ima barem dvije razlicitim bojama
obojane strane
A. 6
B. 12
C. 14
E. ovisi o tome koje su strane velike kocke bile obojane plavo i crveno
Matematicko-fizicki list, LIX 2 (2008. 2009.)

D. 16

131

19. Oznacimo s n! = 1 2 3 . . . (n 1) n . Ako je n! = 215 36 53 72 11 13 ,


tada je n jednako
A. 13

B. 14

C. 15

D. 16

E. 17

20. Kolika je duljina istaknutog (veceg) luka na kruznici


polumjera 1?
5
3
2

5
B.
C.
D.
E.
4
3
2
2
3
21. Kvadrat 4 4 podijeljen je na 16 jedinicnih kvad- najveci broj dijagonala koje se mogu povuci u
ratica. Nadi
tim kvadraticima tako da nikoje dvije nemaju zajednicke tocke.
A.

A. 8

B. 9

C. 10

D. 11

E. 12

skociti tocno 10 metara. Na koliko


22. Kanga uvijek skace po 1 m ili po 3 m. Zeli
nacina moze Kanga izvesti to skakanje? (Skokove 1 + 3 + 3 + 3 i 3 + 3 + 3 + 1 smatramo
razlicitim nacinima.)
A. 28

B. 34

C. 35

D. 55

E. 56

23. Kvadrat ABCD na slici ima stranicu duljine 1, i lukovima su spojeni odgovarajuci nasuprotni vrhovi kvadrata.
Kolika je udaljenost tocaka P i Q?

3
3
C. 5 2
E. 3 1
B.
D.
A. 2 2
4
2
24. Koliko postoji 2007-teroznamenkastih brojeva s ovim svojstvom: svaki dvoznamenkasti broj sacinjen od dvije uzastopne znamenke pocetnog broja je djeljiv ili sa 17
ili s 23?
A. 5

B. 6

C. 7

D. 9

E. vise od 9

Zadaci za za ucenike 4. razreda srednje skole (Student)


Pitanja za 3 boda:
1. Brojevi 3, 4 i jos dva nepoznata broja upisani su u polja tablice 2 2. Poznato je
da je zbroj brojeva po retcima jednak 5 i 10, a zbroj po jednom stupcu je 9. Veci od ta
dva nepoznata broja je
A. 5

B. 6

2. Ako je x + y = 0 i x = 0 , tada je
A. 1

132

B. 0

C. 7

D. 8

E. 3

2008

x
=
y2008
C. 1

D. 22008

E.

x
y

Matematicko-fizicki list, LIX 2 (2008. 2009.)

3. Matrica se sastoji od 21 stupca oznacenih brojevima 1,2,. . . ,21 i 33 retka numeriranih brojevima 1,2,. . . ,33. Obrisemo one retke c iji redni broj nije visekratnik od 3 i
sve one stupce ciji redni broj je paran. Koliko je u matrici ostalo polja?
A. 110

B. 121

C. 115.5

D. 119

E. 242

4. Koliko prostih brojeva p ima svojstvo da je i broj p + 1 prost broj?


A. nijedan

B. 1

C. 2

D. 3

E. beskonacno

5. Promatramo rijeku od tocke


2
1
A. U svom toku racva se na dva
8
3
2
A
rukavca. Prvi rukavac preuzima
3
5
kolicine vode, a drugi ostatak. Nakon
8
toga se prvi rukavac opet racva u tri, 1
vode,
pri c emu jedan novi rukavac preuzima
B
8
5
drugi , a treci ostatak. Nakon nekog vremena, taj treci se
8
rukavac spaja s onim rukavcem rijeke nastalim pri prvom racvanju. Mapa rijeke
prikazana je na slici. Kolika kolicina vode dolazi u mjesto B?
1
5
2
1
1
B.
C.
D.
E.
3
4
9
2
4
6. U kutiji je 7 karata. Brojevi od 1 do 7 napisani su na njima i to tocno jedan broj
na jednoj karti. Prvi igrac iz kutije na slucajan nacin odabere 3 karte, a drugi igrac uzme
2 karte. Tada prvi igrac kaze drugome: Znam da je zbroj brojeva na tvojim kartama
paran. Koliki je zbroj brojeva na kartama prvog igraca?

A.

A. 10

B. 12

C. 6

D. 9

E. 15

7. Dan je jednakokracan trokut ABC (|CA| = |CB|).


Na osnovici je oznacena tocka D tako da je |AD| = |AC|
i |DB| = |DC|. Koliki je kut ACB?
A. 98

B. 100

C. 104

D. 108

E. 110

8. Najveca vrijednost funkcije f (x) = |5 sin x 3| u skupu R iznosi:


A. 2

C.

B. 3

D. 5

E. 8

Pitanja za 4 boda:
9. Svaka od kocaka na slici ima bridove duljine 1. Kolika je
udaljenost od tocke A do tocke B?

A. 17
B. 7
C. 13
D. 7
E. 14
10. Matilda je nacrtala 36 klokana koristeci tri razlicite boje. 25 klokana ima nesto
- a za 20 klokana je koristila crnu boju. Samo 5
obojano z utom, 28 ih ima nesto smede,
od njih ima na sebi sve tri boje. Koliko klokana imaju na sebi samo jednu boju?
A. 0

B. 4

C. 12

Matematicko-fizicki list, LIX 2 (2008. 2009.)

D. 31

E. nemoguce je odrediti

133

11. Tri se kruga diraju u parovima kao na slici.


Polumjer svakoga je r . Kolika je povrsina podrucja A
omedenog
s ta tri kruga?



1
2
3 2
r

r
3
B.
C. r2
A.
2
2
2
8



3 2
3

D.
3
r2
E.
r
2
3
2
12. Dva pravilna s esterokuta na slici su jednaka. Kolika je povrsina osjencanog dijela paralelograma?

1
1
1
1
1
B.
C.
D.
E.
2
3
4
5
6
13. Na natjecanju Matematicki Kup zadano je 5 problema koji svi nose razliciti broj
bodova. Damjan je rijesio svih pet i pri tome je osvojio svih 10 bodova koje je bilo
moguce osvojiti na dva problema s najmanjim brojem bodova. Osim toga, za dva problema s najvecim brojem bodova dobio je svih 18 bodova. Koliko je bodova osvojio Damjan?

A.

A. 30

B. 32

C. 34

D. 35

E. 40

14. Brojnik i nazivnik razlomka su negativni cijeli brojevi. Brojnik je veci od


nazivnika. Koja od sljedecih tvrdnji o tom razlomu je istinita?
- 1 i 0.
B. Razlomak je izmedu
D. Razlomak je veci od 1.

A. Razlomak je manji od 1 .
C. Razlomak je pozitivan broj manji od 1.
E. Ne moze mu se odrediti predznak.

15. Tri su tocke izabrane na slucajan nacin iz skupa tocaka na


slici. Kolika je vjerojatnost da su te tocke kolinearne?
A.

1
12

B.

1
11

C.

1
16

D.

1
8

E.

3
12

16. Ako je x2 yz3 = 73 i xy2 = 79 , tada je xyz =


A. 74

B. 76

C. 78

D. 79

E. 710

Pitanja za 5 bodova:
17. Duljine stranica kvadra u centrimetrima su prirodni brojevi koji c ine geometrijski
niz s kvocijentom 2. Koji od brojeva mogu biti obujam tog kvadra?
A. 120 cm2

B. 188 cm2

C. 216 cm2

D. 350 cm2

18. U ovom racunu svaka zvjezdica oznacava jednu znamenku.


Koliki je zbroj svih znamenaka u rezultatu ovog mnozenja?
A. 16

B. 20
C. 26
D. 30
E. neki drugi odgovor
- vrijednost izraza x2 + y2 + z2 , ako je x + y + z = 1 i
19. Nadi
1 1 1
+ + = 0.
x
y
z
A. 0
B. 1
C. 2
D. 3
E. nemoguce je odrediti

134

E. 500 cm2

* * *
1**
22**
+ 90*
**2
56***

Matematicko-fizicki list, LIX 2 (2008. 2009.)

20. Kruznica je upisana trokutu ABC , pri cemu je |AC| = 5 ,


|AB| = 6 i |BC| = 3 . Duzina ED dodiruje kruznicu. Koliki je
opseg trokuta ADE ?
A. 7

B. 4

C. 9

D. 6

E. 8

21. Niz je zadan ovako: an+1 = an + (1)n n , za n  1 , i


a1 = 0 . Ako je ak = 2008 , koliki je k ?
A. 2008

B. 2009

C. 4017

D. 4018

E. neki drugi

22. Broj 332 1 ima tocno dva djelitelja koji su veci od 75 i manji od 85. Koliki je
produkt tih djelitelja?
A. 5852

B. 6560

C. 6804

D. 6888

E. 6972

23. Kvadrat ABCD ima stranice duljine 1. Ako je tocka M


poloviste stranice AB, kolika je povrsina osjencanog podrucja?
A.

1
24

B.

1
16

C.

1
8

D.

1
12

E.

2
13

24. Ako je sin x + cos x = m, tada je sin4 x + cos4 x =


A. 1

(1 m2 )2
2

B. 1 +

(1 m2 )2
2

C.

1 (1 m2 )2
2

D. m4

E. m4 + 1

Rjesenja

Cadet
1.
9.
17.

C
C
C

2.
10.
18.

C
B
D

3.
11.
19.

C
C
C

4.
12.
20.

1.
9.
17.

D
B
D

2.
10.
18.

C
D
E

3.
11.
19.

B
B
D

4.
12.
20.

1.
9.
17.

B
A
C

2.
10.
18.

C
B
A

3.
11.
19.

B
A
B

4.
12.
20.

D
D
A

5.
13.
21.

D
D
B

6.
14.
22.

B
D
B

7.
15.
23.

C
B
B

8.
16.
24.

A
A
B

B
A
A

6.
14.
22.

D
A
A

7.
15.
23.

B
A
E

8.
16.
24.

C
B
A

D
D
C

6.
14.
22.

B
C
B

7.
15.
23.

D
B
D

8.
16.
24.

E
A
A

Junior
B
A
D

5.
13.
21.

Student
B
A
E

5.
13.
21.


Obavijesti o ovom natjecanju mogu se dobiti na internetskoj stranici
http://www.math.hr/hmd.

Matematicko-fizicki list, LIX 2 (2008. 2009.)

135


Zarko
Dadic, Egzaktne znanosti u Hrvata u poslijeprosvjetiteljskom razdoblju
(1789.1835.), edicija: Posebna izdanja, urednik: Milan Miric, Naklada Ljevak, Zagreb,
2007., 326 str.

Najnovijom knjigom, o kojoj je ovdje rijec, akademik Zarko


Dadic obradovao je sve one koji su ocekivali i napokon docekali da
se objelodane rezultati njegovog dugotrajnog ustrajnog i predanog
znanstveno-istrazivackog rada. U knjizi se tako objavljuju izvorna
autorova istrazivanja koja je on obavljao tijekom vise od dvadeset
godina. Ova knjiga se nadovezuje na knjigu Egzaktne znanosti
u Hrvata u doba prosvjetiteljstva koju je godine 2004. objavila
Matica hrvatska a ujedno je jedna od niza knjiga u kojima autor
- cjelokupnu povijest egzaktnih znanosti u Hrvata. Razdoblje
obraduje
od Francuske revolucije do hrvatskog narodnog preporoda, koje je
obradeno
u ovoj knjizi, u pogledu egzaktnih znanosti u Hrvata bilo
je dosad najmanje poznato. Da bi uzitak pri citanju bio s to veci
pobrinuli su se, uz autora i urednika, recenzenti dr. sc. Mijo Korade i dr. sc. Trpimir
Macan. Uz skraceni upadljivi naslov na omotu korica knjige otisnut je fotografski
prikaz u boji armilarnih sfera iz zbirke obitelji Garanjanin u Trogiru. Sfere se cuvaju u
Gradskom muzeju u Trogiru.
U predgovoru knjige autor je opisao dogadanje
u filozofiji, znanosti, knjizevnosti i
kulturi nakon Francuske revolucije 1789. pa do 1830.1840., u Francuskoj, Njemackoj
i Habsburskoj monarhiji. To razdoblje moglo se s jednog motrista oznaciti kao
postjozefinsko, a drugoga kao pretpreporodno, uz napomenu da hrvatski narodni
preporod pocinje 1835. godine. Autor se stoga odlucio da to razdoblje oznaci kao
poslijeprosvjetiteljsko buduci da je najveci dio knjige posvetio prirodnofilozofskim i
prirodoznanstvenim zbivanjima s poslijeprosvjetiteljskog motrista. Jos jedan razlog za to
je i c injenica da se ta knjiga nadovezuje na spomenutu knjigu iz 2004., i u odredenom
smislu s njom cini cjelinu.
Autor je dao detaljan uvod u tematiku, naime prikazao je osobe vazne za razvoj
prirodnofilozofskih i prirodnoznanstvenih shvacanja u poslijeprosvjetiteljskom dobu, a
koje su djelovale u Francuskoj i Njemackoj. Isto tako naglasena je njihova uloga i

u kojima su te osobe djelovale utemeljile su novi


obrazovnom sustavu (skole!). Skole
model razvitka znanstvenih istrazivanja. Autor je naglasio i utjecaj tih s kola na visoke
s kole u Habsburskoj monarhiji s to je znacajno za hrvatske znanstvenike koji su djelovali
u zemljama monarhije, a izvan Hrvatske.
U vecem dijelu knjige su analizirana gotovo sva djela hrvatskih znanstvenika koji
su djelovali na sveucilistu u Budimu, odnosno Pesti, te na akademijama u Madarskoj
i
Slovackoj u tom razdoblju. Knjiga pokazuje da je na Filozofskom fakultetu u Budimu,
odnosno u Pesti mnogo Hrvata u tom razdoblju predavalo matematiku i fiziku, a da ih
- ostalih, na Filozofskom
je mnogo bilo i na Zvjezdarnici u Budimu. Tako su, izmedu
fakultetu predavali Ivan Paskvic, Franjo Josip Domin i Franjo Bruna, a u jednom
razdoblju su na zvjezdarnici djelovali Mirko Danijel Bogdanic, Franjo Bruna i Ivan
Paskvic. Osim toga je autor vrlo iscrpno prikazao rad Ignjata Martinovica u matematici
i fizici, koji je bio najprije u franjevackom redu, a kad je izasao iz njega bio je profesor
u Lavovu. Autor temeljito ispravlja neispravne tvrdnje o etnickoj pripadnosti Ignjata

136

Matematicko-fizicki list, LIX 2 (2008. 2009.)

Martinovica za kojega dokazuje da je hrvatskog podrijetla. Autor je pronasao mnoge


dosad nepoznate rukopise spomenutih znanstvenika, posebno u Sveucilisnoj knjiznici
u Budimpesti i u knjiznici zvjezdarnice Madarske
akademije. Na temelju tiskane i
- prikazan je cjelokupni rad spomenutih hrvatskih znanstvenika u tom
rukopisne grade
razdoblju te je pokazana geneza njihova doprinosa. Na temelju njihove korespondencije
su prikazani njihovi kontakti sa znanstvenicima u velikim znanstvenim sredistima u
Europi. Osobito je na temelju izvora po prvi put opsezno razjasnjem sukob hrvatskog
astronoma Paskvica koji je pocetkom 19. stoljeca bio ravnatelj zvjezdarnice u Budimu s
njegovim asistentom Danielom Kmethom, pri cemu su najveci astronomi toga vremena,
primjerice Gauss, dokazali da je Paskvica neosnovano napao Kmeth i da ga je htio
znanstveno diskreditirati. Koristeni su gotovo svi astronomski c asopisi u kojima su se
objavljivali rezultati motrenja hrvatskih astronoma na zvjezdarnici u Budimu u to doba.
U knjizi je obradena
i znanstvena situacija u Hrvatskoj u to doba, posebno na Kraljevskoj
akademiji u Zagrebu. Obradeni
su rukopisi sacuvani u franjevackim knjiznicama te je
analizirano s kolstvo u hrvatskim krajevima. Posebna je pozornost posvecena promjeni
programa nastave u s kolama u Dalmaciji nakon s to su je okupirali Francuzi, a onda i
nakon Austrijske okupacije kad je s kolstvo u Dalmaciji ujednaceno s programima koje
je vrijedilo za sjevernu Hrvatsku.
U pogovoru naglasava se znacenje 1835. godine na svjetskoj pozornici znanstvenih
zbivanja kada prirodne znanosti odbacuju metafiziku, a unutar njih pojavljuju se
specijalnosti. Autor je, uz to, istaknuo da i u matematici od tog vremena pocinje jedno
sasvim novo razdoblje. Godina 1835. znacajna je i za Hrvatsku, jer se usporedno
s politickim djelovanjem pojavljuju i nastojanja za uspostavu nacionalnih kulturnih
i znanstvenih ustanova. Od tada hrvatski znanstvenici uglavnom djeluju u domovini,
osnivaju domace znanstvene ustanove i drustva, i zapocinju sustavni i moderni znanstveni
rad u Hrvatskoj zakljucio je autor.
Na kraju valja istaci da u pogledu sustavnosti, detaljnosti i cjelokupnosti ova
knjiga nadilazi povijesno-znanstveni i povijesno-filozofski pristup. To je vrhunski prikaz
nastanka egzaktne novovjeke znanosti u Hrvata u spomenutom razdoblju. Zbog toga je
knjiga ne samo nezaobilazna literatura u proucavanju te tematike, nego je jedinstvena u
pogledu sintetickog pristupa. Nakladniku pripada zasluga s to je izdavanjem ove knjige
uvelike obogatio istrazivanje prave povijesne istine naseg naroda.
Branko Hanzek, Zagreb


PAZNJA!
STARI BROJEVI U nasem skladistu ima starih brojeva, i to: god.
XVI, br. 4; god. XXXII, br. 3; god. XXXIII, br. 4; god. XXXIV, br. 3, 4; god. XXXV,
br. 3; god. XXXVI, br. 1, 2, 3, 4; god. XXXVII, br. 1, 4; god. XXXIX, br. 1, 2, 3, 4;
god. XL, br. 2, 3, 4; god. XLI, br. 1, 2, 3, 4; god. XLII, br. 3-4; god. XLIV, br. 1, 2, 3,
4; god. XLV, br. 1, 2, 3, 4; god. XLVI, br. 1, 2, 3, 4; god. XLVII, br. 1, 2, 3, 4; god.
XLVIII, br. 1, 2, 3, 4; god. XLIX, br. 1, 2, 3, 4; god. L, br. 1, 2, 3, 4; god. LI, br. 1, 2,
3, 4; god. LII, br. 1, 2, 3, 4; god. LIII, br. 1, 2, 3, 4; god. LIV, br. 1, 2, 3, 4; god. LV,
br. 1, 2, 3, 4; god. LVI, br. 1, 2, 3, 4; god. LVII, br. 1, 2, 3, 4; god. LVIII, br. 1, 2, 3, 4.
Cijena pojedinog broja je 20 kuna.
Izvanredni broj (E) zadaci iz matematike (cijena 20 kn); Izvanredni broj (F)
Rjecnik matematickih naziva hrvatski, engleski, njemacki (cijena 30 kn); Izvanredni
broj (H) zadaci iz matematike (cijena 20 kn).
Matematicko-fizicki list, LIX 2 (2008. 2009.)

137

Razredbeni ispit na FOI-u iz matematike 2008. g.


Donosimo zadatke s razredbenog ispita na Sveucilistu u Zagrebu, Fakultet organizacije
i informatike, Varazdin, 2008. g. Do 2008. godine bilo je po 20 zadataka iz matematike,
informatike i hrvatskog jezika, a od 2009. godine ce se sastojati od 30 zadataka iz
matematike i 30 zadataka iz informatike. Svaki tocno rijesen zadatak nosi 10 bodova
i nema negativnih bodova za netocno rijesene zadatke. U svakom zadatku ima 4
ponudena
odgovora od kojih je jedan tocan. Za prijelaz praga treba skupiti najmanje
200 bodova, ali mora biti ispunjen i uvjet da je 100 bodova minimalno iz matematike
i 100 iz informatike. Dakle, ako bi pristupnik imao samo 200 bodova i to samo iz
informatike, on ne bi presao prag. Vise informacija o upisima na FOI, mozete naci na
http://www.foi.hr/upisi/. Ovdje donosimo 60 zadataka iz matematike, koji su bili
na razredbenom ispitu 2008. godine.

Grupa A

1. Algebarski razlomak
jednak

xy + y2
x+
x+y

 

1 + a3 1 a + a2
:

za |x| = |y| je
x2 y2
xy

a
1a
D.
x+y
xy
2. Ostatak pri dijeljenju polinoma p(x) = x10 + 2x5 + 1 s polinomom q(x) = x + 1
jednak je
A. x
B. 2
C. 0
D. 1
2i
3. Apsolutna vrijednost kompleksnog broja
jednaka je
3 2i

65
13
B. 4
C. 13
A.
D.
13
4
4. Rjesenje nejednadzbe log (x + 2)  1 je
A. [2, 8]
B.  , 2]
C.  , 8]
D.  2, 8]
5. Ako je g(x) = 2x 8 i h(x) = x2 2 , tada je g(h(2)) jednako
A. 4
B. 4
C. x
D. 2
6. Neka je f (x) = |x + 3| 4 . Minimalna vrijednost funkcije f je
A. 2
B. 4
C. 3
D. 3
7. Koliko realnih rjesenja ima jednadzba x3 + 3x2 x 3 = 0 ?
A. 1
B. 2
C. 3
D. 4
8. Rjesenje kvadratne nejednadzbe x2 + 5x + 6  0 je
A. [1, 6]
B.  , 6]
C. [1, +
D.  0, +
9. Vrhovi c etverokuta su redom tocke A(3, 2), B(1, 2), C(2, 5), D(4, 1). Taj
c etverokut je
A. kvadrat
B. pravokutnik
C. paralelogram
D. romb
A. 1 + a

138

B. x + y

C.

Matematicko-fizicki list, LIX 2 (2008. 2009.)

10. Ako se radijus sfere udvostruci, s to c e se dogoditi s oplosjem sfere?


A. porast c e za faktor 2
B. porast c e za faktor 4
C. porast c e za faktor 6
D. porast c e za faktor 8
11. Volumen uspravnog stosca je 12 , a polumjer baze r = 3 . Visina stosca je
A. 1
B. 2
C. 3
D. 4
12. Mate i Sanja su udaljeni 2000 m i oboje gledaju balon. U isto vrijeme, Sanja vidi
balon pod kutem od 42 , a Mate pod kutem od 70 . Sanja je udaljena od balona
A. 2026.98 m
B. 1984.37 m
C. 1443.36 m
D. 2086.95 m
13. Opseg paralelograma na slici je 80 cm . Povrsina tog paralelograma je

A. 276 cm2
B. 144 cm2
C. 138 cm2
14. Zbroj rjesenja jednadzbe 22x + 2 2x 8 = 0 je
A. 2

B. 1

C. 4

D. 84 cm2
D.

2
3

15. Krivulje y = x2 + 2 i y = 4x se sijeku u


A. I. kvadrantu
B. II. kvadrantu
C. III. kvadrantu
D. IV. kvadrantu
3
16. Srediste i radijus kruznice x2 + x + y2 2y = je
 1 4
1

B. S , 1 , r = 2
A. S , 1 , r = 2
2
2

 1
 1
3
D. S , 1 , r = 2
C. S , 1 , r =
2
4
2

17. Domena funkcije f (x) = ax 1 za 0 < a < 1 je


A.  , 0]
B. [0, +
C.  1, +
D.  , 1]

n cos n
2 jednak je

18. Cetvrti
clan niza an =
n!
1
1
1
1
A.
B.
C.
D.
2
4
6
8
19. Odredite realni parametar a tako da su zadani pravci ax + (a 1)y = 5 i
x + ay = 0 okomiti.
A. 0
B. 2
C. 3
D. 1
20. Omjer povrsina baza krnje piramide jednak je
A. omjeru visine krnje piramide i visine cijele piramide
B. volumenu krnje piramide
C. omjeru kvadrata njihovih udaljenosti od vrha potpune piramide
D. omjeru njihovih udaljenosti od vrha potpune piramide

Matematicko-fizicki list, LIX 2 (2008. 2009.)

139

Grupa B
a3 8b3 a2 + 2ab + 4b2
je jednak
:
a2 4b2
a2 + 2ab
A. 1
B. a
C. a + 2b
D. 1 ab



6
3
2. Izraz 3x2 y : 9xy2 27x4 y3 je jednak

A. 3x2 y
B. 6 3x2 y3
C. 3 xy
D. 9xy2
1
je jednak
3. Imaginarni dio kompleksnog broja
1 + 2i
1
2
5
A. 0
B.
C.
D.
2
5
2
2
4. Kakav treba biti realni parametar a tako da kvadratna jednadzba 3x ax 4 = 0
nema realnihrjesenja?

A. a  4 3, 4 3
B. a  , 4 3
C. a Re
D. a [ 48, 0]
5. Luka je ispucao loptu. Visina u metrima h(t) koju dostize lopta nakon t sekundi
je h(t) = 16t2 + 48t . Lopta ce udariti u tlo nakon
25
A. 1.5 s
B. 2 s
C.
s
D. 3 s
8
6. Ako je f (x) = 3x2 i g(x) = x2 x , tada je g(f (3)) jednako
A. 6
B. 3
C. 2
D. 4
2
x 7x + 10
> 0 je
7. Rjesenje nejednadzbe
x4
A. x  2, 4  5, 
B. x  , 5
C. x  4, 5
D. x [5, +
8. Rjesenje jednadzbe log5 (x 4) + log5 (x + 2) = log5 16 pripada intervalu
A. [0, 4]
B.  4, 10]
C.  , 0
D.  10, +

9. Cijena iznajmljivanja bicikla najprije je povecana 25%, a zatim snizena 22%. Sto
treba napraviti s trenutnom cijenom da opet bude jednaka pocetnoj?
A. Povecati je 3%
B. Sniziti je 3%
C. Povecati je 2.56%
D. Sniziti je 2.56%
1. Algebarski izraz

x2
je
x+1
A.  , 1  1, +
B.  1, 0
C.  , 1]
11. Nadite povrsinu osjencanog kruznog odsjecka.
10. Domena funkcije f (x) = log2

D.  1, + \ {0}

A. 17.01 cm2
B. 18.27 cm2
C. 18.53 cm2
D. 19.34 cm2
2
12. Odredite koordinate sredista S i duljinu polumjera r kruznice x + y2 + 8x +
2y + 13 = 0 .
A. S(4, 1), r = 2 B. S(4, 1), r = 2 C. S(4, 1), r = 4 D. S(8, 2), r = 13

140

Matematicko-fizicki list, LIX 2 (2008. 2009.)

13. Polinom drugog stupnja koji prolazi tockama A(1, 6), B(0, 3) i C(1, 4)
je polinom
B. f (x) = 5x2 + 3x + 5
A. f (x) = 6x2 3x 4
C. f (x) = 2x2 + x 3
D. f (x) = 3x2 + x 2
14. Kakav trokut odreduju
pravci y = 3x 1 , x 7y 7 = 0 i x + y 7 = 0 ?
A. pravokutan
B. tupokutan
C. jednakokracan
D. trokut sa sve tri stranice razlicitih duljina
15. Ako je oplosje valjka 8 cm2 , a polumjer baze jednak visini valjka, tada je
volumen valjka jednak

A. 2 cm3
B. 2 cm3
C. 8 cm3
D. 2 2 cm3
16. Formula za volumen V = B v, gdje je B povrsina baze, a v visina tijela, vrijedi
za
A. prizmu i valjak
B. stozac i kvadar
C. piramidu i kuglu
D. paralelepiped i krnju piramidu
17. Na koliko se nacina 5 ljudi moze posjesti oko okruglog stola?
A. 120
B. 24
C. 20
D. 48
cos x
+ tg x je jednak
18. Izraz
1 + sin x
1
D. ctg x
A. tg x
B. cos x
C.
cos x
19. U kakvom su odnosu pravac y = 2x + 9 i parabola y2 = 3x ?
A. pravac je normala parabole
B. pravac je tangenta parabole
C. pravac ne sijece parabolu
D. pravac sijece parabolu u ishodistu

20. Treci clan geometrijskog


niza kojemu su prva dva clana 2 i 4 je jednak
A. 8
B. 4 2
C. 6
D. 8 2

Grupa C

1. Izraz
A.

a+2
a2

a(a + 2)
je jednak
(a + 1)4 1
1
B. 2
a +2

C. 1

D.

1
a2 + 2a + 2

3+2 3
2. Racionalizacijom nazivnika u izrazu
dobije se
3+2

B. 3
C. 2 3
D. 3 + 2
A. 3
3. Broj rjesenja nejednadzbe x2 5x + 6  0 u skupu Z je
A. 1
B. 2
C. 3
D. 4
4. Otopina soli A mijesa se sa 16% otopinom soli B u omjeru 3 : 4 i dobije se 22%
otopina. Postotak soli u otopini A iznosi
A. 20%
B. 24%
C. 28%
D. 30%
3 3 7
2 5 9
5. Nakon s to se podijele potencije (21x a b) : (7xa b ) dobije se
37 x12 a3
73 x12 a
214x6 a
21
A. 2 38
abx
D. 7 28
B.
C.
16
7 b
9b
7
3b
Matematicko-fizicki list, LIX 2 (2008. 2009.)

141

6. Polinom p(x) = x4 + 3x3 + 4x2 + x + a je djeljiv s q(x) = x2 + x 1 ako je a


jednako
A. 3
B. 3
C. 4
D. 4
 1 + i 4
7. Imaginarni dio kompleksnog broja
je jednak
1i
A. 1
B. 1
C. 2
D. 0
8. Ordinata tocke na osi y koja je jednako udaljena od tocaka A(1, 5) i B(7, 3) je
jednaka
A. 2
B. 3
C. 0
D. 2
x+1
 2 je
9. Skup svih rjesenja nejednadzbe
x1
A.  1, 3]
B. [1, +
C.  , 1 [3, +
D. R
10. Broj realnih rjesenja jednadzbe log x4 + log (x + 4)4 = 4 log 3 je
A. 1
B. 2
C. 3
D. 4
11. Napisite preslikavanje u obliku F(x, y) = (?, ?) tako da opisuje transformaciju
koja tocku (x, y) osno simetricno preslikava s obzirom na os y.
A. F(x, y) = (x, y) B. F(x, y) = (y, x) C. F(x, y) = (x, y) D. F(x, y) = (x, y)
12. Na slici je graf funkcije f (x) = logb x . Odredite b .

A. b = 2
B. b = 4
C. b = 8
D. b = 16
2
2
13. Radijus kruznice x + 2x
+ y 6y = 0 je

A. 10
B. 10
C. 6
D. 2
14. Opseg baze pravilne
uspravne c etverostrane piramide je 24 cm, a povrsina
dijagonalnog presjeka je 3 14 cm2 . Volumen piramide
je

A. 72 cm3
B. 36 cm3
C. 12 7 cm3
D. 4 14 cm3
15. Hipotenuza pravokutnog trokuta iznosi c = 13 cm , a kut pri vrhu A je = 30 .
Povrsinatrokuta je

169 3
cm2
A.
B. 169 cm2
C. 13 3 cm2
D. 144 3 cm2
8
16. Tocka presjeka pravaca 2x + 4y = 3 i 3x y = 0 se nalazi na
A. osi x
B. pravcu 7x + 7y 6 = 0
C. kruznici x2 + y2 = 1
D. osi y
2
2
2
17. Zajednicka tetiva krivulja
y
=
8x
i
x

y
=
9
ima
duljinu

A. 4
B. 3 6
C. 12
D. 12 2
18. Rjesenje jednadzbe 3 2x 2x1 = 20 zadovoljava uvjet
5
9
9
5
C. < x 
D. x >
A. x < 0
B. 0 < x 
2
2
2
2
19. Odredite istinitu tvrdnju.
A. Funkcija f (x) = log x poprima samo pozitivne realne vrijednosti.
inverzne.
B. Funkcije g(x) = log2 x i h(x) = 2x su medusobno
x
C. Jednadzba a = 0 (a > 0) ima uvijek dva realna rjesenja.
D. Os x je asimptota funkcije f (x) = log x .

142

Matematicko-fizicki list, LIX 2 (2008. 2009.)


2
, cos 2x jednako je
20. Ako je cos x =
4
4
3
B.
A.
4
5

C.

4
3

D.

4
5

Rjesenja zadataka
Grupa A
1. A 2. C 3. A 4. D 5. A 6. B 7. C 8. A 9. C 10. B
11. D 12. A 13. A 14. B 15. B 16. B 17. A 18. C 19. A 20. C
Grupa B
1. B 2. A 3. C 4. A 5. D 6. A 7. A 8. B 9. C 10. D
11. B 12. A 13. C 14. C 15. D 16. A 17. B 18. C 19. A 20. D
Grupa C
1. D 2. A 3. B 4. D 5. A 6. B 7. D 8. A 9. C 10. D
11. A 12. A 13. B 14. C 15. A 16. B 17. D 18. C 19. B 20. A

Priredili Blazenka Divjak i Damir Horvat


SVIM SURADNICIMA
U Matematickofizickom listu objavljuju se clanci iz matematike, fizike i informatike, s
malim prilogom iz astronomije, zadaci i rjesenja, prikazi natjecanja i ljetnih s kola iz matematike
i fizike, zanimljivosti u obliku clanaka i zadataka od ucenika, profesora i ostalih matematicara,
novosti iz znanosti, zadaci s razredbenih (kvalifikacijskih) ispita, zabavna matematika i nagradni
natjecaj.
Prilozi trebaju biti napisani racunalom (Word, Tex, Latex) ili pisacim strojem sa s irokim
proredom na formatu A-4. Uz kopiju posaljite i disketu.
Slike trebaju biti jasno nacrtane na posebnom papiru i pogodne za presnimavanje. Slike crtane
racunalom (eps, tif, gif, jpg i sl.) posaljite i na disketi.

Clanci
neka ne budu dulji od osam stranica, a ako je to potrebno neka budu napisani u
nastavcima.
Pozivaju se ucenici da posalju clanak o nekoj od spomenutih tema, originalne zadatke s
rjesenjima ili prikaze nekih manifestacija (ljetne s kole, susreti ucenika, rad s kolske grupe).
Kako se rukopisi ne vracaju, sacuvajte original a posaljite kopiju na papiru formata A-4.
Svi rukopisi podlijezu recenziji redakcije ili neke strucne osobe za odredeno
podrucje.
Prilozi se s alju na adresu ovog casopisa koja je na prvoj stranici lista.

RJESAVATELJIMA
ZADATAKA
Svako rjesenje neka bude napisano na posebnom papiru (formata A-4 ili A-5) i to samo
na jednoj strani papira. Uz svako rjesenje na vrhu papira treba potpuno ispisati tekst zadatka.
Svako rjesenje treba citljivo potpisati (ime i prezime), naznaciti razred, s kolu i mjesto.
Matematicko-fizicki list, LIX 2 (2008. 2009.)

143

Rjesenje nagradnog natjecaja br. 183

Rjesenje. Koristeci binomnu formulu dobivamo


!
!
!
!
100
100
100
2 100
100 100
=
+2
+2
(1 + 2)
+ ... + 2
,
0
1
2
100
tj.

xy = 3100 = (32 )50 = (34 )25 = (35 )20 = (325 )4 = (310 )10 = (320 )5 = (350 )2 ,

odakle je (x, y) {(3, 100), (32 , 50), (34 , 25), (35 , 20), (325 , 4), (310 , 10), (320 , 5), (350 , 2)} .
Knjigom su nagradeni
rjesavatelji:
1. Edin Ajanovic (3), Prva bosnjacka gimnazija, Sarajevo, BiH;
2. Kristijan Kvaternik (1), V. gimnazija, Zagreb.

Rijesili zadatke iz br. 4/232


(Broj u zagradi oznacava razredgodiste srednjeosnovne s kole.)
a) Iz matematike: Edin Ajanovic (3), I. bosnjacka gimnazija, Sarajevo, BiH, 31053108,
31103113, 3115, 3117, 3118; Davor Devald (1), I. prirodoslovno-matematicka gimnazija, SS
Izidora Krsnjavoga, Nasice, 31053108, 3110, 3116; Kristijan Kvaternik (1), V. gimnazija,
Zagreb, 31053108; Vanja Ubovic (2), Gimnazija Petra Preradovica, Virovitica, 31063108,
3112, 3115.

b) Iz fizike: Ana Lucija Alviz (7), OS Fausta Vrancica, Sibenik,


278; Filip Dunaj (8),
OS Augusta Cesarca, Krapina 278, 280; Arijan Golub (8), OS Augusta Cesarca, Krapina 278,
280; Juraj Krsnik (8), OS Augusta Cesarca, Krapina 278, 280; Dorian Lacko (8), OS Augusta

Cesarca, Krapina 278, 280; Ana Paic (7), OS Fausta Vrancica, Sibenik
278281; Karlo Radecic

(7), OS Fausta Vrancica, Sibenik,


278, 280; Kristijan Kvaternik (1), V. gimnazija, Zagreb, 1392,
1395, 1396.

Nagradni natjecaj br. 185


Dokazi da je broj

. . 111 22 . 
. . 222 5
11 . 
2008

2009

potpun kvadrat.

144

Matematicko-fizicki list, LIX 2 (2008. 2009.)

You might also like